sample mbe ii

103

Click here to load reader

Upload: trancong

Post on 21-Dec-2016

379 views

Category:

Documents


71 download

TRANSCRIPT

Page 1: Sample MBE II

Sample MBE IIJuly 1991

®

SampleMBEII.qxp 1/29/2010 10:05 AM Page 1

Page 2: Sample MBE II

PREFACE

The Multistate Bar Examination (MBE) is an objective six-hourexamination developed by the National Conference of BarExaminers (NCBE) that contains 200 questions. It was firstadministered in February 1972, and is currently a component of thebar examination in most U.S. jurisdictions.

From time to time NCBE releases test questions to acquaint test-takers with authentic test materials. This publication consists of theactual 200-item, multiple-choice test that was administerednationally in July 1991.

The July 1991 MBE consisted of questions in the following areas:Constitutional Law, Contracts, Criminal Law and Procedure,Evidence, Real Property, and Torts. Applicants were directed tochoose the best answer from four stated alternatives.

The purpose of this publication is to familiarize you with theformat and nature of MBE questions. The questions in thispublication should not be used for substantive preparation forthe MBE. Because of changes in the law since the time theexamination was administered, the questions and their keysmay no longer be current. The editorial style of questions mayhave changed over time as well.

Applicants are encouraged to use as additional study aids theMBE Online Practice Exams 1 and 2 (MBE OPE 1 and OPE 2),both available for purchase online at www.ncbex2.org/catalog.These study aids, which include explanations for each optionselected, contain questions from more recently administeredMBEs that more accurately represent the current content andformat of the MBE.

If you use the questions in this publication as a practice exam, youshould not rely on your raw score to identify how well you aredoing. MBE raw scores are converted to scaled scores through anequating procedure that is designed to ensure that the level ofdifficulty of the examination remains consistent fromadministration to administration. The Raw Score Conversion Tableshould be used to estimate your scaled score.

Additional copies of this publication may be purchased from theNational Conference of Bar Examiners, 302 South Bedford Street,Madison, Wisconsin 53703-3622; (608) 280-8550. To orderadditional copies of this publication or other National Conferencepublications, visit our website at www.ncbex.org.

Copyright © 2002, 2004 by the National Conference of Bar Examiners.All rights reserved.

Page 3: Sample MBE II

SAMPLE MULTISTATE BAR EXAMINATION

TABLE OF CONTENTS

AM Book . . . . . . . . . . . . . . . . . . . . . . . . . . . . . . . . . . . . . . 2

PM Book . . . . . . . . . . . . . . . . . . . . . . . . . . . . . . . . . . . . . 50

Answer Key . . . . . . . . . . . . . . . . . . . . . . . . . . . . . . . . . . . 97

Raw Score Conversion Table . . . . . . . . . . . . . . . . . . . . . 99

Sample Answer Sheet . . . . . . . . . . . . . . . . . . . . . . . . . . 100

Page 4: Sample MBE II

1. By warranty deed, Marta conveyed Blackacreto Beth and Christine “as joint tenants withright of survivorship.” Beth and Christine arenot related. Beth conveyed all her interest toEugenio by warranty deed and subsequentlydied intestate. Thereafter, Christine conveyedto Darin by warranty deed.

There is no applicable statute, and thejurisdiction recognizes the common-law jointtenancy.

Title to Blackacre is in

(A) Darin.(B) Marta.(C) Darin and Eugenio.(D) Darin and the heirs of Beth.

2. Peavey was walking peacefully along a publicstreet when he encountered Dorwin, whom hehad never seen before. Without provocation orwarning, Dorwin picked up a rock and struckPeavey with it. It was later established thatDorwin was mentally ill and suffered recurrenthallucinations.

If Peavey asserts a claim against Dorwin basedon battery, which of the following, ifsupported by evidence, will be Dorwin’s bestdefense?

(A) Dorwin did not understand that his actwas wrongful.

(B) Dorwin did not desire to cause harm toPeavey.

(C) Dorwin did not know that he wasstriking a person.

(D) Dorwin thought Peavey was about toattack him.

GO ON TO THE NEXT PAGE.

-2-

AM BOOKTIME—3 HOURS

Directions: Each of the questions or incomplete statements below is followed by four suggested answers orcompletions. You are to choose the best of the stated alternatives. Answer all questions according to thegenerally accepted view, except where otherwise noted.

For the purposes of this test, you are to assume that Articles 1 and 2 of the Uniform Commercial Code havebeen adopted. You are also to assume relevant application of Article 9 of the UCC concerning fixtures. TheFederal Rules of Evidence are deemed to control. The terms “Constitution,” “constitutional,” and “unconstitutional”refer to the federal Constitution unless indicated to the contrary. You are also to assume that there is noapplicable community property law, no guest statute, and no No-Fault Insurance Act unless otherwisespecified. In negligence cases, if fault on the claimant’s part is or may be relevant, the statement of facts forthe particular question will identify the contributory or comparative negligence rule that is to be applied.

Page 5: Sample MBE II

3. Penstock owned a large tract of land on theshore of a lake. Drury lived on a stream thatran along one boundary of Penstock’s land andinto the lake. At some time in the past, achannel had been cut across Penstock’s landfrom the stream to the lake at a point somedistance from the mouth of the stream. Fromwhere Drury lived, the channel served as aconvenient shortcut to the lake. Erroneouslybelieving that the channel was a publicwaterway, Drury made frequent trips throughthe channel in his motorboat. His use of thechannel caused no harm to the land throughwhich it passed.

If Penstock asserts a claim for damagesagainst Drury based on trespass, which of thefollowing would be a correct disposition of thecase?

(A) Judgment for Penstock for nominaldamages, because Drury intentionallyused the channel.

(B) Judgment for Drury, if he did not use thechannel after learning of Penstock’sownership claim.

(C) Judgment for Drury, because he causedno harm to Penstock’s land.

(D) Judgment for Drury, because when heused the channel he believed it was apublic waterway.

Questions 4-5 are based on the following fact situation.

Structo contracted with Bailey to construct for$500,000 a warehouse and an access driveway athighway level. Shortly after commencing work onthe driveway, which required for the specified levelsome excavation and removal of surface material,Structo unexpectedly encountered a large mass ofsolid rock.

4. For this question only, assume the followingfacts. Structo informed Bailey (accurately) thatbecause of the rock the driveway as specifiedwould cost at least $20,000 more than figured,and demanded for that reason a total contractprice of $520,000. Since Bailey was expectingwarehousing customers immediately after theagreed completion date, he signed a writingpromising to pay the additional $20,000.Following timely completion of the warehouseand driveway, which conformed to the contractin all respects, Bailey refused to pay Structomore than $500,000.

What is the maximum amount to whichStructo is entitled?

(A) $500,000, because there was noconsideration for Bailey’s promise to paythe additional $20,000.

(B) $500,000, because Bailey’s promise topay the additional $20,000 was exactedunder duress.

(C) $520,000, because the modification wasfair and was made in the light ofcircumstances not anticipated by theparties when the original contract wasmade.

(D) $520,000, provided that the reasonablevalue of Structo’s total performance wasthat much or more.

GO ON TO THE NEXT PAGE.

-3-

Page 6: Sample MBE II

-4-

5. For this question only, assume the followingfacts. Upon encountering the rock formation,Structo, instead of incurring additional costs toremove it, built the access driveway over therock with a steep grade down to the highway.Bailey, who was out of town for several days,was unaware of this nonconformity until thedriveway had been finished. As built, it is toosteep to be used safely by trucks or cars,particularly in the wet or icy weatherfrequently occurring in the area. It would cost$30,000 to tear out and rebuild the driveway athighway level. As built, the warehouse,including the driveway, has a fair market valueof $550,000. Bailey has paid $470,000 toStructo, but refuses to pay more because of thenonconforming driveway, which Structo hasrefused to tear out and rebuild.

If Structo sues Bailey for monetary relief,what is the maximum amount Structo isentitled to recover?

(A) $30,000, because the fair market value ofthe warehouse and driveway “as is”exceeds the contract price by $50,000(more than the cost of correcting thedriveway).

(B) $30,000, because Structo substantiallyperformed and the cost of correcting thedriveway would involve economic waste.

(C) $30,000, minus whatever amount Structosaved by not building the driveway at thespecified level.

(D) Nothing, because Bailey is entitled todamages for the cost of correcting thedriveway.

6. Larson was charged with the murder of a manwho had been strangled and whose body wasfound in some woods near his home. Larsonsuffers from a neurological problem thatmakes it impossible for him to remember anoccurrence for longer than 48 hours.

After Larson was charged, the police visitedhim and asked if they might search his home.Larson consented. The police found a diarywritten by Larson. An entry dated the sameday as the victim’s disappearance read,“Indescribable excitement. Why did no oneever tell me that killing gave such pleasure tothe master?”

Larson was charged with murder. His attorneyhas moved to exclude the diary from evidenceon the ground that its admission would violateLarson’s privilege against self-incrimination.Counsel has also argued that Larson could notgive informed consent to the search becausemore than 48 hours had passed since themaking of the entry and hence he could notremember the existence of the incriminatingentry at the time he gave his consent. There isno evidence that the police officers whosecured Larson’s consent to the search wereaware of his memory impairment.

With regard to the diary, the court should

(A) admit it, because Larson’s consent wasnot obtained by intentional policemisconduct and Larson was notcompelled to make the diary entry.

(B) admit it, pursuant to the good-faithexception to the exclusionary rule.

(C) exclude it, because Larson was notcompetent to consent to a search.

(D) exclude it, because use of the diary asevidence would violate Larson’sprivilege against self-incrimination.

GO ON TO THE NEXT PAGE.

Page 7: Sample MBE II

7. In contract litigation between Pixley and Dill,a fact of consequence to the determination ofthe action is whether Pixley provided Dill witha required notice at Dill’s branch office “in thestate capital.” Pixley introduced evidence thathe gave notice at Dill’s office in the city ofCapitan. Although Capitan is the state’scapital, Pixley failed to offer proof of that fact.

Which of the following statements is mostclearly correct with respect to possible judicialnotice of the fact that Capitan is the state’scapital?

(A) The court may take judicial notice eventhough Pixley does not request it.

(B) The court may take judicial notice only ifPixley provides the court with anauthenticated copy of the statute thatdesignates Capitan as the capital.

(C) If the court takes judicial notice, theburden of persuasion on the issue ofwhether Capitan is the capital shifts toDill.

(D) If the court takes judicial notice, itshould instruct the jury that it may, but isnot required to, accept as conclusive thefact that Capitan is the capital.

8. A statute of the state of East Dakota requireseach insurance company that offers burglaryinsurance policies in the state to charge auniform rate for such insurance to all of itscustomers residing within the same county inthat state. So long as it complies with thisrequirement, a company is free to chargewhatever rate the market will bear for itsburglary insurance policies.

An insurance company located in the state ofEast Dakota files suit in federal district courtagainst appropriate East Dakota state officialsto challenge this statute on constitutionalgrounds. The insurance company wishes tocharge customers residing within the samecounty in East Dakota rates for burglaryinsurance policies that will vary because theywould be based on the specific nature of thecustomer’s business, on its precise location,and on its past claims record.

In this suit, the court should

(A) hold the statute unconstitutional, becausethe statute deprives the insurancecompany of its liberty or propertywithout due process of law.

(B) hold the statute unconstitutional, becausethe statute imposes an undue burden oninterstate commerce.

(C) hold the statute constitutional, becausethe statute is a reasonable exercise of thestate’s police power.

(D) abstain from ruling on the merits of thiscase until the state courts have had anopportunity to pass on theconstitutionality of this state statute.

GO ON TO THE NEXT PAGE.

-5-

Page 8: Sample MBE II

9. Dawson was charged with felony murderbecause of his involvement in a bank robbery.The evidence at trial disclosed that Smithinvited Dawson to go for a ride in his new car,and after a while asked Dawson to drive. AsSmith and Dawson drove around town, Smithexplained to Dawson that he planned to robthe bank and that he needed Dawson to drivethe getaway car. Dawson agreed to drive to thebank and to wait outside while Smith went into rob it. As they approached the bank,Dawson began to regret his agreement to helpwith the robbery. Once there, Smith got out ofthe car. As Smith went out of sight inside thebank, Dawson drove away and went home.Inside the bank, Smith killed a bank guardwho tried to prevent him from leaving with themoney. Smith ran outside and, finding that hiscar and Dawson were gone, ran down an alley.He was apprehended a few blocks away.Dawson later turned himself in after hearingon the radio that Smith had killed the guard.

The jurisdiction has a death penalty thatapplies to felony murder.

Consistent with the law and the Constitution,the jury may convict Dawson of

(A) felony murder and impose the deathpenalty.

(B) felony murder but not impose the deathpenalty.

(C) bank robbery only.(D) no crime.

10. In an automobile negligence action by Popkinagainst Dwyer, Juilliard testified for Popkin.Dwyer later called Watts, who testified thatJuilliard’s reputation for truthfulness was bad.

On cross-examination of Watts, Popkin’scounsel asks, “Isn’t it a fact that when youbought your new car last year, you made afalse affidavit to escape paying the sales tax?”

This question is

(A) proper, because it will indicate Watts’sstandard of judgment as to reputation fortruthfulness.

(B) proper, because it bears on Watts’scredibility.

(C) improper, because character cannot beproved by specific instances of conduct.

(D) improper, because one cannot impeachan impeaching witness.

11. David built in his backyard a garage thatencroached two feet across the property lineonto property owned by his neighbor,Prudence. Thereafter, David sold his propertyto Drake. Prudence was unaware, prior toDavid’s sale to Drake, of the encroachment ofthe garage onto her property. When shethereafter learned of the encroachment, shesued David for damages for trespass.

In this action, will Prudence prevail?

(A) No, unless David was aware of theencroachment when the garage was built.

(B) No, because David no longer owns orpossesses the garage.

(C) Yes, because David knew where thegarage was located, whether or not heknew where the property line was.

(D) Yes, unless Drake was aware of theencroachment when he purchased theproperty.

GO ON TO THE NEXT PAGE.

-6-

Page 9: Sample MBE II

-7-

12. Poole sued Darrel for unlawfully using Poole’sidea for an animal robot as a character inDarrel’s science fiction movie. Darrel admittedthat he had received a model of an animalrobot from Poole, but he denied that it had anysubstantial similarity to the movie character.After the model had been returned to Poole,Poole destroyed it.

In order for Poole to testify to the appearanceof the model, Poole

(A) must show that he did not destroy themodel in bad faith.

(B) must give advance notice of his intent tointroduce the oral testimony.

(C) must introduce a photograph of themodel if one exists.

(D) need do none of the above, because the“best evidence rule” applies only towritings, recordings, and photographs.

13. Lanny, the owner of Whiteacre in fee simple,leased Whiteacre to Teri for a term of tenyears by properly executed written instrument.The lease was promptly and properly recorded.It contained an option for Teri to purchaseWhiteacre by tendering $250,000 as purchaseprice any time “during the term of this lease.”One year later, Teri, by a properly executedwritten instrument, purported to assign theoption to Oscar, expressly retaining all of theremaining term of the lease. The instrument ofassignment was promptly and properlyrecorded.

Two years later, Lanny contracted to sellWhiteacre to Jones and to convey amarketable title “subject to the rights of Teriunder her lease.” Jones refused to closebecause of the outstanding option assigned toOscar.

Lanny brought an appropriate action againstJones for specific performance.

If judgment is rendered in favor of Lanny, itwill be because the relevant jurisdiction hasadopted a rule on a key issue as to whichvarious state courts have split.

Which of the following identifies thedeterminative rule or doctrine upon which thesplit occurs, and states the position favorableto Lanny?

(A) In a contract to buy, any form of “subjectto a lease” clause that fails to mentionexpressly an existing option means thatthe seller is agreeing to sell free andclear of any option originally included inthe lease.

(B) Marketable title can be conveyed so longas any outstanding option not mentionedin the purchase contract has not yet beenexercised.

(C) Options to purchase by lessees aresubject to the Rule Against Perpetuities.

(D) Options to purchase contained in a leasecannot be assigned separately from thelease.

GO ON TO THE NEXT PAGE.

Page 10: Sample MBE II

14. Daniel and a group of his friends are fanaticalbasketball fans who regularly meet at eachothers’ homes to watch basketball games ontelevision. Some of the group are fans of teamA, and others are fans of team B. When thegroup has watched televised games betweenthese two teams, fights sometimes havebroken out among the group. Despite this fact,Daniel invited the group to his home to watcha championship game between teams A and B.

During the game, Daniel’s guests becamerowdy and antagonistic. Fearing that theywould begin to fight, and that a fight woulddamage his possessions, Daniel asked hisguests to leave. They refused to go and soonbegan to fight. Daniel called the police, andOfficer was sent to Daniel’s home. Officersustained a broken nose in his efforts to stopthe fighting.

Officer brought an action against Danielalleging that Daniel was negligent in invitingthe group to his house to watch thischampionship game. Daniel has moved todismiss the complaint.

The best argument in support of this motionwould be that

(A) a rescuer injured while attempting toavert a danger cannot recover damagesfrom the endangered person.

(B) a police officer is not entitled to arecovery based upon the negligentconduct that created the need for theofficer’s professional intervention.

(C) as a matter of law, Daniel’s conduct wasnot the proximate cause of Officer’sinjury.

(D) Daniel did not owe Officer a duty to usereasonable care, because Officer was amere licensee on Daniel’s property.

15. In a prosecution of Drew for forgery, thedefense objects to the testimony of West, agovernment expert, on the ground ofinadequate qualifications. The governmentseeks to introduce a letter from the expert’sformer criminology professor, stating thatWest is generally acknowledged in his field aswell qualified.

On the issue of the expert’s qualifications, theletter may be considered by

(A) the jury, without regard to the hearsayrule.

(B) the judge, without regard to the hearsayrule.

(C) neither the judge nor the jury, because itis hearsay not within any exception.

(D) both the judge and the jury, because theletter is not offered for a hearsaypurpose.

GO ON TO THE NEXT PAGE.

-8-

Page 11: Sample MBE II

Questions 16-17 are based on the following fact situation.

Responding to County’s written advertisement forbids, Tyres was the successful bidder for the sale oftires to County for County’s vehicles. Tyres andCounty entered into a signed, written agreement thatspecified, “It is agreed that Tyres will deliver alltires required by this agreement to County, inaccordance with the attached bid form andspecifications, for a one-year period beginningSeptember 1, 1990.” Attached to the agreement wasa copy of the bid form and specifications. In thewritten advertisement to which Tyres hadresponded, but not in the bid form, County hadstated, “Multiple awards may be issued if they arein the best interests of County.” No definite quantityof tires to be bought by County from Tyres wasspecified in any of these documents.

In January 1991, Tyres learned that County wasbuying some of its tires from one of Tyres’scompetitors. Contending that the Tyres-Countyagreement was a requirements contract, Tyres suedCounty for the damages caused by County’s buyingsome of its tires from the competitor.

16. If County defends by offering proof of theadvertisement concerning the possibility ofmultiple awards, should the court admit theevidence?

(A) Yes, because the provision in the writtenagreement, “all tires required by thisagreement,” is ambiguous.

(B) Yes, because the advertisement was inwriting.

(C) No, because of the parol evidence rule.(D) No, because it would make the contract

illusory.

17. If the court concludes that the Tyres-Countycontract is an agreement by County to buy itstire requirements from Tyres, Tyres probablywill

(A) recover under the contracts clause of theUnited States Constitution.

(B) recover under the provisions of theUniform Commercial Code.

(C) not recover, because the agreement lacksmutuality of obligation.

(D) not recover, because the agreement isindefinite as to quantity.

GO ON TO THE NEXT PAGE.

-9-

Page 12: Sample MBE II

18. Supermarket is in a section of town wherethere are sometimes street fights and wherepedestrians are occasionally the victims ofpickpockets and muggers. In recognition of theunusual number of robberies in the area, thesupermarket posted signs in the store and in itsparking lot that read:

Warning: There are pickpockets andmuggers at work in this part of the city.Supermarket is not responsible for theacts of criminals.

One evening, Lorner drove to Supermarket tosee about a special on turkeys thatSupermarket was advertising. She decided thatthe turkeys were too large and left the storewithout purchasing anything. In the parkinglot, she was attacked by an unknown man whoraped her and then ran away.

If Lorner sues Supermarket, the result shouldbe for the

(A) plaintiff, if Supermarket failed to takereasonable steps to protect customersagainst criminal attack in its parking lot.

(B) plaintiff, because Supermarket is liablefor harm to business invitees on itspremises.

(C) defendant, if the warning signs wereplainly visible to Lorner.

(D) defendant, because the rapist was theproximate cause of Lorner’s injuries.

19. Jones wanted to kill Adams because hebelieved Adams was having an affair withJones’s wife. Early one morning, armed with apistol, he crouched behind some bushes on apark hillside overlooking a path upon whichAdams frequently jogged. On this morning,however, Jones saw Adams jogging on anotherpath about a half mile away. Nonetheless,Jones fired five shots at Adams. None of thefive shots came anywhere close to Adams ashe was well out of the range of the pistolJones was using.

Jones is

(A) guilty of attempted murder, if he was notaware of the limited range of his pistol.

(B) guilty of attempted murder, if areasonable person would not have beenaware of the limited range of his pistol.

(C) not guilty of attempted murder, or anylesser included offense, because, underthe circumstances, it was impossible forhim to have killed Adams.

(D) not guilty of attempted murder, but guiltyof assault.

GO ON TO THE NEXT PAGE.

-10-

Page 13: Sample MBE II

20. Widgets are manufactured wholly from rawmaterials mined and processed in the state ofGreen. The only two manufacturers of widgetsin the United States are also located in thatstate. However, their widgets are purchased byretailers located in every state. The legislatureof the state of Green is considering theadoption of a statute that would impose a taxsolely on the manufacture of widgets. The taxis to be calculated at 3% of their wholesalevalue.

Which of the following arguments would beLEAST helpful to the state in defending theconstitutionality of this proposed state tax onwidgets?

(A) At the time widgets are manufacturedand taxed they have not yet entered thechannels of interstate commerce.

(B) The economic impact of this tax will bepassed on to both in-state and out-of-state purchasers of widgets and,therefore, it is wholly nondiscriminatoryin its effect.

(C) Because of the powers reserved to themby the Tenth Amendment, states haveplenary authority to construct their taxsystem in any manner they choose.

(D) A tax on the manufacture of widgets maybe imposed only by the state in whichthe manufacturing occurs and, therefore,it is not likely to create the danger of amultiple tax burden on interstatecommerce.

21. Blackacre is a large tract of land owned by areligious order known as The Seekers. OnBlackacre, The Seekers erected a largeresidential building where its members reside.Blackacre is surrounded by rural residentialproperties and its only access to a public wayis afforded by an easement over a strip of land30 feet wide. The easement was granted toThe Seekers by deed from Sally, the owner ofone of the adjacent residential properties. TheSeekers built a driveway on the strip, and theeasement was used for 20 years withoutincident or objection.

Last year, as permitted by the applicablezoning ordinance, The Seekers constructed a200-bed nursing home and a parking lot onBlackacre, using all of Blackacre that wasavailable for such development. The nursinghome was very successful, and on Sundaysvisitors to the nursing home overflowed theparking facilities on Blackacre and parked allalong the driveway from early in the morningthrough the evening hours. After two Sundaysof the resulting congestion and inconvenience,Sally erected a barrier across the driveway onSundays preventing any use of the drivewayby anyone seeking access to Blackacre. TheSeekers objected.

Sally brought an appropriate action toterminate the easement.

The most likely result in this action is that thecourt will hold for

(A) Sally, because The Seekers excessivelyexpanded the use of the dominanttenement.

(B) Sally, because the parking on thedriveway exceeded the scope of theeasement.

(C) The Seekers, because expanded use ofthe easement does not terminate theeasement.

(D) The Seekers, because Sally’s use of self-help denies her the right to equitablerelief.

GO ON TO THE NEXT PAGE.

-11-

Page 14: Sample MBE II

22. Ralph and Sam were engaged in a heateddiscussion over the relative merits of theirfavorite professional football teams whenRalph said, “You have to be one of thedumbest persons around.” Sam slapped Ralph.Ralph drew a knife and stabbed Sam in thestomach. Other persons then stepped in andstopped any further fighting. Despite the pleasof the other persons, Sam refused to go to ahospital or to seek medical treatment. Abouttwo hours later, he died as the result of a lossof blood. Ralph was charged with the murderof Sam. At trial, medical evidence establishedthat if Sam had been taken to a hospital, hewould have survived.

At the end of the case, Ralph moves for ajudgment of acquittal or, in the alternative, foran instruction on the elements of voluntarymanslaughter.

The court should

(A) grant the motion for acquittal.(B) deny the motion for acquittal, but

instruct on manslaughter because there isevidence of adequate provocation.

(C) deny both motions, because Ralph failedto retreat.

(D) deny both motions, because malice maybe proved by the intentional use of adeadly weapon on a vital part of thebody.

23. Three months ago, Bert agreed in writing tobuy Sam’s single-family residence, Liveacre,for $110,000. Bert paid Sam a $5,000 depositto be applied to the purchase price. Thecontract stated that Sam had the right at hisoption to retain the deposit as liquidateddamages in the event of Bert’s default. Theclosing was to have taken place last week. Sixweeks ago, Bert was notified by his employerthat he was to be transferred to another job1,000 miles away. Bert immediately notifiedSam that he could not close, and therefore hedemanded the return of his $5,000. Samrefused, waited until after the contract closingdate, listed with a broker, and then conveyedLiveacre for $108,000 to Conner, a purchaserfound by the real estate broker. Conner paidthe full purchase price and immediatelyrecorded his deed. Conner knew of the priorcontract with Bert. In an appropriate action,Bert seeks to recover the $5,000 deposit fromSam.

The most probable result will be that Sam

(A) must return the $5,000 to Bert, becauseSam can no longer carry out his contractwith Bert.

(B) must return the $5,000 to Bert, becauseBert was legally justified in notcompleting the contract.

(C) must return $3,000 to Bert, becauseSam’s damages were only $2,000.

(D) may keep the $5,000 deposit, becauseBert breached the contract.

GO ON TO THE NEXT PAGE.

-12-

Page 15: Sample MBE II

24. Rollem, an automobile retailer, had an adultdaughter, Betsy, who needed a car in heremployment but had only $3,000 with whichto buy one. Rollem wrote to her, “Give meyour $3,000 and I’ll give you the car on ourlot that we have been using as ademonstrator.” Betsy thanked her father andpaid him the $3,000. As both Rollem andBetsy knew, the demonstrator was reasonablyworth $10,000. After Betsy had paid the$3,000, but before the car had been deliveredto her, one of Rollem’s sales staff sold anddelivered the same car to a customer for$10,000. Neither the salesperson nor thecustomer was aware of the transactionbetween Rollem and Betsy.

Does Betsy, after rejecting a tendered return ofthe $3,000 by Rollem, have an action againsthim for breach of contract?

(A) Yes, because Rollem’s promise wassupported by bargained-forconsideration.

(B) Yes, because Rollem’s promise wassupported by the moral obligation afather owes his child as to the necessitiesof modern life.

(C) No, because the payment of $3,000 wasinadequate consideration to supportRollem’s promise.

(D) No, because the salesperson’s delivery ofthe car to the customer made itimpossible for Rollem to perform.

25. Peter, who was 20 years old, purchased a new,high-powered sports car that was marketedwith an intended and recognized appeal toyouthful drivers. The car was designed withthe capability to attain speeds in excess of 100miles per hour. It was equipped with tiresdesigned and tested only for a maximum safespeed of 85 miles per hour. The owner’smanual that came with the car stated that“continuous driving over 90 miles per hourrequires high-speed-capability tires,” but themanual did not describe the speed capabilityof the tires sold with the car.

Peter took his new car out for a spin on astraight, smooth country road where the postedspeed limit was 55 miles per hour. Intendingto test the car’s power, he drove for aconsiderable distance at over 100 miles perhour. While he was doing so, the treadseparated from the left rear tire, causing thecar to leave the road and hit a tree. Petersustained severe injuries.

Peter has brought a strict product liabilityaction in tort against the manufacturer of thecar. You should assume that pure comparativefault principles apply to this case.

Will Peter prevail?

(A) No, because Peter’s driving at anexcessive speed constituted a misuse ofthe car.

(B) No, because the car was not defective.(C) Yes, if the statement in the manual

concerning the tires did not adequatelywarn of the danger of high-speed drivingon the tires mounted on the car.

(D) Yes, unless Peter’s driving at a speed inexcess of the posted speed limit wasnegligence per se that, by the law of thejurisdiction, was not excusable.

GO ON TO THE NEXT PAGE.

-13-

Page 16: Sample MBE II

26. In a federal court diversity action by Plantagainst Decord on an insurance claim, aquestion arose whether the court should applya presumption that, where both husband andwife were killed in a common accident, thehusband died last.

Whether this presumption should be applied isto be determined according to

(A) traditional common law.(B) federal statutory law.(C) the law of the state whose substantive

law is applied.(D) the federal common law.

27. Plagued by neighborhood youths who hadbeen stealing lawn furniture from his backyard, Armando remained awake nightlywatching for them. One evening Armandoheard noises in his backyard. He yelled out,warning intruders to leave. Receiving noanswer, he fired a shotgun filled withnonlethal buckshot into bushes along his backfence where he believed the intruders might behiding. A six-year-old child was hiding in thebushes and was struck in the eye by some ofthe pellets, causing loss of sight.

If Armando is charged with second-degreeassault, which is defined in the jurisdiction as“maliciously causing serious physical injury toanother,” he is

(A) not guilty, because the child wastrespassing and he was using what hebelieved was nondeadly force.

(B) not guilty, because he did not intend tokill or to cause serious physical injury.

(C) guilty, because he recklessly causedserious physical injury.

(D) guilty, because there is no privilege touse force against a person who is tooyoung to be criminally responsible.

28. Twenty percent of the residents of Green Cityare members of minority racial groups. Theseresidents are evenly distributed among themany different residential areas of the city.The five city council members of Green Cityare elected from five single-member electoraldistricts that are nearly equally populated. Nocandidate has ever been elected to the citycouncil who was a member of a minorityracial group.

A group of citizens who are members ofminority racial groups file suit in federaldistrict court seeking a declaratory judgmentthat the single-member districts in Green Cityare unconstitutional. They claim that thesingle-member districting system in that citydiminishes the ability of voters who aremembers of minority racial groups to affectthe outcome of city elections. They seek anorder from the court forcing the city to adoptan at-large election system in which the fivecandidates with the greatest vote totals wouldbe elected to the city council. No state orfederal statutes are applicable to the resolutionof this suit.

Which of the following constitutionalprovisions provides the most obvious basis forplaintiffs’ claim in this suit?

(A) The Thirteenth Amendment.(B) The due process clause of the Fourteenth

Amendment.(C) The privileges and immunities clause of

the Fourteenth Amendment.(D) The Fifteenth Amendment.

GO ON TO THE NEXT PAGE.

-14-

Page 17: Sample MBE II

29. Loomis, the owner and operator of a smallbusiness, encourages “wellness” on the part ofhis employees and supports various physical-fitness programs to that end. Learning that oneof his employees, Graceful, was a dedicatedjogger, Loomis promised to pay her a specialaward of $100 if she could and would run onemile in less than six minutes on the followingSaturday. Graceful thanked him, and did infact run a mile in less than six minutes on theday specified. Shortly thereafter, however,Loomis discovered that for more than a yearGraceful had been running at least one mile inless than six minutes every day as a part of herpersonal fitness program. He refused to paythe $100.

In an action by Graceful against Loomis forbreach of contract, which of the following bestsummarizes the probable decision of thecourt?

(A) Loomis wins, because it is a compellinginference that Loomis’s promise did notinduce Graceful to run the specifiedmile.

(B) Loomis wins, because Graceful’s runningof the specified mile was beneficial, notdetrimental, to her in any event.

(C) Graceful wins, because running a mile inless than six minutes is a significantlydemanding enterprise.

(D) Graceful wins, because she ran thespecified mile as requested, and hermotives for doing so are irrelevant.

30. Able was the owner of Blackacre, anundeveloped city lot. Able and Baker executeda written document in which Able agreed tosell Blackacre to Baker and Baker agreed tobuy Blackacre from Able for $100,000; thedocument did not provide for an earnestmoney down payment. Able recorded thedocument, as authorized by statute.

Able orally gave Baker permission to park hiscar on Blackacre without charge prior to theclosing. Thereafter, Baker frequently parkedhis car on Blackacre.

Another property came on the market thatBaker wanted more than Blackacre. Bakerdecided to try to escape any obligation toAble.

Baker had been told that contracts for thepurchase and sale of real property requireconsideration and concluded that because hehad made no earnest money down payment, hecould refuse to close and not be liable. Bakernotified Able of his intention not to close and,in fact, did refuse to close on the date set forthe closing. Able brought an appropriate actionto compel specific performance by Baker.

If Able wins, it will be because

(A) Baker’s use of Blackacre for parkingconstitutes part performance.

(B) general contract rules regardingconsideration apply to real estatecontracts.

(C) the doctrine of equitable conversionapplies.

(D) the document was recorded.

GO ON TO THE NEXT PAGE.

-15-

Page 18: Sample MBE II

Questions 31-32 are based on the following fact situation.

Under the terms of a written contract, Karp agreedto construct for Manor a garage for $10,000.Nothing was said in the parties’ negotiations or inthe contract about progress payments during thecourse of the work.

31. For this question only, assume the followingfacts. After completing 25% of the garagestrictly according to Manor’s specifications,Karp demanded payment of $2,000 as a“reasonable progress payment.” Manorrefused, and Karp abandoned the job.

If each party sues the other for breach ofcontract, which of the following will the courtdecide?

(A) Both parties are in breach, and each isentitled to damages, if any, from theother.

(B) Only Karp is in breach and liable forManor’s damages, if any.

(C) Only Manor is in breach and liable forKarp’s damages, if any.

(D) Both parties took reasonable positions,and neither is in breach.

32. For this question only, assume the followingfacts. After completing 25% of the garagestrictly according to Manor’s specifications,Karp assigned his rights under the contract toBanquo as security for an $8,000 loan. Banquoimmediately notified Manor of the assignment.Karp thereafter, without legal excuse,abandoned the job before it was half-complete.Karp subsequently defaulted on the loan fromBanquo. Karp has no assets. It will cost Manorat least $8,000 to get the garage finished byanother builder.

If Banquo sues Manor for $8,000, which ofthe following will the court decide?

(A) Banquo wins, because the Karp-Manorcontract was in existence and Karp wasnot in breach when Banquo gave Manornotice of the assignment.

(B) Banquo wins, because Banquo as asecured creditor over Karp is entitled topriority over Manor’s unsecured claimagainst Karp.

(C) Manor wins, because his right torecoupment on account of Karp’s breachis available against Banquo as Karp’sassignee.

(D) Manor wins, because his claim againstKarp arose prior to Karp’s default on hisloan from Banquo.

GO ON TO THE NEXT PAGE.

-16-

Page 19: Sample MBE II

33. The Sports Championship RevenueEnhancement Act is a federal statute that wasenacted as part of a comprehensive program toeliminate the federal budget deficit. That actimposed, for a period of five years, a 50%excise tax on the price of tickets tochampionship sporting events. Such eventsincluded the World Series, the Super Bowl,major college bowl games, and similarchampionship sports events.

This federal tax is probably

(A) constitutional, because the compellingnational interest in reducing the federalbudget deficit justifies this tax as atemporary emergency measure.

(B) constitutional, because an act ofCongress that appears to be a revenueraising measure on its face is notrendered invalid because it may haveadverse economic consequences for theactivity taxed.

(C) unconstitutional, because a 50% tax islikely to reduce attendance atchampionship sporting events and,therefore, is not rationally related to thelegitimate interest of Congress ineliminating the budget deficit.

(D) unconstitutional, because Congressviolates the equal protection componentof the Fifth Amendment by singling outchampionship sporting events for this taxwhile failing to tax other major sporting,artistic, or entertainment events to whichtickets are sold.

34. On June 1, Topline Wholesale, Inc., received apurchase-order form from Wonder-Good, Inc.,a retailer and new customer, in which the latterordered 1,000 anti-recoil widgets for deliveryno later than August 30 at a delivered totalprice of $10,000, as quoted in Topline’scurrent catalog. Both parties are merchantswith respect to widgets of all types. On June 2,Topline mailed to Wonder-Good its own form,across the top of which Topline’s president hadwritten, “We are pleased to accept your order.”This form contained the same terms asWonder-Good’s form except for an additionalprinted clause in Topline’s form that providedfor a maximum liability of $100 for anybreach of contract by Topline.

As of June 5, when Wonder-Good receivedTopline’s acceptance form, which of thefollowing is an accurate statement concerningthe legal relationship between Topline andWonder-Good?

(A) There is no contract, because theliability-limitation clause in Topline’sform is a material alteration of Wonder-Good’s offer.

(B) There is no contract, because Wonder-Good did not consent to the liability-limitation clause in Topline’s form.

(C) There is an enforceable contract whoseterms include the liability-limitationclause in Topline’s form, becauseliquidation of damages is expresslyauthorized by the Uniform CommercialCode.

(D) There is an enforceable contract whoseterms do not include the liability-limitation clause in Topline’s form.

GO ON TO THE NEXT PAGE.

-17-

Page 20: Sample MBE II

35. Electco operates a factory that requires the useof very high voltage electricity. Paul ownsproperty adjacent to the Electco plant wherehe has attempted to carry on a business thatrequires the use of sensitive electronicequipment. The effectiveness of Paul’selectronic equipment is impaired by electricalinterference arising from the high voltagecurrents used in Electco’s plant. Paul hascomplained to Electco several times, with noresult. There is no way that Electco, by takingreasonable precautions, can avoid theinterference with Paul’s operation that arisesfrom the high voltage currents necessary toElectco’s operation.

In Paul’s action against Electco to recoverdamages for the economic loss caused to himby the electrical interference, will Paulprevail?

(A) Yes, because Electco’s activity isabnormally dangerous.

(B) Yes, for loss suffered by Paul afterElectco was made aware of the harm itsactivity was causing to Paul.

(C) No, unless Electco caused a substantialand unreasonable interference withPaul’s business.

(D) No, because Paul’s harm was purelyeconomic and did not arise from physicalharm to his person or property.

36. Les leased a barn to his neighbor, Tom, for aterm of three years. Tom took possession ofthe barn and used it for his farming purposes.The lease made Les responsible for structuralrepairs to the barn, unless they were madenecessary by actions of Tom.

One year later, Les conveyed the barn and itsassociated land to Lottie “subject to the leaseto Tom.” Tom paid the next month’s rent toLottie. The next day a portion of an exteriorwall of the barn collapsed because of rot in theinterior structure of the wall. The wall hadappeared to be sound, but a competentengineer, on inspection, would havediscovered its condition. Neither Lottie norTom had the barn inspected by an engineer.Tom was injured as a result of the collapse ofthe wall.

Les had known that the wall was dangerouslyweakened by rot and needed immediaterepairs, but had not told Tom or Lottie. Thereis no applicable statute.

Tom brought an appropriate action against Lesto recover damages for the injuries hesustained. Lottie was not a party.

Which of the following is the most appropriatecomment concerning the outcome of thisaction?

(A) Tom should lose, because Lottie assumedall of Les’s obligations by reason ofTom’s attornment to her.

(B) Tom should recover, because there isprivity between lessor and lessee and itcannot be broken unilaterally.

(C) Tom should recover, because Les knewof the danger but did not warn Tom.

(D) Tom should lose, because he failed toinspect the barn.

GO ON TO THE NEXT PAGE.

-18-

Page 21: Sample MBE II

-19-

37. Dahle is charged with possession of heroin.Prosecution witness Walker, an experienceddog trainer, testified that he was in the airportwith a dog trained to detect heroin. As Dahleapproached, the dog immediately became alertand pawed and barked frantically at Dahle’sbriefcase. Dahle managed to run outside andthrow his briefcase into the river, from whichit could not be recovered. After Walker’sexperience is established, he is asked to testifyas an expert that the dog’s reaction told himthat Dahle’s briefcase contained heroin.

Walker’s testimony is

(A) admissible, as evidence of Dahle’s guilt.(B) admissible, because an expert may rely

on hearsay.(C) inadmissible, because it is based on

hearsay not within any exception.(D) inadmissible, because of the unreliability

of the reactions of an animal.

38. Doe negligently caused a fire in his house, andthe house burned to the ground. As a result,the sun streamed into Peter’s yard next door,which previously had been shaded by Doe’shouse. The sunshine destroyed some delicateand valuable trees in Peter’s yard that couldgrow only in the shade. Peter has brought anegligence action against Doe for the loss ofPeter’s trees. Doe has moved to dismiss thecomplaint.

The best argument in support of this motionwould be that

(A) Doe’s negligence was not the activecause of the loss of Peter’s trees.

(B) Doe’s duty to avoid the risks created by afire did not encompass the risk thatsunshine would damage Peter’s trees.

(C) the loss of the trees was not a natural andprobable consequence of Doe’snegligence.

(D) Peter suffered a purely economic loss,which is not compensable in anegligence action.

39. Phillips bought a new rifle and wanted to try itout by doing some target shooting. He wentout into the country to an area where he hadpreviously hunted. Much to his surprise, henoticed that the area beyond a clearingcontained several newly constructed housesthat had not been there before. Between thehouses there was a small playground whereseveral children were playing. Nevertheless,Phillips nailed a paper target to a tree and wentto a point where the tree was between himselfand the playground. He then fired severalshots at the target. One of the shots missed thetarget and the tree and hit and killed one of thechildren in the playground.

Phillips was convicted of murder. Heappealed, contending that the evidence was notsufficient to support a conviction of murder.

The appellate court should

(A) affirm the conviction, as the evidence issufficient to support a conviction ofmurder.

(B) reverse the conviction and remand for anew trial, because the evidence is notsufficient for murder but will support aconviction of voluntary manslaughter.

(C) reverse the conviction and remand for anew trial, because the evidence is notsufficient for murder but will support aconviction of involuntary manslaughter.

(D) reverse the conviction and order the casedismissed, because the evidence issufficient only for a finding ofnegligence and negligence alone cannotsupport a criminal conviction.

GO ON TO THE NEXT PAGE.

Page 22: Sample MBE II

Questions 40-41 are based on the following fact situation.

Dominique obtained a bid of $10,000 to tear downher old building and another bid of $90,000 toreplace it with a new structure in which she plannedto operate a sporting goods store. Having onlylimited cash available, Dominique asked Hardcashfor a $100,000 loan. After reviewing the plans forthe project, Hardcash in a signed writing promisedto lend Dominique $100,000 secured by a mortgageon the property and repayable over ten years inequal monthly installments at 10% annual interest.Dominique promptly accepted the demolition bidand the old building was removed, but Hardcashthereafter refused to make the loan. Despite diligentefforts, Dominique was unable to obtain a loan fromany other source.

40. Does Dominique have a cause of actionagainst Hardcash?

(A) Yes, because by having the buildingdemolished, she accepted Hardcash’soffer to make the loan.

(B) Yes, because her reliance on Hardcash’spromise was substantial, reasonable, andforeseeable.

(C) No, because there was no bargained-forexchange of consideration for Hardcash’spromise to make the loan.

(D) No, because Dominique’s inability toobtain a loan from any other sourcedemonstrated that the project lacked thefinancial soundness that was aconstructive condition to Hardcash’sperformance.

41. For this question only, assume that Dominiquehas a cause of action against Hardcash.

If she sues him for monetary relief, what is theprobable measure of her recovery?

(A) Expectancy damages, measured by thedifference between the value of the newbuilding and the old building, less theamount of the proposed loan ($100,000).

(B) Expectancy damages, measured by theestimated profits from operating theproposed sporting goods store for tenyears, less the cost of repaying a$100,000 loan at 10% interest over tenyears.

(C) Reliance damages, measured by the$10,000 expense of removing the oldbuilding, adjusted by the decrease orincrease in the market value ofDominique’s land immediately thereafter.

(D) Nominal damages only, because bothexpectancy and reliance damages arespeculative, and there is no legal orequitable basis for awarding restitution.

GO ON TO THE NEXT PAGE.

-20-

Page 23: Sample MBE II

42. Dan, an eight-year-old, rode his bicycle downhis driveway into a busy highway and Driverhad to stop her car suddenly to avoid collidingwith the bike. Because of the sudden stop,Driver’s two-year-old son, Peter, who wassitting on the seat without any restraint, wasthrown into the dashboard and injured. HadPeter been properly restrained in a baby carseat, as required by a state safety statute ofwhich his mother was aware, he would nothave been injured.

In an action brought on Peter’s behalf againstDan’s parents to recover for Peter’s injuries,Peter will

(A) not prevail, because parents are notvicariously liable for the negligent actsof their children.

(B) not prevail, because Peter’s injury wasattributable to his mother’s knowingviolation of a safety statute.

(C) prevail, if Dan’s parents knew that hesometimes drove into the highway, andthey took no steps to prevent it.

(D) prevail, if Dan’s riding into the highwaywas negligent and the proximate cause ofPeter’s injuries.

43. While Hill was in her kitchen, she heard thescreech of automobile tires. She ran to thewindow and saw a tricycle flying through theair. The tricycle had been hit by a car drivenby Weber, who had been speeding. She alsosaw a child’s body in the grass adjacent to thestreet. As a result of her shock from thisexperience, Hill suffered a heart attack.

In a claim by Hill against Weber, the issue onwhich Hill’s right to recover will depend iswhether

(A) a person can recover damages based onthe defendant’s breach of a duty owed toanother.

(B) it is foreseeable that a person may sufferphysical harm caused solely by an injuryinflicted on another.

(C) a person can recover damages caused byshock unaccompanied by bodily impact.

(D) a person can recover damages for harmresulting from shock caused solely byanother’s peril or injury.

44. Suffering from painful and terminal cancer,Willa persuaded Harold, her husband, to killher to end her misery. As they reminiscedabout their life together and reaffirmed theirlove for each other, Harold tried to discourageWilla from giving up. Willa insisted, however,and finally Harold held a gun to her head andkilled her.

The most serious degree of criminal homicideof which Harold can be legally convicted is

(A) no degree of criminal homicide.(B) involuntary manslaughter.(C) voluntary manslaughter.(D) murder.

GO ON TO THE NEXT PAGE.

-21-

Page 24: Sample MBE II

45. Peterson sued Dylan for libel. After Petersontestified that Dylan wrote to Peterson’semployer that Peterson was a thief, Dylanoffers evidence that Peterson once stole moneyfrom a former employer.

The evidence of Peterson’s prior theft is

(A) admissible, as substantive evidence toprove that Peterson is a thief.

(B) admissible, but only to impeachPeterson’s credibility.

(C) inadmissible, because character may notbe shown by specific instances ofconduct.

(D) inadmissible, because such evidence ismore unfairly prejudicial than probative.

46. The Federal Computer Abuse Act establishesthe Federal Computer Abuse Commission,authorizes the Commission to issue licensesfor the possession of computers on terms thatare consistent with the purposes of the act, andmakes the unlicensed possession of acomputer a crime. The provisions of theFederal Computer Abuse Act are inseverable.

User applied to the Federal Computer AbuseCommission for a license to possess acomputer. The Commission held, and Userparticipated in, a trial-type proceeding onUser’s license application. In that proceedingit was demonstrated that User repeatedly andintentionally used computers to introducesecret destructive computer programs(computer viruses) into electronic data bankswithout the consent of their owners. As aresult, the Commission denied User’sapplication for a license. The license denialwas based on a Commission rule authorizedby the Computer Abuse Act that prohibited theissuance of computer licenses to persons whohad engaged in such conduct. Nevertheless,User retained and continued to use hiscomputer. He was subsequently convicted ofthe crime of unlicensed possession of acomputer. On appeal, he challenges theconstitutionality of the licensing provision ofthe Federal Computer Abuse Act.

In this case, the reviewing court wouldprobably hold that act to be

(A) constitutional, because the Constitutiongenerally authorizes Congress to enactall laws that are necessary and proper toadvance the general welfare, andCongress could reasonably believe thatpossession of computers by people likeUser constitutes a threat to the generalwelfare.

(B) constitutional, because Congress may usethe authority vested in it by thecommerce clause to regulate thepossession of computers and theprovisions of this act do not violate anyprohibitory provision of the Constitution.

(C) unconstitutional, because Congress maynot impose a criminal penalty on actionthat is improper only because it isinconsistent with an agency rule.

(D) unconstitutional, because the merepossession of a computer is a whollylocal matter that is beyond the regulatoryauthority of Congress.

GO ON TO THE NEXT PAGE.

-22-

Page 25: Sample MBE II

47. Defendant left her car parked on the side of ahill. Two minutes later, the car rolled down thehill and struck and injured Plaintiff.

In Plaintiff’s negligence action againstDefendant, Plaintiff introduced into evidencethe facts stated above, which are undisputed.Defendant testified that, when she parked hercar, she turned the front wheels into the curband put on her emergency brakes, which werein good working order. She also introducedevidence that, in the weeks before thisincident, juveniles had been seen tamperingwith cars in the neighborhood. The juryreturned a verdict in favor of Defendant, andPlaintiff moved for a judgment notwithstandingthe verdict.

Plaintiff’s motion should be

(A) granted, because it is more likely thannot that Defendant’s negligent conductwas the legal cause of Plaintiff’s injuries.

(B) granted, because the evidence does notsupport the verdict.

(C) denied, because, given Defendant’sevidence, the jury was not required todraw an inference of negligence from thecircumstances of the accident.

(D) denied, if Defendant was in no betterposition than Plaintiff to explain theaccident.

48. Able conveyed Blackacre to Baker by awarranty deed. Baker recorded the deed fourdays later. After the conveyance but prior toBaker’s recording of the deed, Smollettproperly filed a judgment against Able.

The two pertinent statutes in the jurisdictionprovide the following: 1) any judgmentproperly filed shall, for ten years from filing,be a lien on the real property then owned orsubsequently acquired by any person againstwhom the judgment is rendered, and 2) noconveyance or mortgage of real property shallbe good against subsequent purchasers forvalue and without notice unless the same berecorded according to law.

The recording act has no provision for a graceperiod.

Smollett joined both Able and Baker in anappropriate action to foreclose the judgmentlien against Blackacre.

If Smollett is unsuccessful, it will be because

(A) Able’s warranty of title to Baker defeatsSmollett’s claim.

(B) Smollett is not a purchaser for value.(C) any deed is superior to a judgment lien.(D) four days is not an unreasonable delay in

recording a deed.

GO ON TO THE NEXT PAGE.

-23-

Page 26: Sample MBE II

49. The United States Department of Energyregularly transports nuclear materials throughCenterville on the way to a nuclear weaponsprocessing plant it operates in a nearby state.The city of Centerville recently adopted anordinance prohibiting the transportation of anynuclear materials in or through the city. Theordinance declares that its purpose is to protectthe health and safety of the residents of thatcity.

May the Department of Energy continue totransport these nuclear materials through thecity of Centerville?

(A) No, because the ordinance is rationallyrelated to the public health and safety ofCenterville residents.

(B) No, because the Tenth Amendmentreserves to the states certainunenumerated sovereign powers.

(C) Yes, because the Department of Energyis a federal agency engaged in a lawfulfederal function and, therefore, itsactivities may not be regulated by a localgovernment without the consent ofCongress.

(D) Yes, because the ordinance enacted byCenterville is invalid because it deniespersons transporting such materials theequal protection of the laws.

50. Dart is charged with the statutory offense of“knowingly violating a regulation of the StateAlcoholic Beverage Control Board” andspecifically that he knowingly violatedregulation number 345-90 issued by the StateAlcoholic Beverage Control Board. Thatregulation prohibits the sale of alcoholicbeverages to any person under the age of 18and also prohibits the sale of any alcoholicbeverage to a person over the age of 17 andunder the age of 22 without the presentation ofsuch person’s driver’s license or otheridentification showing the age of the purchaserto be 18 or older.

The evidence showed that Dart was a bartenderin a tavern and sold a bottle of beer to aperson who was 17 years old and that Dart did not ask for or see the purchaser’s driver’slicense or any other identification.

Which of the following, if found by the jury,would be of the most help to Dart?

(A) The purchaser had a driver’s license thatfalsely showed his age to be 21.

(B) Dart had never been told he wassupposed to check identification ofpersons over 17 and under 22 beforeselling them alcohol.

(C) Dart did not know that the regulationsclassified beer as an alcoholic beverage.

(D) Dart mistakenly believed the purchaserto be 24 years old.

GO ON TO THE NEXT PAGE.

-24-

Page 27: Sample MBE II

-25-

Questions 51-52 are based on the following fact situation.

In a writing signed by both parties on December 1,Kranc agreed to buy from Schaff a gasoline enginefor $1,000, delivery to be made on the followingFebruary 1. Through a secretarial error, the writingcalled for delivery on March 1, but neither partynoticed the error until February 1. Before signingthe agreement, Kranc and Schaff orally agreed thatthe contract of sale would be effective only if Krancshould notify Schaff in writing not later thanJanuary 2 that Kranc had arranged to resell theengine to a third person. Otherwise, they agreedorally, “There is no deal.” On December 15, Krancentered into a contract with Trimota to resell theengine to Trimota at a profit.

51. For this question only, assume the followingfacts. Kranc did not give Schaff notice of theresale until January 25, and Schaff received itby mail on January 26. Meantime, the value ofthe engine had unexpectedly increased about75% since December 1, and Schaff renouncedthe agreement.

If Kranc sues Schaff on February 2 for breachof contract, which of the following is Schaff’sbest defense?

(A) The secretarial error in the writtendelivery-term was a mutual mistakeconcerning a basic fact, and theagreement is voidable by either party.

(B) Kranc’s not giving written notice byJanuary 2 of his resale was a failure of acondition precedent to the existence of acontract.

(C) In view of the unexpected 75% increasein value of the engine after December 1,Schaff’s performance is excused by thedoctrine of commercial frustration.

(D) The agreement, if any, is unenforceablebecause a material term was not includedin the writing.

52. For this question only, assume the followingfacts. On December 16, Kranc notified Schaffby telephone of Kranc’s resale agreement withTrimota, and explained that a written noticewas unfeasible because Kranc’s secretary wasill. Schaff replied, “That’s okay. I’ll get theengine to you on February 1, as we agreed.”Having learned, however, that the engine hadincreased in value about 75% since December1, Schaff renounced the agreement onFebruary 1.

If Kranc sues Schaff on February 2 for breachof contract, which of the following conceptsbest supports Kranc’s claim?

(A) Substantial performance.(B) Nonoccurrence of a condition

subsequent.(C) Waiver of condition.(D) Novation of buyers.

GO ON TO THE NEXT PAGE.

Page 28: Sample MBE II

53. David owned a shotgun that he used forhunting. David knew that his old friend, Mark,had become involved with a violent gang thatrecently had a shoot-out with a rival gang.David, who was going to a farm to hunt quail,placed his loaded shotgun on the back seat ofhis car. On his way to the farm, David pickedup Mark to give him a ride to a friend’s house.After dropping off Mark at the friend’s house,David proceeded to the farm, where hediscovered that his shotgun was missing fromhis car. Mark had taken the shotgun and, laterin the day, Mark used it to shoot Paul, amember of the rival gang. Paul was severelyinjured.

Paul recovered a judgment for his damagesagainst David, as well as Mark, on the groundthat David was negligent in allowing Mark toobtain possession of the gun, and wastherefore liable jointly and severally withMark for Paul’s damages. The jurisdiction hasa statute that allows contribution based uponproportionate fault and adheres to thetraditional common-law rules on indemnity.

If David fully satisfies the judgment, Davidthen will have a right to recover from Mark

(A) indemnity for the full amount of thejudgment, because Mark was anintentional tortfeasor.

(B) contribution only, based on comparativefault, because David himself wasnegligent.

(C) one-half of the amount of the judgment.(D) nothing, because David’s negligence was

a substantial proximate cause of theshooting.

54. The legislature of the state of Chetopahenacted a statute requiring that all lawenforcement officers in that state be citizens ofthe United States. Alien, lawfully admitted topermanent residency five years before theenactment of this statute, sought employmentas a forensic pathologist in the Chetopahcoroner’s office. He was denied such a jobsolely because he was not a citizen.

Alien thereupon brought suit in federal districtcourt against appropriate Chetopah officialsseeking to invalidate this citizenshiprequirement on federal constitutional grounds.

The strongest ground upon which to attackthis citizenship requirement is that it

(A) constitutes an ex post facto law as topreviously admitted aliens.

(B) deprives an alien of a fundamental rightto employment without the due processof law guaranteed by the FourteenthAmendment.

(C) denies an alien a right to employment inviolation of the privileges andimmunities clause of the FourteenthAmendment.

(D) denies an alien the equal protection ofthe laws guaranteed by the FourteenthAmendment.

GO ON TO THE NEXT PAGE.

-26-

Page 29: Sample MBE II

-27-

55. Olwen owned 80 acres of land, fronting on atown road. Two years ago, Olwen sold to Buckthe back 40 acres. The 40 acres sold to Buckdid not adjoin any public road. Olwen’s deedto Buck expressly granted a right-of-way overa specified strip of Olwen’s retained 40 acres,so Buck could reach the town road. The deedwas promptly and properly recorded.

Last year, Buck conveyed the back 40 acres toSam. They had discussed the right-of-wayover Olwen’s land to the road, but Buck’s deedto Sam made no mention of it. Sam began touse the right-of-way as Buck had, but Olwensued to enjoin such use by Sam.

The court should decide for

(A) Sam, because he has an easement byimplication.

(B) Sam, because the easement appurtenantpassed to him as a result of Buck’s deedto him.

(C) Olwen, because Buck’s easement ingross was not transferable.

(D) Olwen, because Buck’s deed failedexpressly to transfer the right-of-way toSam.

56. Dickinson was charged with possession ofcocaine. At Dickinson’s trial, the prosecutionestablished that, when approached by policeon a suburban residential street corner,Dickinson dropped a plastic bag and ran, andthat when the police returned to the corner afew minutes later after catching Dickinson,they found a plastic bag containing whitepowder. Dickinson objects to introduction ofthis bag (the contents of which would later beestablished to be cocaine), citing lack ofadequate identification.

The objection should be

(A) overruled, because there is sufficientevidence to find that the bag was the oneDickinson dropped.

(B) overruled, because the objection shouldhave been made on the basis ofincomplete chain of custody.

(C) sustained, because Dickinson did nothave possession of the bag at the time hewas arrested.

(D) sustained, unless the judge makes afinding by a preponderance of theevidence that the bag was the onedropped by Dickinson.

GO ON TO THE NEXT PAGE.

Page 30: Sample MBE II

57. Chemco manufactured a liquid chemicalproduct known as XRX. Some XRX leakedfrom a storage tank on Chemco’s property,seeped into the groundwater, flowed toFarmer’s adjacent property, and pollutedFarmer’s well. Several of Farmer’s cows drankthe polluted well water and died.

If Farmer brings an action against Chemco torecover the value of the cows that died,Farmer will

(A) prevail, because a manufacturer isstrictly liable for harm caused by itsproducts.

(B) prevail, because the XRX escaped fromChemco’s premises.

(C) not prevail, unless Farmer can establishthat the storage tank was defective.

(D) not prevail, unless Chemco failed toexercise reasonable care in storing theXRX.

58. A threatening telephone call that purports to befrom Defendant to Witness is most likely to beadmitted against Defendant if

(A) the caller identified himself asDefendant.

(B) Witness had previously given damagingtestimony against Defendant in anotherlawsuit.

(C) Witness had given his unlisted numberonly to Defendant and a few otherpersons.

(D) Witness believes that Defendant iscapable of making such threats.

GO ON TO THE NEXT PAGE.

-28-

Page 31: Sample MBE II

59. The open-air amphitheater in the city park ofRightville has been utilized for concerts andother entertainment programs. Until this year,each of the groups performing in that cityfacility was allowed to make its ownarrangements for sound equipment and soundtechnicians.

After recurring complaints from occupants ofresidential buildings adjacent to the city parkabout intrusive noise from some performancesheld in the amphitheater, the Rightville CityCouncil passed an ordinance establishing citycontrol over all sound amplification at allprograms held there. The ordinance providedthat Rightville’s Department of Parks wouldbe the sole provider in the amphitheater ofsound amplification equipment and of thetechnicians to operate the equipment “toensure a proper balance between the quality ofthe sound at such performances and respect forthe privacy of nearby residential neighbors.”

Which of the following standards should acourt use to determine the constitutionality onits face of this content neutral ordinance?

(A) The ordinance is narrowly tailored toserve a substantial government interest,and does not unreasonably limitalternative avenues of expression.

(B) The ordinance is rationally related to alegitimate government interest, and doesnot unreasonably limit alternativeavenues of expression.

(C) The ordinance is rationally related to alegitimate government interest andrestricts the expressive rights involvedno more than is reasonable under thecircumstances.

(D) The ordinance is substantially related toa legitimate governmental interest andrestricts the expressive rights involvedno more than is reasonable in light of thesurrounding circumstances.

60. Smith and Penn were charged with murder.Each gave a confession to the police thatimplicated both of them. Smith later retractedher confession, claiming that it was coerced.

Smith and Penn were tried together. Theprosecutor offered both confessions intoevidence. Smith and Penn objected. After ahearing, the trial judge found that bothconfessions were voluntary and admitted bothinto evidence. Smith testified at trial. Shedenied any involvement in the crime andclaimed that her confession was false and theresult of coercion. Both defendants wereconvicted.

On appeal, Smith contends her convictionshould be reversed because of the admissioninto evidence of Penn’s confession.

Smith’s contention is

(A) correct, unless Penn testified at trial.(B) correct, whether or not Penn testified at

trial.(C) incorrect, because Smith testified in her

own behalf.(D) incorrect, because Smith’s own

confession was properly admitted intoevidence.

GO ON TO THE NEXT PAGE.

-29-

Page 32: Sample MBE II

61. The state of Orrington wanted to prevent itsonly major league baseball team, the privatelyowned and operated Orrington Opossums,from moving to the rival state of Atrium. Aftera heated political debate in the legislature,Orrington enacted legislation providing for aone-time grant of $10 million in state funds tothe Opossums to cover part of the projectedincome losses the team would suffer duringthe next five years if it remained in that state.The legislation required that the team remainin the state for at least ten years if it acceptedthe grant.

After accepting the grant, the owners of theOpossums decided to build a new $150million stadium in Orrington. As plans for theconstruction of the new stadium proceeded, itbecame evident that all of the contractors andsubcontractors would be white males, and thatthey had been chosen by the owners of theOpossums without any public bids becausethese contractors and subcontractors hadsuccessfully built the only other new baseballstadium in the region. Several contractors whowere females or members of minority racialgroups filed suit against the owners of theOpossums in federal district court to compelpublic solicitation of bids for the constructionof its new stadium on an equal opportunitybasis, and to enjoin construction of thestadium until compliance was ensured. Theironly claim was that the contracting practicesof the owners of the Opossums denied themthe equal protection of the laws in violation ofthe Fourteenth Amendment.

In this suit, the court will probably rule that

(A) the nexus between the actions of theowners of the Opossums and the one-time grant of monies to them by the stateis sufficiently substantial to subject theiractions to the limitations of theFourteenth Amendment.

(B) the intense public preoccupation with theactivities of major league baseball teamscoupled with the fact that baseball isconsidered to be our national pastime issufficient to justify application of theFourteenth Amendment to the activitiesof major league teams.

(C) in the absence of additional evidence ofstate involvement in the operations ordecisions of the owners of theOpossums, a onetime grant of statemonies to them is insufficient to warranttreating their actions as subject to thelimitations of the FourteenthAmendment.

(D) the issues presented by this case arenonjusticiable political questions becausethere is a lack of judicially manageablestandards to resolve them and they arelikely to be deeply involved in partisanpolitics.

GO ON TO THE NEXT PAGE.

-30-

Page 33: Sample MBE II

Questions 62-63 are based on the following fact situation.

Walker, who knew nothing about horses, inheritedAberlone, a thoroughbred colt whose disagreeablebehavior made him a pest around the barn. Walkersold the colt for $1,500 to Sherwood, anexperienced racehorse-trainer who knew of Walker’signorance about horses. At the time of sale, Walkersaid to Sherwood, “I hate to say it, but this horse isbad-tempered and nothing special.”

62. For this question only, assume that soon afterthe sale, Aberlone won three races and earned$400,000 for Sherwood.

Which of the following additional facts, ifestablished by Walker, would best support hischance of obtaining rescission of the sale toSherwood?

(A) Walker did not know until after the salethat Sherwood was an experiencedracehorse-trainer.

(B) At a pre-sale exercise session of whichSherwood knew that Walker was notaware, Sherwood clocked Aberlone inrecord-setting time, far surpassing anyprevious performance.

(C) Aberlone was the only thoroughbred thatWalker owned, and Walker did not knowhow to evaluate young and untestedracehorses.

(D) At the time of the sale, Walker was angryand upset over an incident in whichAberlone had reared and thrown a rider.

63. Which one of the following scenarios wouldbest support an action by Sherwood, ratherthan Walker, to rescind the sale?

(A) In his first race after the sale, Aberlonegalloped to a huge lead but dropped dead100 yards from the finish line because ofa rare congenital heart defect that wasundiscoverable except by autopsy.

(B) Aberlone won $5 million for Sherwoodover a three-year racing career but uponbeing retired was found to be incurablysterile and useless as a breeder.

(C) After Aberlone had won three races forSherwood, it was discovered that byclerical error, unknown to either party,Aberlone’s official birth registrationlisted an undistinguished racehorse as thesire rather than the famous racehorse thatin fact was the sire.

(D) A week after the sale, Aberlone wentberserk and inflicted injuries uponSherwood that required hishospitalization for six months and a fullyear for his recovery.

GO ON TO THE NEXT PAGE.

-31-

Page 34: Sample MBE II

64. Sixty years ago by a properly executed andrecorded deed, Albert conveyed Greenacre, atract of land: “To Louis for life, then toLouis’s widow for her life, then to Louis’schild or children in equal shares.” At that time,Louis, who was Albert’s grandson, was sixyears old.

Shortly thereafter, Albert died testate. Louiswas his only heir at law. Albert’s will left hisentire estate to First Church.

Twenty-five years ago, when he was 41, Louismarried Maria who was then 20 years old;they had one child, Norman. Maria andNorman were killed in an automobile accidentthree years ago when Norman was 21. Normandied testate, leaving his entire estate to theAmerican Red Cross. His father, Louis, wasNorman’s sole heir at law.

Two years ago, Louis married Zelda. They hadno children. This year, Louis died testate,survived by his widow, Zelda, to whom he lefthis entire estate.

The common-law Rule Against Perpetuities isunchanged by statute in the jurisdiction.

In an appropriate action to determine theownership of Greenacre, the court should findthat title is vested in

(A) First Church, because the widow ofLouis was unborn at the time ofconveyance and, hence, the remainderviolated the Rule Against Perpetuities.

(B) Zelda, because her life estate and herinheritance from Louis (who wasAlbert’s sole heir at law and who wasNorman’s sole heir at law) merged theentire title in her.

(C) the American Red Cross, becauseNorman had a vested remainder interest(as the only child of Louis) that itinherited, the life estate to Louis’s widowbeing of no force and effect.

(D) Zelda for life under the terms of Albert’sdeed, with the remainder to the AmericanRed Cross as the successor in interest toNorman, Louis’s only child.

65. In an automobile collision case brought by Poeagainst Davies, Poe introduced evidence thatEllis made an excited utterance that Davies ranthe red light.

Davies called Witt to testify that later Ellis, abystander, now deceased, told Witt that Davieswent through a yellow light.

Witt’s testimony should be

(A) excluded, because it is hearsay notwithin any exception.

(B) excluded, because Ellis is not availableto explain or deny the inconsistency.

(C) admitted only for the purpose ofimpeaching Ellis.

(D) admitted as impeachment and assubstantive evidence of the color of thelight.

GO ON TO THE NEXT PAGE.

-32-

Page 35: Sample MBE II

66. Plaintiff, a jockey, was seriously injured in arace when another jockey, Daring, cut toosharply in front of her without adequateclearance. The two horses collided, causingPlaintiff to fall to the ground, sustaining injury.The State Racetrack Commission ruled that,by cutting in too sharply, Daring committed afoul in violation of racetrack rules requiringadequate clearance for crossing lanes. Plaintiffhas brought an action against Daring fordamages in which one count is based onbattery.

Will Plaintiff prevail on the battery claim?

(A) Yes, if Daring was reckless in cuttingacross in front of Plaintiff’s horse.

(B) Yes, because the State RacetrackCommission determined that Daringcommitted a foul in violation of rulesapplicable to racing.

(C) No, unless Daring intended to causeimpermissible contact between the twohorses or apprehension of such contactby Plaintiff.

(D) No, because Plaintiff assumed the risk ofaccidental injury inherent in riding as ajockey in a horse race.

67. Able entered into a written contract withBaker to sell Greenacre. The contract wasdated June 19 and called for a closing date onthe following August 19. There was no otherprovision in the contract concerning theclosing date. The contract contained thefollowing clause: “subject to the purchaser,Baker, obtaining a satisfactory mortgage at thecurrent rate.” On the date provided for closing,Baker advised Able that he was unable toclose because his mortgage application wasstill being processed by a bank. Able desiredto declare the contract at an end and consultedhis attorney in regard to his legal position.

Which of the following are relevant inadvising Able of his legal position?

I. Is time of the essence?II. Parol evidence rule.III. Statute of Frauds.IV. Specific performance.

(A) I and III only.(B) II and IV only.(C) II, III, and IV only.(D) I, II, III, and IV.

GO ON TO THE NEXT PAGE.

-33-

Page 36: Sample MBE II

68. Lester was engaged to marry Sylvia. Oneevening, Lester became enraged at thecomments of Sylvia’s eight-year-old daughter,Cynthia, who was complaining, in her usualfashion, that she did not want her mother tomarry Lester. Lester, who had had too much todrink, began beating her. Cynthia sufferedsome bruises and a broken arm. Sylvia tookCynthia to the hospital. The police werenotified by the hospital staff. Lester wasindicted for felony child abuse.

Lester pleaded with Sylvia to forgive him andto run away with him. She agreed. Theymoved out of state and took Cynthia withthem. Without the testimony of the child, theprosecution was forced to dismiss the case.

Some time later, Sylvia returned for a visitwith her family and was arrested and indictedas an accessory-after-the-fact to child abuse.

At her trial, the court should

(A) dismiss the charge, because Lester hadnot been convicted.

(B) dismiss the charge, because the evidenceshows that any aid she rendered occurredafter the crime was completed.

(C) submit the case to the jury, on aninstruction to convict only if Sylviaknew Lester had been indicted.

(D) submit the case to the jury, on aninstruction to convict only if her purposein moving was to prevent Lester’sconviction.

69. In response to massive layoffs of employees ofautomobile assembly plants located in the stateof Ames, the legislature of that state enacted astatute which prohibits the parking ofautomobiles manufactured outside of theUnited States in any parking lot or parkingstructure that is owned or operated by the stateor any of its instrumentalities. This statutedoes not apply to parking on public streets.

Which of the following is the strongestargument with which to challenge theconstitutionality of this statute?

(A) The statute imposes an undue burden onforeign commerce.

(B) The statute denies the owners of foreign-made automobiles the equal protection ofthe laws.

(C) The statute deprives the owners offoreign-made automobiles of liberty orproperty without due process of law.

(D) The statute is inconsistent with theprivileges and immunities clause of theFourteenth Amendment.

GO ON TO THE NEXT PAGE.

-34-

Page 37: Sample MBE II

70. Pate sued Dr. Doke for psychiatric malpracticeand called Dr. Will as an expert witness.During Will’s direct testimony, Will identifieda text as a reliable authority in the field. Heseeks to read to the jury passages from thisbook on which he had relied in forming hisopinion on the proper standard of care.

The passage is

(A) admissible, as a basis for his opinion andas substantive evidence of the properstandard of care.

(B) admissible, as a basis for his opinion butnot as substantive evidence of the properstandard of care.

(C) inadmissible, because a witness’scredibility cannot be supported unlessattacked.

(D) inadmissible, because the passage shouldbe received as an exhibit and not read tothe jury by the witness.

71. The Daily Sun, a newspaper, printed an articlethat stated:

Kitchen, the popular restaurant on thetown square, has closed its doors.Kitchen employees have told the DailySun that the closing resulted from theowner’s belief that Kitchen’s generalmanager has embezzled thousands ofdollars from the restaurant over the lastseveral years. A decision on reopeningthe restaurant will be made after thecompletion of an audit of Kitchen’sbooks.

Plaintiff, who is Kitchen’s general manager,brought a libel action against the Daily Sunbased on the publication of this article. Theparties stipulated that Plaintiff neverembezzled any funds from Kitchen. They alsostipulated that Plaintiff is well known amongmany people in the community because of hisjob with Kitchen.

The case went to trial before a jury.

The defendant’s motion for a directed verdictin its favor, made at the close of the evidence,should be granted if the

(A) record contains no evidence that Plaintiffsuffered special harm as a result of thepublication.

(B) record contains no evidence that thedefendant was negligent as to the truth orfalsity of the charge of embezzlement.

(C) evidence is not clear and convincing thatthe defendant published the article with“actual malice.”

(D) record contains uncontradicted evidencethat the article accurately reported whatthe employees told the Daily Sun.

GO ON TO THE NEXT PAGE.

-35-

Page 38: Sample MBE II

72. Surgeon performed a sterilization operation onPatient. After the surgery, Surgeon performeda test that showed that Patient’s fallopian tubeswere not severed, as was necessary forsterilization. Surgeon did not reveal the failureof the operation to Patient, who three yearslater became pregnant and delivered a babyafflicted with a severe birth defect that willrequire substantial medical care throughout itslife. The birth defect resulted from a geneticdefect unknown to, and undiscoverable by,Surgeon. Patient brought an action on her ownbehalf against Surgeon, seeking to recover thecost of her medical care for the delivery of thebaby, and the baby’s extraordinary futuremedical expenses for which Patient will beresponsible.

Which of the following questions is relevantto the lawsuit and currently most difficult toanswer?

(A) Did Surgeon owe a duty of care to thebaby in respect to medical servicesrendered to Patient three years before thebaby was conceived?

(B) Can a person recover damages for a lifeburdened by a severe birth defect basedon a physician’s wrongful failure toprevent that person’s birth fromoccurring?

(C) Did Surgeon owe a duty to Patient toinform her that the sterilization operationhad failed?

(D) Is Patient entitled to recover damages forthe baby’s extraordinary future medicalexpenses?

73. Robert walked into a store that had a check-cashing service and tried to cash a $550 checkwhich was payable to him. The attendant onduty refused to cash the check because Robertdid not have two forms of identification,which the store’s policies required. Robert,who had no money except for the check andwho needed cash to pay for food and a placeto sleep, became agitated. He put his hand intohis pocket and growled, “Give me the moneyor I’ll start shooting.” The attendant, whoknew Robert as a neighborhood character, didnot believe that he was violent or had a gun.However, because the attendant felt sorry forRobert, he handed over the cash. Robert leftthe check on the counter and departed. Theattendant picked up the check and found thatRobert had failed to endorse it.

If Robert is guilty of any crime, he is mostlikely guilty of

(A) robbery.(B) attempted robbery.(C) theft by false pretenses.(D) larceny by trick.

GO ON TO THE NEXT PAGE.

-36-

Page 39: Sample MBE II

Questions 74-75 are based on the following fact situation.

Kabb, the owner of a fleet of taxis, contracted withPetrol, a dealer in petroleum products, for thepurchase and sale of Kabb’s total requirements ofgasoline and oil for one year. As part of thatagreement, Petrol also agreed with Kabb that forone year Petrol would place all his advertising withAda Artiste, Kabb’s wife, who owned her own smalladvertising agency. When Artiste was informed ofthe Kabb-Petrol contract, she declined to accept anadvertising account from the Deturgid SoapCompany because she could not handle both thePetrol and Deturgid accounts during the same year.

74. For this question only, assume the followingfacts. During the first month of the contract,Kabb purchased substantial amounts of hisgasoline from a supplier other than Petrol, andPetrol thereupon notified Artiste that he wouldno longer place his advertising with heragency.

In an action against Petrol for breach ofcontract, Artiste probably will

(A) succeed, because she is a third-partybeneficiary of the Kabb-Petrol contract.

(B) succeed, because Kabb was acting asArtiste’s agent when he contracted withPetrol.

(C) not succeed, because the failure of aconstructive condition precedent excusedPetrol’s duty to place his advertising withArtiste.

(D) not succeed, because Artiste did notprovide any consideration to supportPetrol’s promise to place his advertisingwith her.

75. For this question only, make the followingassumptions. Artiste was an intendedbeneficiary under the Kabb-Petrol contract.Kabb performed his contract with Petrol forsix months, and during that time Petrol placedhis advertising with Artiste. At the end of thesix months, Kabb and Artiste were divorced,and Kabb then told Petrol that he had nofurther obligation to place his advertising withArtiste. Petrol thereupon notified Artiste thathe would no longer place his advertising withher.

In an action against Petrol for breach ofcontract, Artiste probably will

(A) succeed, because, on the facts of thiscase, Petrol and Kabb could not, withoutArtiste’s consent, modify their contractso as to discharge Petrol’s duties toArtiste.

(B) succeed, because Kabb acted in bad faithin releasing Petrol from his duty withrespect to Artiste.

(C) not succeed, because, absent a provisionin the contract to the contrary, thepromisor and promisee of a third-partybeneficiary contract retain by law theright to modify or terminate the contract.

(D) not succeed, because the agencyrelationship, if any, between Kabb andArtiste terminated upon their divorce.

GO ON TO THE NEXT PAGE.

-37-

Page 40: Sample MBE II

76. Drew, the owner of a truck leasing company,asked Pat, one of Drew’s employees, to deliver$1,000 to the dealership’s main office. Thefollowing week, as a result of a dispute overwhether the money had been delivered, Drewinstructed Pat to come to the office to submitto a lie detector test.

When Pat reported to Drew’s office for thetest, it was not administered. Instead, withouthearing Pat’s story, Drew shouted at him,“You’re a thief!” and fired him. Drew’s shoutwas overheard by several other employeeswho were in another office, which wasseparated from Drew’s office by a thinpartition. The next day, Pat accepted anotherjob at a higher salary. Several weeks later,upon discovering that the money had not beenstolen, Drew offered to rehire Pat.

In a suit for slander by Pat against Drew, Patwill

(A) prevail, because Pat was fraudulentlyinduced to go to the office for a liedetector test, which was not, in fact,given.

(B) prevail, if Drew should have foreseenthat the statement would be overheard byother employees.

(C) not prevail, if Drew made the charge ingood faith, believing it to be true.

(D) not prevail, because the statement wasmade to Pat alone and intended for hisears only.

77. Adam owns his home, Blackacre, which wasmortgaged to Bank by a duly recordedpurchase money mortgage. Last year, Adamreplaced all of Blackacre’s old windows withnew windows.

Each new window consists of a window framewith three inserts: regular windows, stormwindows, and screens. The windows aredesigned so that each insert can be easilyinserted or removed from the window framewithout tools to adjust to seasonal change andto facilitate the cleaning of the inserts.

The new windows were expensive. Adampurchased them on credit, signed a financingstatement, and granted a security interest in thewindows to Vend, the supplier of the windows.Vend promptly and properly filed and recordedthe financing statement before the windowswere installed. Adam stored the old windowsin the basement of Blackacre.

This year, Adam has suffered severe financialreverses and has defaulted on his mortgageobligation to Bank and on his obligation toVend.

Bank brought an appropriate action to enjoinVend from its proposed repossession of thewindow inserts.

In the action, the court should rule for

(A) Bank, because its mortgage was recordedfirst.

(B) Bank, because windows and screens, nomatter their characteristics, are anintegral part of a house.

(C) Vend, because the inserts are removable.(D) Vend, because the availability of the old

windows enables Bank to returnBlackacre to its original condition.

GO ON TO THE NEXT PAGE.

-38-

Page 41: Sample MBE II

-39-

78. In a suit by Palmer against Denby, Palmersought to subpoena an audiotape on whichDenby had narrated his version of the disputefor his attorney. Counsel for Denby moves toquash the subpoena on the ground of privilege.

The audiotape is most likely to be subject tosubpoena if

(A) Denby played the audiotape for his fatherto get his reactions.

(B) the lawsuit involved alleged criminalbehavior by Denby.

(C) Denby has been deposed and there isgood reason to believe that the audiotapemay contain inconsistent statements.

(D) Denby is deceased and thus unavailableto give testimony in person.

79. The National Ecological Balance Act prohibitsthe destruction or removal of any wild animalslocated on lands owned by the United Stateswithout express permission from the FederalBureau of Land Management. Violators aresubject to fines of up to $1,000 per offense.

After substantial property damage wasinflicted on residents of the state of Arkota byhungry coyotes, the state legislature passed theCoyote Bounty Bill, which offers $25 for eachcoyote killed or captured within the state. TheKota National Forest, owned by the federalgovernment, is located entirely within the stateof Arkota. Many coyotes live in the KotaNational Forest.

Without seeking permission from the Bureauof Land Management, Hunter shot severalcoyotes in the Kota National Forest andcollected the bounty from the state of Arkota.As a result, he was subsequently tried infederal district court, convicted, and fined$1,000 for violating the National EcologicalBalance Act. Hunter appealed his conviction tothe United States Court of Appeals.

On appeal, the Court of Appeals should holdthe National Ecological Balance Act, asapplied to Hunter, to be

(A) constitutional, because the propertyclause of Article IV, Section 3, of theConstitution authorizes such federalstatutory controls and sanctions.

(B) constitutional, because Article I, Section8, of the Constitution authorizesCongress to enact all laws necessary andproper to advance the general welfare.

(C) unconstitutional, because Congress maynot use its delegated powers to overridethe Tenth Amendment right of the stateof Arkota to legislate in areas oftraditional state governmental functions,such as the protection of the property ofits residents.

(D) unconstitutional, because Congressviolates the full faith and credit clause ofArticle IV when it punishes conduct thathas been authorized by state action.

GO ON TO THE NEXT PAGE.

Page 42: Sample MBE II

80. A kidnapping statute in State A makes it acrime for a person, including a parent, to “takea child from the custody of his custodialparent, knowing he has no privilege to do so.”

After a bitter court battle Ann and Dave weredivorced and Ann was given custody of theirdaughter, Maria. Dave later moved to State Bwhere he brought an action to obtain custodyof Maria. A local judge awarded him custody.His attorney incorrectly advised him that,under this award, he was entitled to take Mariaaway from Ann. Dave drove to State A, pickedMaria up at her preschool, and took her backto State B with him.

He was indicted for kidnapping in State A,extradited from State B, and tried. At trial, hetestified that he had relied on his attorney’sadvice in taking Maria, and that at the time hebelieved his conduct was not illegal.

If the jury believes his testimony, Dave shouldbe

(A) acquitted, because he acted on the adviceof an attorney.

(B) acquitted, because he lacked a necessarymental element of the crime.

(C) convicted, because reliance on anattorney’s advice is not a defense.

(D) convicted, provided a reasonable personwould have known that the attorney’sadvice was erroneous.

81. Owen, the owner of Greenacre, a tract of land,mortgaged Greenacre to ABC Bank to securehis preexisting obligation to ABC Bank. Themortgage was promptly and properly recorded.Owen and Newton then entered into a validwritten contract for the purchase and sale ofGreenacre, which provided for the transfer of“a marketable title, free of encumbrances.”The contract did not expressly refer to themortgage.

Shortly after entering into the contract,Newton found another property that muchbetter suited her needs and decided to try toavoid her contract with Owen. When Newtondiscovered the existence of the mortgage, sheasserted that the title was encumbered and thatshe would not close. Owen responded byoffering to provide for payment and dischargeof the mortgage at the closing from theproceeds of the closing. Newton refused to goforward, and Owen brought an appropriateaction against her for specific performance.

If the court holds for Owen in this action, itwill most likely be because

(A) the mortgage is not entitled to prioritybecause it was granted for preexistingobligations.

(B) the doctrine of equitable conversionsupports the result.

(C) Owen’s arrangements for the payment ofthe mortgage fully satisfied Owen’sobligation to deliver marketable title.

(D) the existence of the mortgage was notNewton’s real reason for refusing toclose.

GO ON TO THE NEXT PAGE.

-40-

Page 43: Sample MBE II

82. Pawn sued Dalton for injuries received whenshe fell down a stairway in Dalton’s apartmentbuilding. Pawn, a guest in the building, allegedthat she caught the heel of her shoe in a tear inthe stair carpet. Pawn calls Witt, a tenant, totestify that Young, another tenant, had said tohim a week before Pawn’s fall: “When I paidmy rent this morning, I told the manager hehad better fix that torn carpet.”

Young’s statement, reported by Witt, is

(A) admissible, to prove that the carpet wasdefective.

(B) admissible, to prove that Dalton hadnotice of the defect.

(C) admissible, to prove both that the carpetwas defective and that Dalton had noticeof the defect.

(D) inadmissible, because it is hearsay notwithin any exception.

83. A law of the state of Wonatol imposed agenerally applicable sales tax payable by thevendor. That law exempted from its provisionsthe sale of “all magazines, periodicals,newspapers, and books.” In order to raiseadditional revenue, the state legislatureeliminated that broad exemption andsubstituted a narrower exemption. The new,narrower exemption excluded from the statesales tax only the sale of those “magazines,periodicals, newspapers, and books that arepublished or distributed by a recognizedreligious faith and that consist wholly ofwritings sacred to such a religious faith.”

Magazine is a monthly publication devoted tohistory and politics. Magazine paid underprotest the sales tax due on its sales accordingto the amended sales tax law. Magazine thenfiled suit against the state in an appropriatestate court for a refund of the sales taxes paid.It contended that the state’s elimination of theearlier, broader exemption and adoption of thenew, narrower exemption restricted to sacredwritings of recognized religious faiths violatesthe First and Fourteenth Amendments to theConstitution.

In this case, the court will probably rule that

(A) Magazine lacks standing to sue for arefund of sales taxes imposed by agenerally applicable state law becauseArticle III of the Constitution precludestaxpayers from bringing such suits.

(B) the Eleventh Amendment bars the statecourt from exercising jurisdiction overthis suit in the absence of a law ofWonatol expressly waiving the state’simmunity.

(C) the new, narrower exemption from thestate sales tax law violates theestablishment clause of the First andFourteenth Amendments by grantingpreferential state support to recognizedreligious faiths for the communication oftheir religious beliefs.

(D) the new, narrower exemption from thestate sales tax law violates the freedomof the press guaranteed by the First andFourteenth Amendments because itimposes a prior restraint on nonreligiouspublications that are required to pay thetax.

GO ON TO THE NEXT PAGE.

-41-

Page 44: Sample MBE II

-42-

84. For five years, Rancher had kept his horse in aten-acre field enclosed by a six-foot wovenwire fence with six inches of barbed wire ontop. The gate to the field was latched andcould not be opened by an animal. Rancherhad never had any trouble with people comingonto his property and bothering the horse, andthe horse had never escaped from the field.One day, however, when Rancher went to thefield, he found that the gate was open and thehorse was gone. Shortly before Rancher’sdiscovery, Driver was driving with due care ona nearby highway when suddenly Rancher’shorse darted in front of his car. When Driverattempted to avoid hitting the horse, he lostcontrol of the car, which then crashed into atree. Driver was injured.

Driver sued Rancher to recover damages forhis injuries and Rancher moved for summaryjudgment.

If the facts stated above are undisputed, thejudge should

(A) deny the motion, because, pursuant tothe doctrine of res ipsa loquitur, a jurycould infer that Rancher was negligent.

(B) deny the motion, because an animaldangerous to highway users escapedfrom Rancher’s property and caused thecollision.

(C) grant the motion, because there is noevidence that Rancher was negligent.

(D) grant the motion, because Rancher didnot knowingly permit the horse to run atlarge.

85. Defendant was prosecuted for bankruptcyfraud. Defendant’s wife, now deceased, hadtestified adversely to Defendant during earlierbankruptcy proceedings that involved similarissues. Although the wife had been cross-examined, no serious effort was made tochallenge her credibility despite theavailability of significant impeachmentinformation. At the fraud trial, the prosecutoroffers into evidence the testimony given byDefendant’s wife at the bankruptcyproceeding.

This evidence should be

(A) admitted, under the hearsay exceptionfor former testimony.

(B) admitted, because it is a statement by aperson identified with a party.

(C) excluded, because it is hearsay notwithin any exception.

(D) excluded, because Defendant has theright to prevent use of his spouse’stestimony against him in a criminal case.

GO ON TO THE NEXT PAGE.

Page 45: Sample MBE II

Questions 86-87 are based on the following fact situation.

Mermaid owns an exceptionally seaworthy boat thatshe charters for sport fishing at a $500 daily rate.The fee includes the use of the boat with Mermaidas the captain, and one other crew member, as wellas fishing tackle and bait. On May 1, Phinneyagreed with Mermaid that Phinney would have thefull-day use of the boat on May 15 for himself andhis family for $500. Phinney paid an advancedeposit of $200 and signed an agreement that thedeposit could be retained by Mermaid as liquidateddamages in the event Phinney canceled or failed toappear.

86. For this question only, assume the followingfacts. At the time of contracting, Mermaid toldPhinney to be at the dock at 5 a.m. on May 15.Phinney and his family, however, did not showup on May 15 until noon. Meantime, Mermaidagreed at 10 a.m. to take Tess and her familyout fishing for the rest of the day. Tess hadhappened to come by and inquire about thepossibility of such an outing. In view of thelate hour, Mermaid charged Tess $400 andstayed out two hours beyond the customaryreturn time. Phinney’s failure to appear untilnoon was due to the fact that he had beentrying to charter another boat across the bay ata lower rate and had gotten lost after he wasunsuccessful in getting such a charter.

Which of the following is an accuratestatement concerning the rights of the parties?

(A) Mermaid can retain the $200 paid byPhinney, because it would be difficult forMermaid to establish her actual damagesand the sum appears to have been areasonable forecast in light of anticipatedloss of profit from the charter.

(B) Mermaid is entitled to retain only $50(10% of the contract price) and mustreturn $150 to Phinney.

(C) Mermaid must return $100 to Phinney inorder to avoid her own unjust enrichmentat Phinney’s expense.

(D) Mermaid must return $100 to Phinney,because the liquidated-damage clauseunder the circumstances would operateas a penalty.

GO ON TO THE NEXT PAGE.

-43-

Page 46: Sample MBE II

87. For this question only, assume the followingfacts. On May 15 at 1 a.m., the Coast Guardhad issued offshore “heavy weather” warningsand prohibited all small vessels the size ofMermaid’s from leaving the harbor. Thisprohibition remained in effect throughout theday. Phinney did not appear at all on May 15,because he had heard the weather warnings onhis radio.

Which of the following is an accuratestatement?

(A) The contract is discharged because ofimpossibility, and Phinney is entitled toreturn of his deposit.

(B) The contract is discharged because ofmutual mistake concerning an essentialfact, and Phinney is entitled to return ofhis deposit.

(C) The contract is not discharged, becauseits performance was possible in view ofthe exceptional seaworthiness ofMermaid’s boat, and Phinney is notentitled to return of his deposit.

(D) The contract is not discharged, andPhinney is not entitled to return of hisdeposit, because the liquidated-damageclause in effect allocated the risk of badweather to Phinney.

88. Eight years ago, Orben, prior to moving to adistant city, conveyed Blackacre, an isolatedfarm, to his son, Sam, by a quitclaim deed.Sam paid no consideration. Sam, who was 19years old, without formal education, andwithout experience in business, tookpossession of Blackacre and operated the farmbut neglected to record his deed. Subsequently,Orben conveyed Blackacre to Fred bywarranty deed. Fred, a substantial land andtimber promoter, paid valuable considerationfor the deed to him. He was unaware of Sam’spossession, his quitclaim deed, or hisrelationship to Orben. Fred promptly andproperly recorded his deed and beganremoving timber from the land. Immediatelyupon learning of Fred’s actions, Sam recordedhis deed and brought an appropriate action toenjoin Fred from removing the timber and toquiet title in Sam. The recording act of thejurisdiction provides:

“No conveyance or mortgage of realproperty shall be good againstsubsequent purchasers for value andwithout notice unless the same berecorded according to law.”

In this action, Fred should

(A) prevail, because a warranty deed forvaluable consideration takes priority overa quitclaim deed without consideration.

(B) prevail, because Orben’s subsequentconveyance to Fred revoked the gift toSam.

(C) lose, because Sam’s possession chargedFred with notice.

(D) lose, because the equities favor Sam.

GO ON TO THE NEXT PAGE.

-44-

Page 47: Sample MBE II

89. Brown owned Blackacre, a tract ofundeveloped land. Blackacre abuts Whiteacre,a tract of land owned by Agency, the state’sgovernmental energy agency. At Whiteacre,Agency has operated a waste-to-electricityrecycling facility for 12 years. Blackacre andWhiteacre are in a remote area and Whiteacreis the only developed parcel of real estatewithin a ten-mile radius. The boundary linebetween Blackacre and Whiteacre had neverbeen surveyed or marked on the face of theearth.

During the past 12 years, some of the trucksbringing waste to the Agency facility havedumped their loads so that the piles of wasteextend from Whiteacre onto a portion ofBlackacre. However, prior to the four-weekperiod during each calendar year when theAgency facility is closed for inspection andrepairs, the waste piles are reduced to minimallevels so that during each of the four-weekclosures no waste was, in fact, piled onBlackacre. Neither Brown nor anyrepresentative of Agency knew the facts aboutthe relation of the boundary line to the wastepiles.

The time for acquiring title by adversepossession in the jurisdiction is ten years.

Last year, Brown died, and his son, Silas,succeeded him as the owner of Blackacre.Silas became aware of the facts, demandedthat Agency stop using Blackacre for thepiling of waste, and, when Agency refused hisdemand, brought an appropriate action toenjoin any such use of Blackacre in the future.

If Agency prevails in that action, it will bebecause

(A) the facts constitute adverse possessionand title to the portion of Blackacreconcerned has vested in Agency.

(B) Brown’s failure to keep himself informedas to Agency’s use of Blackacre and hisfailure to object constituted impliedconsent to the continuation of that use.

(C) the interest of the public in theconversion of waste to energy overridesany entitlement of Silas to equitableremedies.

(D) the power of eminent domain of the statemakes the claim of Silas moot.

90. Defendant was charged with possession ofcocaine with intent to distribute. He had beenstopped while driving a car and several poundsof cocaine were found in the trunk. In hisopening statement, defendant’s counselasserted that his client had no key to the trunkand no knowledge of its contents. Theprosecutor offers the state motor vehicleregistration, shown to have been found in theglove compartment of the car, listingDefendant as the owner.

The registration should be

(A) admitted, as a statement against interest.(B) admitted, as evidence of Defendant’s

close connection with the car and,therefore, knowledge of its contents.

(C) excluded, unless authenticated bytestimony of or certification by a stateofficial charged with custody of vehicleregistration records.

(D) excluded, as hearsay not within anyexception.

GO ON TO THE NEXT PAGE.

-45-

Page 48: Sample MBE II

91. Donald was arrested in Marilyn’s apartmentafter her neighbors had reported sounds of astruggle and the police had arrived to findDonald bent over Marilyn’s prostrate body.Marilyn was rushed to the hospital where shelapsed into a coma. Despite the explanationthat he was trying to revive Marilyn after shesuddenly collapsed, Donald was charged withattempted rape and assault after a neighborinformed the police that she had heard Marilynsobbing, “No, please no, let me alone.”

At trial, the forensic evidence wasinconclusive. The jury acquitted Donald ofattempted rape but convicted him of assault.While he was serving his sentence for assault,Marilyn, who had never recovered from thecoma, died. Donald was then indicted andtried on a charge of felony murder. In thiscommon-law jurisdiction, there is no statutethat prevents a prosecutor from proceeding inthis manner, but Donald argued that a secondtrial for felony murder after his original trialfor attempted rape and assault would violatethe double jeopardy clause.

His claim is

(A) correct, because he was acquitted of theattempted rape charge.

(B) correct, because he was convicted of theassault charge.

(C) incorrect, because Marilyn had not diedat the time of the first trial and he wasnot placed in jeopardy for murder.

(D) incorrect, because he was convicted ofthe assault charge.

92. Ogle owned Greenacre, a tract of land, in feesimple. Five years ago, he executed anddelivered to Lilly an instrument in the properform of a warranty deed that conveyedGreenacre to Lilly “for and during the term ofher natural life.” No other estate or interest orperson taking an interest was mentioned. Lillytook possession of Greenacre and hasremained in possession.

Fifteen months ago, Ogle died, leaving a willthat has been duly admitted to probate. Thewill, inter alia, had the following provision:

“I devise Greenacre to Mina for hernatural life and from and after Mina’sdeath to Rex, his heirs and assigns,forever.”

Administration of Ogle’s estate has beencompleted. Mina claims the immediate right topossession of Greenacre. Rex also asserts aright to immediate possession.

In an appropriate lawsuit to which Lilly, Mina,and Rex are parties, who should be adjudgedto have the right to immediate possession?

(A) Lilly, because no subsequent act of Oglewould affect her life estate.

(B) Mina, because Ogle’s will was the finaland definitive expression of his intent.

(C) Mina, because Lilly’s estate terminatedwith the death of Ogle.

(D) Rex, because Lilly’s estate terminatedwith Ogle’s death and all that Ogle hadwas the right to transfer his reversion infee simple.

GO ON TO THE NEXT PAGE.

-46-

Page 49: Sample MBE II

93. Devlin was charged with murder. Severalwitnesses testified that the crime wascommitted by a person of Devlin’s generaldescription who walked with a severe limp.Devlin in fact walks with a severe limp. Heobjected to a prosecution request that the courtorder him to walk across the courtroom inorder to display his limp to the jury to assist itin determining whether Devlin was the personthat the witnesses had seen.

Devlin’s objection will most likely be

(A) sustained, because the order sought bythe prosecution would violate Devlin’sprivilege against self-incrimination.

(B) sustained, because the order sought bythe prosecution would constitute anillegal search and seizure.

(C) denied, because the order sought by theprosecution is a legitimate part of aproper courtroom identification process.

(D) denied, because a criminal defendant hasno legitimate expectation of privacy.

94. A statute of the state of Kiowa provided statemonetary grants to private dance, theater, andopera groups located in that state. The statuterequired recipients of such grants to use thegranted monies for the acquisition,construction, and maintenance of appropriatefacilities for the public performance of theirperforming arts. The last section of the statuteconditioned the award of each such grant onthe recipient’s agreement to refrain from allkinds of political lobbying calculated to secureadditional tax support for the performing arts.

The strongest constitutional basis for an attackupon the validity of the last section of thestatute would be based upon the

(A) commerce clause.(B) obligation of contracts clause.(C) Fifth Amendment.(D) First and Fourteenth Amendments.

95. Penkov suffered a severe loss when hismanufacturing plant, located in a shallowravine, was flooded during a sustained rainfall.The flooding occurred because City had failedto maintain its storm drain, which was locatedon City land above Penkov’s premises, andbecause Railroad had failed to maintain itsstorm drain, which was located on Railroadland below Penkov’s premises. The floodingwould not have occurred if either one of thetwo storm drains had been maintainedproperly.

Penkov sued Railroad to recover compensationfor his loss. The evidence in the caseestablished that the failures of the two drainswere caused by the respective negligence ofCity and Railroad. There is no special ruleinsulating City from liability.

In his action against Railroad, Penkov shouldrecover

(A) nothing, because he should have joinedCity, without whose negligence he wouldhave suffered no loss.

(B) nothing, unless he introduces evidencethat enables the court reasonably toapportion responsibility between Cityand Railroad.

(C) one-half his loss, in the absence ofevidence that enables the court toallocate responsibility fairly betweenCity and Railroad.

(D) all of his loss, because but for Railroad’snegligence none of the flooding wouldhave occurred.

GO ON TO THE NEXT PAGE.

-47-

Page 50: Sample MBE II

96. Smith asked Jones if he would loan him $500,promising to repay the amount within twoweeks. Jones loaned him the $500. The nextday Smith took the money to the race trackand lost all of it betting on horse races. Hethen left town for six months. He has notrepaid Jones.

Smith has committed

(A) both larceny by trick and obtainingmoney by false pretenses (although hecan only be convicted of one offense).

(B) larceny by trick only.(C) obtaining money by false pretenses only.(D) neither larceny by trick nor obtaining

money by false pretenses.

97. Assume that Congress passed and thePresident signed the following statute:

“The appellate jurisdiction of the United StatesSupreme Court shall not extend to any caseinvolving the constitutionality of any statestatute limiting the circumstances in which awoman may obtain an abortion, or involvingthe constitutionality of this statute.”

The strongest argument against theconstitutionality of this statute is that

(A) Congress may not exercise its authorityover the appellate jurisdiction of theSupreme Court in a way that seriouslyinterferes with the establishment of asupreme and uniform body of federalconstitutional law.

(B) Congress may only regulate the appellatejurisdiction of the Supreme Court overcases initially arising in federal courts.

(C) the appellate jurisdiction of the SupremeCourt may only be altered byconstitutional amendment.

(D) the statute violates the equal protectionclause of the Fourteenth Amendment.

98. The federal statute admitting the state of Blueto the Union granted Blue certain public lands,and established some very ambiguousconditions on the subsequent disposition ofthese lands by Blue. This federal statute alsorequired the new state to write those exactsame conditions into its state constitution. Onehundred years later, a statute of Blue dealingwith the sale of these public lands waschallenged in a state court lawsuit on theground that it was inconsistent with theconditions contained in the federal statute, andwith the provisions of the Blue Constitutionthat exactly copy the conditions contained inthe federal statute. The trial court decision inthis case was appealed to the Blue SupremeCourt. In its opinion, the Blue Supreme Courtdealt at length with the ambiguous language ofthe federal statute and with cases interpretingidentical language in federal statutes admittingother states to the union. The Blue SupremeCourt opinion did not discuss the similarprovisions of the Blue Constitution, but it didhold that the challenged Blue statute is invalidbecause it is “inconsistent with the language ofthe federal statute and therefore is inconsistentwith the identical provisions of our stateconstitution.”

If the losing party in the Blue Supreme Courtseeks review of the decision of that court inthe United States Supreme Court, the UnitedStates Supreme Court should

(A) accept the case for review and determinethe validity and interpretation of thefederal statute if it is an important andsubstantial question.

(B) ask the Blue Supreme Court to indicatemore clearly whether it relied on thestate constitutional provision inrendering its decision.

(C) decline to review the case on the groundthat the decision of the Blue SupremeCourt rests on an adequate andindependent state ground.

(D) decline to review the case because adecision by a state supreme courtconcerning the proper disposition of statepublic lands is not reviewable by theUnited States Supreme Court.

GO ON TO THE NEXT PAGE.

-48-

Page 51: Sample MBE II

-49-

99. Trawf, the manager of a state fair, contractedwith Schweinebauch, a renowned hog breeder,to exhibit Schweinebauch’s world championanimal, Megahawg, for the three weeks of theannual fair, at the conclusion of whichSchweinebauch would receive an honorariumof $300. Two days before the opening of thefair, Megahawg took sick with boarsitis, acommunicable disease among swine, and,under the applicable state quarantine law, veryprobably could not be exhibited for at least amonth.

Upon learning this, Trawf can legally pursuewhich of the following courses of action withrespect to his contract with Schweinebauch?

(A) Suspend his own performance, demandassurances from Schweinebauch, andtreat a failure by Schweinebauch to givethem as an actionable repudiation.

(B) Suspend his own performance andrecover damages from Schweinebauchfor breach of contract unlessSchweinebauch at once supplies anundiseased hog of exhibition quality as asubstitute for Megahawg.

(C) Terminate his own performance and treatMegahawg’s illness as discharging allremaining duties under the contract.

(D) Terminate the contract, but only if he(Trawf) seeks promptly to obtain for theexhibit a suitable substitute forMegahawg from another hog owner.

100. The manager of a department store noticedthat Paula was carrying a scarf with her as sheexamined various items in the blousedepartment. The manager recognized the scarfas an expensive one carried by the store. Paulawas trying to find a blouse that matched acolor in the scarf, and, after a while, foundone. The manager then saw Paula put the scarfinto her purse, pay for the blouse, and head forthe door. The manager, who was eight inchestaller than Paula, blocked Paula’s way to thedoor and asked to see the scarf in Paula’spurse. Paula produced the scarf, as well as areceipt for it, showing that it had beenpurchased from the store on the previous day.The manager then told Paula there was noproblem, and stepped out of her way.

If Paula brings a claim against the store basedon false imprisonment, the store’s best defensewould be that

(A) by carrying the scarf in public view andthen putting it into her purse, Paulaassumed the risk of being detained.

(B) the manager had a reasonable belief thatPaula was shoplifting and detained heronly briefly for a reasonableinvestigation of the facts.

(C) Paula should have realized that herconduct would create a reasonable beliefthat facts existed warranting a privilegeto detain.

(D) Paula was not detained, but was merelyquestioned about the scarf.

STOPIF YOU FINISH BEFORE TIME IS CALLED, CHECK YOUR WORK ON THIS TEST.

Page 52: Sample MBE II

-50-

101. A proposed federal statute would prohibit alltypes of discrimination against black personson the basis of their race in every businesstransaction executed anywhere in the UnitedStates by any person or entity, governmentalor private.

Is this proposed federal statute likely to beconstitutional?

(A) Yes, because it could reasonably beviewed as an exercise of Congress’sauthority to enact laws for the generalwelfare.

(B) Yes, because it could reasonably beviewed as a means of enforcing theprovisions of the Thirteenth Amendment.

(C) No, because it would regulate purelylocal transactions that are not ininterstate commerce.

(D) No, because it would invade the powersreserved to the states by the TenthAmendment.

102. Sam told Horace, his neighbor, that he wasgoing away for two weeks and asked Horaceto keep an eye on his house. Horace agreed.Sam gave Horace a key to use to check on thehouse.

Horace decided to have a party in Sam’shouse. He invited a number of friends. Onefriend, Lewis, went into Sam’s bedroom, tooksome of Sam’s rings, and put them in hispocket.

Which of the following is true?

(A) Horace and Lewis are guilty of burglary.(B) Horace is guilty of burglary and Lewis is

guilty of larceny.(C) Horace is guilty of trespass and Lewis is

guilty of larceny.(D) Lewis is guilty of larceny and Horace is

not guilty of any crime.

GO ON TO THE NEXT PAGE.

PM BOOKTIME—3 HOURS

Directions: Each of the questions or incomplete statements below is followed by four suggested answers orcompletions. You are to choose the best of the stated alternatives. Answer all questions according to thegenerally accepted view, except where otherwise noted.

For the purposes of this test, you are to assume that Articles 1 and 2 of the Uniform Commercial Code havebeen adopted. You are also to assume relevant application of Article 9 of the UCC concerning fixtures. TheFederal Rules of Evidence are deemed to control. The terms “Constitution,” “constitutional,” and “unconstitutional”refer to the federal Constitution unless indicated to the contrary. You are also to assume that there is noapplicable community property law, no guest statute, and no No-Fault Insurance Act unless otherwisespecified. In negligence cases, if fault on the claimant’s part is or may be relevant, the statement of facts forthe particular question will identify the contributory or comparative negligence rule that is to be applied.

Page 53: Sample MBE II

-51-

103. John’s father, Jeremiah, died in Hospital.Hospital maintains a morgue with refrigerateddrawers a bit larger than a human body.Jeremiah’s body was placed in such a drawerawaiting pickup by a mortician. Before themortician called for the body, a Hospitalorderly placed two opaque plastic bags in thedrawer with Jeremiah’s body. One bagcontained Jeremiah’s personal effects, and theother contained an amputated leg from someother Hospital patient. It is stipulated thatHospital was negligent to allow the amputatedleg to get into Jeremiah’s drawer. Themortician delivered the two opaque plasticbags to John, assuming both containedpersonal effects. John was shocked when heopened the bag containing the amputated leg.John sued Hospital to recover for hisemotional distress. At the trial, John testifiedthat the experience had been extremelyupsetting, that he had had recurring nightmaresabout it, and that his family and businessrelationships had been adversely affected for aperiod of several months. He did not seekmedical or psychiatric treatment for hisemotional distress.

Who should prevail?

(A) John, because of the sensitivity peoplehave regarding the care of the bodies ofdeceased relatives.

(B) John, because hospitals are strictly liablefor mishandling dead bodies.

(C) Hospital, because John did not requiremedical or psychiatric treatment.

(D) Hospital, because John suffered nobodily harm.

104. Able was the owner of Greenacre, a large tractof land. Able entered into a binding writtencontract with Baker for the sale and purchaseof Greenacre for $125,000. The contractrequired Able to convey marketable recordtitle.

Baker decided to protect his interest andpromptly and properly recorded the contract.

Thereafter, but before the date scheduled forthe closing, Charlie obtained and properlyfiled a final judgment against Able in theamount of $1 million in a personal injury suit.A statute in the jurisdiction provides: “Anyjudgment properly filed shall, for ten yearsfrom filing, be a lien on the real property thenowned or subsequently acquired by any personagainst whom the judgment is rendered.”

The recording act of the jurisdiction authorizesrecording of contracts and also provides: “Noconveyance or mortgage of real property shallbe good against subsequent purchasers forvalue and without notice unless the same berecorded according to law.”

There are no other relevant statutoryprovisions.

At the closing, Baker declined to accept thetitle of Able on the ground that Charlie’sjudgment lien encumbered the title he wouldreceive and rendered it unmarketable. Ablebrought an appropriate action against Bakerfor specific performance of the contract andjoined Charlie as a party.

In this action, the judgment should be for

(A) Able, because in equity a purchaser takesfree of judgment liens.

(B) Able, because the contract had beenrecorded.

(C) Baker, because Able cannot benefit fromBaker’s action in recording the contract.

(D) Baker, because the statute creatingjudgment liens takes precedence over therecording act.

GO ON TO THE NEXT PAGE.

Page 54: Sample MBE II

-52-

105. Post sued Dint for dissolution of their year-long partnership. One issue concerned theamount of money Post had received in cash. It was customary for Dint to give Post moneyfrom the cash register as Post needed it forpersonal expenses. Post testified that, as hereceived money, he jotted down the amountsin the partnership ledger. Although Dint hadaccess to the ledger, he made no changes in it.The ledger was admitted into evidence. Dintseeks to testify to his memory of much largeramounts he had given Post.

Dint’s testimony is

(A) admissible, because it is based on Dint’sfirsthand knowledge.

(B) admissible, because the ledger entriesoffered by a party opponent opened thedoor.

(C) inadmissible, because the ledger is thebest evidence of the amounts Postreceived.

(D) inadmissible, because Dint’s failure tochallenge the accuracy of the ledgerconstituted an adoptive admission.

106. In a signed writing, Nimrod contracted topurchase a 25-foot travel trailer from Trailcofor $15,000, cash on delivery no later thanJune 1. Nimrod arrived at the Trailco sales loton Sunday, May 31, to pay for and takedelivery of the trailer, but refused to do sowhen he discovered that the spare tire wasmissing.

Trailco offered to install a spare tire onMonday when its service department wouldopen, but Nimrod replied that he did not wantthe trailer and would purchase another oneelsewhere.

Which of the following is accurate?

(A) Nimrod had a right to reject the trailer,but Trailco was entitled to a reasonableopportunity to cure the defect.

(B) Nimrod had a right to reject the trailerand terminate the contract under theperfect tender rule.

(C) Nimrod was required to accept thetrailer, because the defect could bereadily cured.

(D) Nimrod was required to accept thetrailer, because the defect did notsubstantially impair its value.

GO ON TO THE NEXT PAGE.

Page 55: Sample MBE II

-53-

107. Oker owned in fee simple two adjoining lots,Lots 1 and 2. He conveyed in fee simple Lot 1to Frank. The deed was in usual form of awarranty deed with the following provisioninserted in the appropriate place:

“Grantor, for himself, his heirs and assigns,does covenant and agree that any reasonableexpense incurred by grantee, his heirs andassigns, as the result of having to repair theretaining wall presently situated on Lot 1 atthe common boundary with Lot 2, shall bereimbursed one-half the costs of repairs; andby this provision the parties intend a covenantrunning with the land.”

Frank conveyed Lot 1 in fee simple to Sara bywarranty deed in usual and regular form. Thedeed omitted any reference to the retainingwall or any covenant. Fifty years after Oker’sconveyance to Frank, Sara conveyed Lot 1 infee simple to Tim by warranty deed in usualform; this deed omitted any reference to theretaining wall or the covenant.

There is no statute that applies to any aspect ofthe problems presented except a recording actand a statute providing for acquisition of titleafter ten years of adverse possession.

All conveyances by deeds were for aconsideration equal to fair market value.

The deed from Oker to Frank was neverrecorded. All other deeds were promptly andproperly recorded.

Lot 2 is now owned by Henry, who took byintestate succession from Oker, now dead.

Tim expended $3,500 on the retaining wall.Then he obtained all of the original deeds inthe chain from Oker to him. Shortly thereafter,Tim discovered the covenant in Oker’s deed toFrank. He demanded that Henry pay $1,750,and when Henry refused, Tim instituted anappropriate action to recover that sum fromHenry. In such action, Henry asserted alldefenses available to him.

If judgment is for Henry, it will be because

(A) Tim is barred by adverse possession.(B) Frank’s deed from Oker was never

recorded.(C) Tim did not know about the covenant

until after he had incurred the expensesand, hence, could not have relied on it.

(D) Tim’s expenditures were not proved tobe reasonable and customary.

108. While Prudence was leaving an elevator, itsuddenly dropped several inches, causing herto fall. An investigation of the accidentrevealed that the elevator dropped because ithad been negligently maintained by the AcmeElevator Company. Acme had a contract withthe owner of the building to inspect andmaintain the elevator. Prudence’s fall severelyaggravated a preexisting physical disability.

If Prudence sues Acme Elevator Company fordamages for her injuries, she should recover

(A) nothing, if Acme could not reasonablyhave been expected to foresee the extentof the harm that Prudence suffered as aresult of the accident.

(B) nothing, if the accident would not havecaused significant harm to an ordinarilyprudent elevator passenger.

(C) damages for the full amount of herdisability, because a tortfeasor must takeits victim as it finds her.

(D) damages for the injury caused by thefalling elevator, including theaggravation of her preexisting disability.

GO ON TO THE NEXT PAGE.

Page 56: Sample MBE II

-54-

109. Dix is on trial for killing Vetter. Theprosecutor calls Winn to testify that after beingshot, Vetter said, “Dix did it.” Before thetestimony is given, Dix’s lawyer asks for ahearing on whether Vetter believed his deathwas imminent when he made the statement.

Before permitting evidence of the dyingdeclaration, the judge should hear evidence onthe issue from

(A) both sides, with the jury not present, anddecide whether Winn may testify toVetter’s statement.

(B) both sides, with the jury present, anddecide whether Winn may testify toVetter’s statement.

(C) both sides, with the jury present, andallow the jury to determine whetherWinn may testify to Vetter’s statement.

(D) the prosecutor only, with the jury notpresent, and if the judge believes a jurycould reasonably find that Vetter knewhe was dying, permit Winn to testify tothe statement, with Dix allowed to offerevidence on the issue as a part of thedefendant’s case.

110. Police received an anonymous tip that Tusitalawas growing marijuana in her backyard, whichwas surrounded by a 15-foot high, solidwooden fence. Officer Boa was unable to viewthe yard from the street, so he used a policehelicopter to fly over Tusitala’s house. Boaidentified a large patch of marijuana plantsgrowing right next to the house and used thisobservation to obtain a search warrant.

Tusitala is prosecuted for possession ofmarijuana and moves to suppress use of themarijuana in evidence.

The court should

(A) grant the motion, because the onlypurpose of Boa’s flight was to observethe yard.

(B) grant the motion, because Tusitala had areasonable expectation of privacy in thecurtilage around her house and the policedid not have a warrant.

(C) deny the motion, because a warrant isnot required for a search of a residentialyard.

(D) deny the motion, because Tusitala had noreasonable expectation of privacy fromaerial observation.

GO ON TO THE NEXT PAGE.

Page 57: Sample MBE II

-55-

111. Buyem faxed the following signed message toZeller, his long-time widget supplier:“Urgently need blue widgets. Shipimmediately three gross at your current listprice of $600.” Upon receipt of the fax, Zellershipped three gross of red widgets to Buyem,and faxed to Buyem the following message:“Temporarily out of blue. In case red willhelp, am shipping three gross at the sameprice. Hope you can use them.”

Upon Buyem’s timely receipt of both theshipment and Zeller’s fax, which of thefollowing best describes the rights and dutiesof Buyem and Zeller?

(A) Buyem may accept the shipment, inwhich case he must pay Zeller the listprice, or he must reject the shipment andrecover from Zeller for total breach ofcontract.

(B) Buyem may accept the shipment, inwhich case he must pay Zeller the listprice, or he may reject the shipment, inwhich case he has no further rightsagainst Zeller.

(C) Buyem may accept the shipment, inwhich case he must pay Zeller the listprice, less any damages sustainedbecause of the nonconforming shipment,or he may reject the shipment andrecover from Zeller for total breach ofcontract, subject to Zeller’s right to cure.

(D) Buyem may accept the shipment, inwhich case he must pay Zeller the listprice, less any damages sustainedbecause of the nonconforming shipment,or he may reject the shipment providedthat he promptly covers by obtainingconforming widgets from anothersupplier.

112. Members of a religious group calling itself theFriends of Lucifer believe in Lucifer as theirSupreme Being. The members of this groupmeet once a year on top of Mt. Snow, locatedin a U.S. National Park, to hold an overnightencampment and a midnight dance around alarge campfire. They believe this overnightencampment and all of its rituals are requiredby Lucifer to be held on the top of Mt. Snow.U.S. National Park Service rules that havebeen consistently enforced prohibit allovernight camping and all campfires on Mt.Snow because of the very great dangersovernight camping and campfires would posein that particular location. As a result, the parkSuperintendent denied a request by the Friendsof Lucifer for a permit to conduct theseactivities on top of Mt. Snow. The parkSuperintendent, who was known to beviolently opposed to cults and otherunconventional groups had, in the past, issuedpermits to conventional religious groups toconduct sunrise services in other areas of thatU.S. National Park.

The Friends of Lucifer brought suit in FederalCourt against the U.S. National Park Serviceand the Superintendent of the park to compelissuance of the requested permit.

As a matter of constitutional law, the mostappropriate result in this suit would be adecision that denial of the permit was

(A) invalid, because the free exercise clauseof the First Amendment prohibits thePark Service from knowingly interferingwith religious conduct.

(B) invalid, because these facts demonstratethat the action of the Park Servicepurposefully and invidiouslydiscriminated against the Friends ofLucifer.

(C) valid, because the establishment clauseof the First Amendment prohibits theholding of religious ceremonies onfederal land.

(D) valid, because religiously motivatedconduct may be subjected tonondiscriminatory time, place, andmanner restrictions that advanceimportant public interests.

GO ON TO THE NEXT PAGE.

Page 58: Sample MBE II

-56-

113. Park sued Davis Co. for injuries suffered inthe crash of Park’s dune buggy, allegedlycaused by a defective auto part manufacturedby Davis Co. Davis Co. claims that the partwas a fraudulent imitation, not produced byDavis Co.

Which of the following is NOT admissible onthe issue of whether the part wasmanufactured by Davis Co.?

(A) The fact that the defective part bearsDavis Co.’s insignia or trademark.

(B) Testimony that the part was purchasedfrom a parts house to which Davis Co.regularly sold parts.

(C) The part itself and a concededly genuinepart manufactured by Davis Co. (for thejury’s comparison).

(D) A judgment for another plaintiff againstDavis Co. in another case involvingsubstantially similar facts.

114. Anna entered into a valid written contract topurchase Blackacre, a large tract of land, fromJones for its fair market value of $50,000. Thecontract was assignable by Anna. Anna dulynotified Jones to convey title to Anna andCharles, Charles being Anna’s friend whomAnna had not seen for many years.

When Anna learned that Charles would haveto sign certain documents in connection withthe closing, she prevailed upon her brother,Donald, to attend the closing and pretend to beCharles. Anna and Donald attended theclosing, and Jones executed an instrument inthe proper form of a deed, purporting toconvey Blackacre to Anna and Charles, astenants in common. Donald pretended that hewas Charles, and he signed Charles’s name toall the required documents. Anna provided theentire $50,000 consideration for thetransaction. The deed was promptly andproperly recorded.

Unknown to Anna or Donald, Charles haddied several months before the closing.Charles’s will, which was duly probated,devised “All my real estate to my nephew,Nelson” and the residue of his estate to Anna.

Anna and Nelson have been unable to agree asto the status or disposition of Blackacre.Nelson brought an appropriate action againstJones and Anna to quiet legal title to anundivided one-half interest in Blackacre.

The court should hold that legal title toBlackacre is vested

(A) all in Jones.(B) all in Anna.(C) one-half in Anna and one-half in Jones.(D) one-half in Anna and one-half in Nelson.

GO ON TO THE NEXT PAGE.

Page 59: Sample MBE II

-57-

Questions 115-116 are based on the following fact situation.

Staff, Inc., a flour wholesaler, contracted to deliverto Eclaire, a producer of fine baked goods, her flourrequirements for a one-year period. Before deliveryof the first scheduled installment, Staff sold itsbusiness and “assigned” all of its sale contracts toMiller, Inc., another reputable and long-time flourwholesaler. Staff informed Eclaire of this transaction.

115. For this question only, assume that whenMiller tendered the first installment to Eclairein compliance with the Staff-Eclaire contract,Eclaire refused to accept the goods.

Which of the following arguments, if any,legally support(s) Eclaire’s rejection of thegoods?

I. Executory requirements contracts arenonassignable.

II. Duties under an executory bilateralcontract are assumable only by anexpress promise to perform on the part ofthe delegatee.

III. Language of “assignment” in the transferfor value of a bilateral sale-of-goodscontract affects only a transfer of rights,not a delegation of duties.

(A) I only.(B) II and III only.(C) I and II and III.(D) Neither I nor II nor III.

116. For this question only, assume that Eclaireaccepted Miller’s delivery of the firstinstallment under the Staff-Eclaire contract,but that Eclaire paid the contract price for thatinstallment to Staff and refused to payanything to Miller.

In an action by Miller against Eclaire for thecontractual amount of the first installment,which of the following, if any, will be aneffective defense for Eclaire?

I. Eclaire had not expressly agreed toaccept Miller as her flour supplier.

II. Eclaire’s payment of the contractualinstallment to Staff discharged herobligation.

III. Staff remained obligated to Eclaire eventhough Staff had assigned the contract toMiller.

(A) I only.(B) II only.(C) I and III only.(D) Neither I nor II nor III.

GO ON TO THE NEXT PAGE.

Page 60: Sample MBE II

-58-

117. On October 22, Officer Jones submitted anapplication for a warrant to search 217 ElmStreet for cocaine. In the application, OfficerJones stated under oath that he believed therewas cocaine at that location because ofinformation supplied to him on the morning ofOctober 22 by Susie Schultz. He describedSchultz as a cocaine user who had previouslysupplied accurate information concerning theuse of cocaine in the community andsummarized what Schultz had told him asfollows: the previous night, October 21,Schultz was in Robert Redd’s house at 217Elm Street. Redd gave her cocaine. She alsosaw three cellophane bags containing cocainein his bedroom.

The warrant was issued and a search of 217Elm Street was conducted on October 22. Thesearch turned up a quantity of marijuana butno cocaine. Robert Redd was arrested andcharged with possession of marijuana. Reddmoved to suppress the use of the marijuana asevidence contending that Susie Schultz wasnot in 217 Elm Street on October 21 or at anyother time.

If, after hearing evidence, the judge concludesthat the statement in the application attributedto Susie Schultz is incorrect, the judge shouldgrant the motion to suppress

(A) because the application contains amaterial statement that is false.

(B) because of the false statement andbecause no cocaine was found in thehouse.

(C) only if he also finds that Susie Schultz’sstatement was a deliberate lie.

(D) only if he also finds that Officer Jonesknew the statement was false.

118. The Personnel Handbook of Green Citycontains all of that city’s personnel policies.One section of the handbook states that“where feasible and practicable supervisors areencouraged to follow the procedures specifiedin this Handbook before discharging a cityemployee.” Those specified proceduresinclude a communication to the employee ofthe reasons for the contemplated discharge andan opportunity for a pretermination trial-typehearing at which the employee may challengethose reasons. After a year of service, Baker,the secretary to the Green City Council, wasdischarged without receiving anycommunication of reasons for hercontemplated discharge and without receivingan opportunity for a pretermination trial-typehearing. Baker files suit in federal districtcourt to challenge her discharge solely onconstitutional grounds.

Which of the following best describes theinitial burden of persuasion in that suit?

(A) The Green City Council mustdemonstrate that its personnel handbookcreated no constitutionally protectedinterest in city employment or in theprocedures by which such employment isterminated.

(B) The Green City Council mustdemonstrate that Baker’s termination wasfor good cause.

(C) Baker must demonstrate that state lawcreates a constitutionally protectedinterest in her employment or in theprocedures by which her employment isterminated.

(D) Baker must demonstrate that shereasonably believed that she could workfor Green City for as long as she wished.

GO ON TO THE NEXT PAGE.

Page 61: Sample MBE II

-59-

119. Dean was prosecuted in federal court formaking threats against the President of theUnited States. Dean was a voluntary patient ina private psychiatric hospital and told a nurse,shortly before the President came to town, thatDean planned to shoot the President. Thenurse reported the threat to FBI agents.

Dean’s motion to prevent the nurse fromtestifying is likely to be

(A) successful, because the statement wasmade in a medical setting.

(B) successful, because the nurse violated aconfidence in reporting the statement.

(C) unsuccessful, because the statement wasnot within any privilege.

(D) unsuccessful, because Dean had not beencommitted involuntarily by court order.

120. Able, who owned Blackacre, a residential lotimproved with a dwelling, conveyed it for avaluable consideration to Baker. The dwellinghad been constructed by a prior owner. Bakerhad inspected Blackacre prior to the purchaseand discovered no defects. After moving in,Baker became aware that sewage seeped intothe basement when the toilets were flushed.Able said that this defect had been present foryears and that he had taken no steps to hidethe facts from Baker. Baker paid for thenecessary repairs and brought an appropriateaction against Able to recover his cost ofrepair.

If Baker wins, it will be because

(A) Able failed to disclose a latent defect.(B) Baker made a proper inspection.(C) the situation constitutes a health hazard.(D) Able breached the implied warranty of

habitability and fitness for purpose.

GO ON TO THE NEXT PAGE.

Page 62: Sample MBE II

-60-

121. Gardner’s backyard, which is landscaped withexpensive flowers and shrubs, is adjacent to agolf course. While Driver was playing golf onthe course, a thunderstorm suddenly came up.As Driver was returning to the clubhouse inhis golf cart, lightning struck a tree on thecourse, and the tree began to fall in Driver’sdirection. In order to avoid being hit by thetree, Driver deliberately steered his cart ontoGardner’s property, causing substantialdamage to Gardner’s expensive plantings.

In an action by Gardner against Driver torecover damages for the harm to his plantings,Gardner will

(A) prevail, because, although occasioned bynecessity, Driver’s entry onto Gardner’sproperty was for Driver’s benefit.

(B) prevail, for nominal damages only,because Driver was privileged to enterGardner’s property.

(C) not prevail, because the lightning was anact of God.

(D) not prevail, because Driver’s entry ontoGardner’s property was occasioned bynecessity and therefore privileged.

122. Steve, in desperate need of money, decided tohold up a local convenience store. Determinednot to harm anyone, he carried a toy gun thatresembled a real gun. In the store, he pointedthe toy gun at the clerk and demanded money.A customer who entered the store and saw therobbery in progress pulled his own gun andfired at Steve. The bullet missed Steve butstruck and killed the clerk.

Steve was charged with felony murder.

His best argument for being found NOT guiltyis that he

(A) did not intend to kill.(B) did not commit the robbery because he

never acquired any money from theclerk.

(C) did not intend to create any risk of harm.(D) is not responsible for the acts of the

customer.

GO ON TO THE NEXT PAGE.

Page 63: Sample MBE II

-61-

123. Client consulted Lawyer about handling thesale of Client’s building, and asked Lawyerwhat her legal fee would be. Lawyer repliedthat her usual charge was $100 per hour, andestimated that the legal work on behalf ofClient would cost about $5,000 at that rate.Client said, “Okay; let’s proceed with it,” andLawyer timely and successfully completed thework. Because of unexpected title problems,Lawyer reasonably spent 75 hours on thematter and shortly thereafter mailed Client abill for $7,500, with a letter itemizing the workperformed and time spent. Client responded bya letter expressing his good-faith belief thatLawyer had agreed to a total fee of no morethan $5,000. Client enclosed a check in theamount of $5,000 payable to Lawyer andconspicuously marked, “Payment in full forlegal services in connection with the sale ofClient’s building.” Despite reading the“Payment in full...” language, Lawyer, withoutany notation of protest or reservation of rights,endorsed and deposited the check to her bankaccount. The check was duly paid by Client’sbank. A few days later, Lawyer unsuccessfullydemanded payment from Client of the $2,500difference between the amount of her bill andthe check, and now sues Client for thatdifference.

What, if anything, can Lawyer recover fromClient?

(A) Nothing, because the risk of unexpectedtitle problems in a real-propertytransaction is properly allocable to theseller’s attorney and thus to Lawyer inthis case.

(B) Nothing, because the amount ofLawyer’s fee was disputed in good faithby Client, and Lawyer impliedly agreedto an accord and satisfaction.

(C) $2,500, because Client agreed to anhourly rate for as many hours as thework reasonably required, and the sum of$5,000 was merely an estimate.

(D) The reasonable value of Lawyer’sservices in excess of $5,000, if any,because there was no specific agreementon the total amount of Lawyer’s fee.

124. Pauline and Doris own adjacent parcels ofland. On each of their parcels was a low-riseoffice building. The two office buildings wereof the same height.

Last year Doris decided to demolish the low-rise office building on her parcel and to erect anew high-rise office building of substantiallygreater height on the parcel as permitted by thezoning and building ordinances. She securedall the governmental approvals necessary topursue her project.

As Doris’s new building was in the course ofconstruction, Pauline realized that the shadowsit would create would place her (Pauline’s)building in such deep shade that the rent shecould charge for space in her building wouldbe substantially reduced.

Pauline brought an appropriate action againstDoris to enjoin the construction in order toeliminate the shadow problem and fordamages. Pauline presented uncontrovertedevidence that her evaluation as to the impactof the shadow on the fair rental value of herbuilding was correct. There is no statute orordinance (other than the building and zoningordinances) that is applicable to the issuesbefore the court.

The court should

(A) grant to Pauline the requested injunction.(B) award Pauline damages measured by the

loss of rental value, but not aninjunction.

(C) grant judgment for Doris, because shehad secured all the necessarygovernmental approvals for the newbuilding.

(D) grant judgment for Doris, becausePauline has no legal right to havesunshine continue to reach the windowsof her building.

GO ON TO THE NEXT PAGE.

Page 64: Sample MBE II

-62-

Questions 125-126 are based on the following fact situation.

Dumont, a real estate developer, was trying topurchase land on which he intended to build a largecommercial development. Perkins, an elderlywidow, had rejected all of Dumont’s offers to buyher ancestral home, where she had lived all her lifeand which was located in the middle of Dumont’splanned development. Finally, Dumont offered her$250,000. He told her that it was his last offer andthat if she rejected it, state law authorized him tohave her property condemned.

Perkins then consulted her nephew, a law student,who researched the question and advised her thatDumont had no power of condemnation under statelaw. Perkins had been badly frightened by Dumont’sthreat, and was outraged when she learned thatDumont had lied to her.

125. If Perkins sues Dumont for damages foremotional distress, will she prevail?

(A) Yes, if Dumont’s action was extreme andoutrageous.

(B) Yes, because Perkins was frightened andoutraged.

(C) No, if Perkins did not suffer emotionaldistress that was severe.

(D) No, if it was not Dumont’s purpose tocause emotional distress.

126. If Perkins asserts a claim based onmisrepresentation against Dumont, will sheprevail?

(A) Yes, if Dumont knew he had no legalpower of condemnation.

(B) Yes, if Dumont tried to take unfairadvantage of a gross difference betweenhimself and Perkins in commercialknowledge and experience.

(C) No, if Dumont’s offer of $250,000equaled or exceeded the market value ofPerkins’s property.

(D) No, because Perkins suffered nopecuniary loss.

127. State Y employs the Model Penal Code orAmerican Law Institute test for insanity, andrequires the state to prove sanity, when it is inissue, beyond a reasonable doubt. At Askew’strial for murder, he pleaded insanity. The stateput on an expert psychiatrist who hadexamined Askew. He testified that, in hisopinion, Askew was sane at the time of themurder. Askew’s attorney did not introduceexpert testimony on the question of sanity.Rather, he presented lay witnesses whotestified that, in their opinion, Askew wasinsane at the time of the murder. At the end ofthe trial, each side moves for a directed verdicton the question of sanity.

Which of the following correctly describes thejudge’s situation?

(A) She may grant a directed verdict for thedefense if she believes that the jury couldnot find the prosecution to have provedsanity beyond a reasonable doubt.

(B) She may grant a directed verdict for theprosecution if she believes that Askew’switnesses on the insanity question are notbelievable.

(C) She may not grant a directed verdict forthe defense, because the state had experttestimony and the defense only laywitnesses.

(D) She may grant a directed verdict for theprosecution if she is convinced by theirexperts that Askew was sane beyond areasonable doubt.

GO ON TO THE NEXT PAGE.

Page 65: Sample MBE II

-63-

128. Trelawney worked at a day-care center run bythe Happy Faces Day Care Corporation. At thecenter, one of the young charges, Smith, oftenarrived with bruises and welts on his back andlegs. A statute in the jurisdiction requires allday-care workers to report to the police caseswhere there is probable cause to suspect childabuse and provides for immediate removalfrom the home of any suspected child abusevictims. Trelawney was not aware of thisstatute. Nevertheless, he did report Smith’scondition to his supervisor, who advised himto keep quiet about it so the day-care centerwould not get into trouble for defaming aparent. About two weeks after Trelawney firstnoticed Smith’s condition, Smith was beatento death by his father. Trelawney has beencharged with murder in the death of Smith.The evidence at trial disclosed, in addition tothe above, that the child had been the victim ofbeatings by the father for some time, and thatthese earlier beatings had been responsible forthe marks that Trelawney had seen. Smith’smother had been aware of the beatings but hadnot stopped them because she was herselfafraid of Smith’s father.

Trelawney’s best argument that he is NOTguilty of murder is

(A) he was not aware of the duty-to-reportstatute.

(B) he lacked the mental state necessary tothe commission of the crime.

(C) his omission was not the proximatecause of death.

(D) the day-care corporation, rather thanTrelawney, was guilty of the omission,which was sanctioned by its supervisory-level agent.

Questions 129-130 are based on the following fact situation.

Perkins and Morton were passengers sitting inadjoining seats on a flight on Delval Airline. Therewere many empty seats on the aircraft.

During the flight, a flight attendant served Mortonnine drinks. As Morton became more and moreobviously intoxicated and attempted to engagePerkins in a conversation, Perkins chose to ignoreMorton. This angered Morton, who suddenly struckPerkins in the face, giving her a black eye.

129. If Perkins asserts a claim for damages againstDelval Airline based on negligence, Perkinswill

(A) not recover, because a person is notrequired by law to come to the assistanceof another who is imperiled by a thirdparty.

(B) not recover, if Perkins could easily havemoved to another seat.

(C) recover, because a common carrier isstrictly liable for injuries suffered by apassenger while aboard the carrier.

(D) recover, if the flight attendants shouldhave perceived Morton’s condition andacted to protect Perkins before the blowwas struck.

130. If Perkins asserts a claim for damages againstDelval Airline based on battery, she will

(A) prevail, because she suffered anintentionally inflicted harmful oroffensive contact.

(B) prevail, if the flight attendant actedrecklessly in continuing to serve liquorto Morton.

(C) not prevail, because Morton was notacting as an agent or employee of DelvalAirline.

(D) not prevail, unless she can establishsome permanent injury from the contact.

GO ON TO THE NEXT PAGE.

Page 66: Sample MBE II

-64-

131. Terrorists in the foreign country of Ruritaniakidnapped the United States ambassador tothat country. They threatened to kill her unlessthe President of the United States secured therelease of an identified person who was acitizen of Ruritania and was held in a prison ofthe state of Aurora in the United Statespursuant to a valid conviction by that state.

The President responded by entering into anagreement with Ruritania which provided thatRuritania would secure the release of theUnited States ambassador on a specified datein return for action by the President that wouldsecure the release of the identified person heldin the Aurora prison. The President thenordered the governor of Aurora to release theprisoner in question. The governor refused. Nofederal statutes are applicable.

Which of the following is the strongestconstitutional argument for the authority of thePresident to take action in these circumstancesrequiring the governor of Aurora to release theAurora prisoner?

(A) The power of the President to conductthe foreign affairs of the United Statesincludes a plenary authority to takewhatever action the President deemswise to protect the safety of ourdiplomatic agents.

(B) The power of the President to appointambassadors authorizes him to take anyaction that he may think desirable toprotect them from injury because, uponappointment, those officials becomeagents of the President.

(C) The power of the President to negotiatewith foreign nations impliedly authorizesthe President to make executiveagreements with them which prevail overstate law.

(D) The duty of the President to executefaithfully the laws authorizes him toresolve finally any conflicts betweenstate and federal interests, making thedetermination of such matters whollynonjusticiable.

132. Damson was charged with murder, andWagner testified for the prosecution. On cross-examination of Wagner, Damson seeks toelicit an admission that Wagner was alsocharged with the same murder and that theprosecutor told her, “If you testify againstDamson, we will drop the charges against youafter the conclusion of Damson’s trial.”

The evidence about the prosecutor’s promise is

(A) admissible, as proper impeachment ofWagner.

(B) admissible, as an admission by an agentof a party-opponent.

(C) inadmissible, because the lawencourages plea-bargaining.

(D) inadmissible, because the evidence ishearsay not within any exception.

GO ON TO THE NEXT PAGE.

Page 67: Sample MBE II

-65-

Questions 133-134 are based on the following fact situation.

On November 1, Debbit, an accountant, andBarrister, a lawyer, contracted for the sale by Debbitto Barrister of the law books Debbit had inheritedfrom his father. Barrister agreed to pay the purchaseprice of $10,000 when Debbit delivered the bookson December 1.

On November 10, Barrister received a signed letterfrom Debbit that stated: “I have decided to disposeof the book stacks containing the law books youhave already purchased. If you want the stacks, Iwill deliver them to you along with the books onDecember 1 at no additional cost to you. Let meknow before November 15 whether you want them.I will not sell them to anyone else before then.” OnNovember 14, Barrister faxed and Debbit receivedthe following message: “I accept your offer of thestacks.” Debbit was not a merchant with respect toeither law books or book stacks.

133. Debbit is contractually obligated to deliver thestacks because

(A) Barrister provided a new bargained-forexchange by agreeing to take the stacks.

(B) Debbit’s letter (received by Barrister onNovember 10) and Barrister’s fax-message of November 14 constituted aneffective modification of the originalsale-of-books contract.

(C) Barrister’s fax-message of November 14operated to rescind unilaterally theoriginal sale-of-books contract.

(D) Debbit’s letter (received by Barrister onNovember 10) waived the bargained-forconsideration that would otherwise berequired.

134. For this question only, assume that onNovember 12 Debbit told Barrister that he haddecided not to part with the stacks.

Will this communication operate as a legallyeffective revocation of his offer to deliver thestacks?

(A) Yes, because Barrister had a pre-existingobligation to pay $10,000 for the lawbooks.

(B) Yes, because Debbit was not a merchantwith respect to book stacks.

(C) No, because Debbit had given a signedassurance that the offer would be heldopen until November 15.

(D) No, because by delaying his acceptanceuntil November 14, Barristerdetrimentally relied on Debbit’s promisenot to sell the stacks to anyone else inthe meantime.

GO ON TO THE NEXT PAGE.

Page 68: Sample MBE II

-66-

135. Seller owned Blackacre, improved with anaging four-story warehouse. The warehousewas built to the lot lines on all four sides. Onthe street side, recessed loading dockspermitted semi-trailers to be backed in. Afterthe tractors were unhooked, the trailersextended into the street and occupied most ofone lane of the street. Over the years, astrailers became larger, the blocking of thestreet became more severe. The municipalityadvised Seller that the loading docks could notcontinue to be used because the trailersblocked the street; it gave Seller 90 days tocease and desist.

During the 90 days, Seller sold and conveyedBlackacre by warranty deed for a substantialconsideration to Buyer. The problem of theloading docks was not discussed in thenegotiations.

Upon expiration of the 90 days, themunicipality required Buyer to stop using theloading docks. This action substantiallyreduced the value of Blackacre.

Buyer brought an appropriate action againstSeller seeking cancellation of the deed andreturn of all monies paid.

Such action should be based upon a claim of

(A) misrepresentation.(B) breach of the covenant of warranty.(C) failure of consideration.(D) mutual mistake.

136. Prescott sued Doxie for fraud. After verdict forPrescott, Doxie talked with juror Wall aboutthe trial.

Doxie’s motion for a new trial would be mostlikely granted if Wall is willing to testify thathe voted for Prescott because he

(A) misunderstood the judge’s instructionsconcerning the standard of proof in afraud case.

(B) was feeling ill and needed to get homequickly.

(C) relied on testimony that the judge hadstricken and ordered the jury todisregard.

(D) learned from a court clerk that Doxie hadbeen accused of fraud in several recentlawsuits.

137. Despondent over losing his job, Wilmontdrank all night at a bar. While driving home,he noticed a car following him and, in hisintoxicated state, concluded he was beingfollowed by robbers. In fact, a police car wasfollowing him on suspicion of drunk driving.In his effort to get away, Wilmont spedthrough a stop sign and struck and killed apedestrian. He was arrested by the police.

Wilmont is prosecuted for manslaughter.

He should be

(A) acquitted, because he honestly believedhe faced an imminent threat of death orsevere bodily injury.

(B) acquitted, because his intoxicationprevented him from appreciating the riskhe created.

(C) convicted, because he acted recklesslyand in fact was in no danger.

(D) convicted, because he acted recklesslyand his apprehension of danger was notreasonable.

GO ON TO THE NEXT PAGE.

Page 69: Sample MBE II

-67-

138. Ody, owner of Profitacre, executed aninstrument in the proper form of a deed,purporting to convey Profitacre “to Leon forlife, then to Ralph in fee simple.” Leon, who isOdy’s brother and Ralph’s father, promptlybegan to manage Profitacre, which is valuableincome-producing real estate. Leon collectedall rents and paid all expenses, including realestate taxes. Ralph did not object, and thisstate of affairs continued for five years until1987. In that year, Leon executed aninstrument in the proper form of a deed,purporting to convey Profitacre to Mona.Ralph, no admirer of Mona, asserted his rightto ownership of Profitacre. Mona asserted herownership and said that if Ralph had anyrights he was obligated to pay real estate taxes,even though Leon had been kind enough topay them in the past. Income from Profitacreis ample to cover expenses, including realestate taxes.

In an appropriate action to determine the rightsof the parties, the court should decide

(A) Leon’s purported deed forfeited his lifeestate, so Ralph owns Profitacre in feesimple.

(B) Mona owns an estate for her life, isentitled to all income, and must pay realestate taxes; Ralph owns the remainderinterest.

(C) Mona owns an estate for the life of Leon,is entitled to all income, and must payreal estate taxes; Ralph owns theremainder interest.

(D) Mona owns an estate for the life of Leonand is entitled to all income; Ralph ownsthe remainder interest, and must pay realestate taxes.

139. Homer and Ethel were jointly in possession ofGreenacre in fee simple as tenants in common.They joined in a mortgage of Greenacre toFortunoff Bank. Homer erected a fence alongwhat he considered to be the true boundarybetween Greenacre and the adjoining property,owned by Mitchell. Shortly thereafter, Homerhad an argument with Ethel and gave up hispossession to Greenacre. The debt secured bythe mortgage had not been paid.

Mitchell surveyed his land and found that thefence erected a year earlier by Homer did notfollow the true boundary. Part of the fence waswithin Greenacre. Part of the fence encroachedon Mitchell’s land. Mitchell and Ethelexecuted an agreement fixing the boundaryline in accordance with the fence constructedby Homer. The agreement, which met all theformalities required in the jurisdiction, waspromptly and properly recorded.

A year after the agreement was recorded,Homer temporarily reconciled his differenceswith Ethel and resumed joint possession ofGreenacre. Thereafter, Homer repudiated theboundary line agreement and brought anappropriate action against Mitchell and Ethelto quiet title along the original true boundary.

In such action, Homer will

(A) win, because Fortunoff Bank was not aparty to the agreement.

(B) win, because one tenant in commoncannot bind another tenant in common toa boundary line agreement.

(C) lose, because the agreement, as a matterof law, was mutually beneficial to Etheland Homer.

(D) lose, because Ethel was in solepossession of said premises at the timethe agreement was signed.

GO ON TO THE NEXT PAGE.

Page 70: Sample MBE II

-68-

140. At the trial of an action against Grandmotheron behalf of Patrick, the following evidencehas been introduced. Grandson and his friend,Patrick, both aged eight, were visiting atGrandmother’s house when, while exploringthe premises, they discovered a hunting rifle inan unlocked gun cabinet. They removed itfrom the cabinet and were examining it whenthe rifle, while in Grandson’s hands, somehowdischarged. The bullet struck and injuredPatrick. The gun cabinet was normally locked.Grandmother had opened it for dusting severaldays before the boys’ visit, and had thenforgotten to relock it. She was not aware thatit was unlocked when the boys arrived.

If the defendant moves for a directed verdictin her favor at the end of the plaintiff’s case,that motion should be

(A) granted, because Grandmother is notlegally responsible for the acts ofGrandson.

(B) granted, because Grandmother did notrecall that the gun cabinet was unlocked.

(C) denied, because a firearm is aninherently dangerousinstrumentality.

(D) denied, because a jury could find thatGrandmother breached a duty of care sheowed to Patrick.

Questions 141-142 are based on the following fact situation.

On November 15, Joiner in a signed writingcontracted with Galley for an agreed price topersonally remodel Galley’s kitchen according tospecifications provided by Galley, and to start workon December 1. Joiner agreed to provide allmaterials for the job in addition to all of the laborrequired.

141. For this question only, assume that onNovember 26 Joiner without legal excuserepudiated the contract and that Galley, after areasonable and prolonged effort, could notfind anyone to remodel his kitchen for a priceapproximating the price agreed to by Joiner.

If one year later Galley brings an action forspecific performance against Joiner, which ofthe following will provide Joiner with the bestdefense?

(A) An action for equitable relief not broughtwithin a reasonable time is barred bylaches.

(B) Specific performance is generally notavailable as a remedy to enforce acontractual duty to perform personalservices.

(C) Specific performance is generally notavailable as a remedy in the case of ananticipatory repudiation.

(D) Specific performance is not available asa remedy where even nominal damagescould have been recovered as a remedyat law.

142. For this question only, assume the followingfacts. On November 26, Galley without legalexcuse repudiated the contract.Notwithstanding Galley’s repudiation,however, Joiner subsequently purchased for$5,000 materials that could only be used inremodeling Galley’s kitchen, and promptlynotified Galley, “I will hold you to ourcontract.” If allowed to perform, Joiner wouldhave made a profit of $3,000 on the job.

If Galley refuses to retract his repudiation, andJoiner sues him for damages, what is themaximum that Joiner is entitled to recover?

(A) Nothing, because he failed to mitigatehis damages.

(B) $3,000, his expectancy damages.(C) $5,000, on a restitutionary theory.(D) $5,000, his reliance damages, plus

$3,000, his expectancy damages.

GO ON TO THE NEXT PAGE.

Page 71: Sample MBE II

-69-

Questions 143-144 are based on the following fact situation.

The police suspected that Yancey, a 16-year-oldhigh school student, had committed a series ofburglaries. Two officers went to Yancey’s highschool and asked the principal to call Yancey out ofclass and to search his backpack. While the officerswaited, the principal took Yancey into the hallwhere she asked to look in his backpack. WhenYancey refused, the principal grabbed it from him,injuring Yancey’s shoulder in the process. In thebackpack, she found jewelry that she turned over tothe officers.

The officers believed that the jewelry had beentaken in one of the burglaries. They arrested Yancey,took him to the station, and gave him Mirandawarnings. Yancey asked to see a lawyer. The policecalled Yancey’s parents to the station. WhenYancey’s parents arrived, the police asked them tospeak with Yancey. They put them in a room andsecretly recorded their conversation with aconcealed electronic device. Yancey broke downand confessed to his parents that he had committedthe burglaries.

Yancey was charged with the burglaries.

143. Yancey moves to suppress the use of thejewelry.

The court should

(A) deny the motion on the ground that thesearch was incident to a lawful arrest.

(B) deny the motion on the ground thatschool searches are reasonable ifconducted by school personnel on schoolgrounds on the basis of reasonablesuspicion.

(C) grant the motion on the ground that thesearch was conducted with excessiveforce.

(D) grant the motion on the ground that thesearch was conducted without probablecause or a warrant.

144. Assume for this question only that the courtdenied the motion to suppress the jewelry.Yancey moves to suppress the use of thestatement Yancey made to his parents.

The best argument for excluding it would bethat

(A) Yancey was in custody at the time thestatement was recorded.

(B) the police did not comply with Yancey’srequest for a lawyer.

(C) once Yancey had invoked his right tocounsel, it was improper for the police tolisten to any of his private conversations.

(D) the meeting between Yancey and hisparents was arranged by the police toobtain an incriminating statement.

GO ON TO THE NEXT PAGE.

Page 72: Sample MBE II

-70-

145. A newly enacted federal statute appropriates$100 million in federal funds to support basicresearch by universities located in the UnitedStates. The statute provides that “the ten bestuniversities in the United States” will eachreceive $10 million. It also provides that “theten best universities” shall be “determined bya poll of the presidents of all the universitiesin the nation, to be conducted by the UnitedStates Department of Education.” Inresponding to that poll, each universitypresident is required to apply the well-recognized and generally accepted standardsof academic quality that are specified in thestatute. The provisions of the statute areinseverable.

Which of the following statements about thisstatute is correct?

(A) The statute is unconstitutional, becausethe reliance by Congress on a poll ofindividuals who are not federal officialsto determine the recipients of itsappropriated funds is an unconstitutionaldelegation of legislative power.

(B) The statute is unconstitutional, becausethe limitation on recipients to the ten bestuniversities is arbitrary and capriciousand denies other high quality universitiesthe equal protection of the laws.

(C) The statute is constitutional, becauseCongress has plenary authority todetermine the objects of its spending andthe methods used to achieve them, solong as they may reasonably be deemedto serve the general welfare and do notviolate any prohibitory language in theConstitution.

(D) The validity of the statute isnonjusticiable, because the use byCongress of its spending powernecessarily involves politicalconsiderations that must be resolvedfinally by those branches of thegovernment that are closest to thepolitical process.

146. Which of the following fact patterns mostclearly suggests an implied-in-fact contract?

(A) A county tax assessor mistakenly billsAlgernon for taxes on Bathsheba’sproperty, which Algernon, in good faith,pays.

(B) Meddick, a physician, treated Ryderwithout Ryder’s knowledge or consent,while Ryder was unconscious as theresult of a fall from his horse.

(C) Asphalt, thinking that he was pavingCustomer’s driveway, for which Asphalthad an express contract, mistakenlypaved Nabor’s driveway while Naborlooked on without saying anything orraising any objection.

(D) At her mother’s request, Iris, anaccountant, filled out and filed hermother’s “E-Z” income-tax form (asimple, short form).

GO ON TO THE NEXT PAGE.

Page 73: Sample MBE II

-71-

147. Ashton owned Woodsedge, a tract used forcommercial purposes, in fee simple andthereafter mortgaged it to First Bank. Shesigned a promissory note secured by a dulyexecuted and recorded mortgage. There wasno “due on sale” clause, that is, no provisionthat, upon sale, the whole balance then owingwould become due and owing. Ashtonconveyed Woodsedge to Beam “subject to amortgage to First Bank, which the granteeassumes and agrees to pay.” Beam conveyedWoodsedge to Carter “subject to an existingmortgage to First Bank.” A copy of the noteand the mortgage that secured it had beenexhibited to each grantee.

After Carter made three timely payments, nofurther payments were made by any party. Infact, the real estate had depreciated to a pointwhere it was worth less than the debt.

There is no applicable statute or regulation.

In an appropriate foreclosure action, FirstBank joined Ashton, Beam, and Carter asdefendants. At the foreclosure sale, althoughthe fair market value for Woodsedge in itsdepreciated state was obtained, a deficiencyresulted.

First Bank is entitled to collect a deficiencyjudgment against

(A) Ashton only.(B) Ashton and Beam only.(C) Beam and Carter only.(D) Ashton, Beam, and Carter.

148. Landco purchased a large tract of landintending to construct residential housing onit. Landco hired Poolco to build a large in-ground swimming pool on the tract. Thecontract provided that Poolco would carry outblasting operations that were necessary tocreate an excavation large enough for the pool.The blasting caused cracks to form in thewalls of Plaintiff’s home in a nearbyresidential neighborhood.

In Plaintiff’s action for damages againstLandco, Plaintiff should

(A) prevail, only if Landco retained the rightto direct and control Poolco’sconstruction of the pool.

(B) prevail, because the blasting that Poolcowas hired to perform damaged Plaintiff’shome.

(C) not prevail, if Poolco used reasonablecare in conducting the blastingoperations.

(D) not prevail, if Landco used reasonablecare to hire a competent contractor.

GO ON TO THE NEXT PAGE.

Page 74: Sample MBE II

-72-

149. The state of Atlantica spends several milliondollars a year on an oyster conservationprogram. As part of that program, the statelimits, by statute, oyster fishing in its coastalwaters to persons who have state oysterpermits. In order to promote conservation, itissues only a limited number of oyster permitseach year. The permits are effective for onlyone year from the date of their issuance andare awarded on the basis of a lottery, in whichthere is no differentiation between resident andnonresident applicants. However, eachnonresident who obtains a permit is chargedan annual permit fee that is $5 more than thefee charged residents.

Fisher, Inc., is a large fishing company thatoperates from a port in another state and isincorporated in that other state. Each of theboats of Fisher, Inc., has a federal shippinglicense that permits it “to engage in all aspectsof the coastal trade, to fish and to carry cargofrom place to place along the coast, and toengage in other lawful activities along thecoast of the United States.” These shippinglicenses are authorized by federal statute.Assume no other federal statutes oradministrative rules apply.

Although it had previously held an Atlanticaoyster permit, Fisher, Inc., did not obtain apermit in that state’s lottery this year.

Which of the following is the strongestargument that can be made in support of acontinued right of Fisher, Inc., to fish foroysters this year in the coastal waters ofAtlantica?

(A) Because the Atlantica law provideshigher permit charges for nonresidents, itis an undue burden on interstatecommerce.

(B) Because the Atlantica law provideshigher permit charges for nonresidents, itdenies Fisher, Inc., the privileges andimmunities of state citizenship.

(C) Because it holds a federal shippinglicense, Fisher, Inc., has a right to fishfor oysters in Atlantica waters despite thestate law.

(D) Because Fisher, Inc., previously held anAtlantica oyster permit and Atlanticaknows that company is engaged in acontinuing business operation, the refusalto grant Fisher, Inc., a permit this year isa taking of its property without dueprocess of law.

GO ON TO THE NEXT PAGE.

Page 75: Sample MBE II

-73-

150. The United States Department of the Interiorgranted Concessionaire the food and drinkconcession in a federal park located in thestate of New Senora. Concessionaire operatedhis concession out of federally owned facilitiesin the park. The federal statute authorizing theInterior Department to grant such concessionsprovided that the grantees would pay only anominal rental for use of these federalfacilities because of the great benefit theirconcessions would provide to the people of theUnited States.

The legislature of the state of New Senoraenacted a statute imposing an occupancy taxon the occupants of real estate within that statethat is not subject to state real estate taxes. Thestatute was intended to equalize the state taxburden on such occupants with that on peopleoccupying real estate that is subject to statereal estate taxes. Pursuant to that statute, theNew Senora Department of Revenue attemptedto collect the state occupancy tax fromConcessionaire because the federal facilitiesoccupied by Concessionaire were not subjectto state real estate taxes. Concessionaire suedto invalidate the state occupancy tax as appliedto him.

The strongest ground upon whichConcessionaire could challenge the occupancytax is that it violates the

(A) commerce clause by unduly burdeningthe interstate tourist trade.

(B) privileges and immunities clause of theFourteenth Amendment by interferingwith the fundamental right to do businesson federal property.

(C) equal protection of the laws clause of theFourteenth Amendment because the taxtreats him less favorably than federalconcessionaires in other states who donot have to pay such occupancy taxes.

(D) supremacy clause of Article VI and thefederal statute authorizing suchconcessions.

151. Davis has a small trampoline in his backyardwhich, as he knows, is commonly used byneighbor children as well as his own. Thetrampoline is in good condition, is notdefective in any way, and normally issurrounded by mats to prevent injury if a usershould fall off. Prior to leaving with his familyfor the day, Davis leaned the trampoline upagainst the side of the house and placed themats in the garage.

While the Davis family was away, Philip, aged11, a new boy in the neighborhood, wanderedinto Davis’s yard and saw the trampoline.Philip had not previously been aware of itspresence, but, having frequently used atrampoline before, he decided to set it up, andstarted to jump. He lost his balance on onejump and took a hard fall on the bare ground,suffering a serious injury that would have beenprevented by the mats.

An action has been brought against Davis onPhilip’s behalf to recover damages for theinjuries Philip sustained from his fall. In thisjurisdiction, the traditional common-law rulespertaining to contributory negligence havebeen replaced by a pure comparativenegligence rule.

In his action against Davis, will Philip prevail?

(A) No, if children likely to be attracted bythe trampoline would normally realizethe risk of using it without mats.

(B) No, if Philip failed to exercise reasonablecare commensurate with his age,intelligence, and experience.

(C) No, because Philip entered Davis’s yardand used the trampoline without Davis’spermission.

(D) No, because Philip did not know aboutthe trampoline before entering Davis’syard and thus was not “lured” onto thepremises.

GO ON TO THE NEXT PAGE.

Page 76: Sample MBE II

-74-

152. Deben was charged with using a forgedprescription from a Dr. Kohl to obtainPercodan® from Smith’s Drugstore on May 1.At trial, Smith identified Deben as thecustomer, but Deben testified that he had notbeen in the store.

In rebuttal, the prosecutor calls Wallman andWitler to testify that on May 1 a man theyidentified as Deben had presentedprescriptions for Percodan® from a Dr. Kohl at,respectively, Wallman’s Drugs and Witler’sDrugstore.

Wallman’s and Witler’s testimony is

(A) admissible, to prove a pertinent trait ofDeben’s character and Deben’s action inconformity therewith.

(B) admissible, to identify the man whopresented the prescription at Smith’sDrugstore.

(C) inadmissible, because it proves specificacts rather than reputation or opinion.

(D) inadmissible, because other crimes maynot be used to show propensity.

153. An ordinance of the city of Green requires thatits mayor must have been continuously aresident of the city for at least five years at thetime he or she takes office. Candidate, who isthinking about running for mayor in anelection that will take place next year, willhave been a resident of Green for only fourand one-half years at the time the mayorelected then takes office. Before he decideswhether to run for the position of mayor,Candidate wants to know whether he couldlawfully assume that position if he wereelected. As a result, Candidate files suit in thelocal federal district court for a declaratoryjudgment that the Green five-year-residencerequirement is unconstitutional and that he isentitled to a place on his political party’sprimary election ballot for mayor. He namesthe chairman of his political party as the soledefendant but does not join any electionofficial. The chairman responds by joiningCandidate in requesting the court to declarethe Green residence requirement invalid.

In this case, the court should

(A) refuse to determine the merits of thissuit, because there is no case orcontroversy.

(B) refuse to issue such a declaratoryjudgment, because an issue of this kindinvolving only a local election does notpresent a substantial federalconstitutional question.

(C) issue the declaratory judgment, becausea residency requirement of this type is adenial of the equal protection of thelaws.

(D) issue the declaratory judgment, becauseCandidate will have substantiallycomplied with the residencyrequirement.

GO ON TO THE NEXT PAGE.

Page 77: Sample MBE II

-75-

154. Oliver, owner of Blackacre, needed money.Blackacre was fairly worth $100,000, soOliver tried to borrow $60,000 from Len onthe security of Blackacre. Len agreed, but onlyif Oliver would convey Blackacre to Lenoutright by warranty deed, with Len agreeingorally to reconvey to Oliver once the loan waspaid according to its terms. Oliver agreed,conveyed Blackacre to Len by warranty deed,and Len paid Oliver $60,000 cash. Lenpromptly and properly recorded Oliver’s deed.

Now, Oliver has defaulted on repayment with$55,000 still due on the loan. Oliver is still inpossession.

Which of the following best states the parties’rights in Blackacre?

(A) Len’s oral agreement to reconvey isinvalid under the Statute of Frauds, soLen owns Blackacre outright.

(B) Oliver, having defaulted, has no furtherrights in Blackacre, so Len may obtainsummary eviction.

(C) The attempted security arrangement is acreature unknown to the law, hence anullity; Len has only a personal right to$55,000 from Oliver.

(D) Len may bring whatever foreclosureproceeding is appropriate under the lawsof the jurisdiction.

155. Big City High School has had a very high rateof pregnancy among its students. In order toassist students who keep their babies tocomplete high school, Big City High Schoolhas established an infant day-care center forchildren of its students, and also offers classesin child-care. Because the child-care classesare always overcrowded, the school limitsadmission to those classes solely to Big CityHigh School students who are the mothers ofbabies in the infant day-care center.

Joe, a student at Big City High School, haslegal custody of his infant son. The schoolprovides care for his son in its infant day-carecenter, but will not allow Joe to enroll in thechild-care classes. He brings suit against theschool challenging, on constitutional grounds,his exclusion from the child-care classes.

Which of the following best states the burdenof persuasion in this case?

(A) Joe must demonstrate that the admissionrequirement is not rationally related to alegitimate governmental interest.

(B) Joe must demonstrate that the admissionrequirement is not as narrowly drawn aspossible to achieve a substantialgovernmental interest.

(C) The school must demonstrate that theadmission policy is the least restrictivemeans by which to achieve a compellinggovernmental interest.

(D) The school must demonstrate that theadmission policy is substantially relatedto an important governmental interest.

GO ON TO THE NEXT PAGE.

Page 78: Sample MBE II

-76-

156. Defendant was upset because he was going tohave to close his liquor store due tocompetition from a discount store in a newshopping mall nearby. In desperation, hedecided to set fire to his store to collect theinsurance. While looking through thebasement for flammable material, he lit amatch to read the label on a can. The matchburned his finger and, in a reflex action, hedropped the match. It fell into a barrel andignited some paper. Defendant made no effortto put out the fire but instead left the building.The fire spread and the store was destroyed byfire. Defendant was eventually arrested andindicted for arson.

Defendant is

(A) guilty, if he could have put out the firebefore it spread and did not do sobecause he wanted the buildingdestroyed.

(B) guilty, if he was negligent in starting thefire.

(C) not guilty, because even if he wanted toburn the building there was noconcurrence between his mens rea andthe act of starting the fire.

(D) not guilty, because his starting the firewas the result of a reflex action and not avoluntary act.

157. In his employment, Grinder operates agrinding wheel. To protect his eyes, he wearsglasses, sold under the trade name “SafetyGlasses,” manufactured by Glassco. Theglasses were sold with a warning label statingthat they would protect only against small,flying objects. One day, the grinding wheelGrinder was using disintegrated and fragmentsof the stone wheel were thrown off with greatforce. One large fragment hit Grinder,knocking his safety glasses up onto hisforehead. Another fragment then hit andinjured his eye.

Grinder brought an action against Glassco forthe injury to his eye. The jurisdiction adheresto the traditional common-law rule pertainingto contributory negligence.

In this action, will Grinder prevail?

(A) Yes, because the safety glasses weredefective in that they did not protect himfrom the disintegrating wheel.

(B) Yes, because the glasses were sold underthe trade name “Safety Glasses.”

(C) No, because the glasses were notdesigned or sold for protection againstthe kind of hazard Grinder encountered.

(D) No, if Grinder will be compensatedunder the workers’ compensation law.

GO ON TO THE NEXT PAGE.

Page 79: Sample MBE II

-77-

Questions 158-160 are based on the following fact situation.

Oscar purchased a large bottle of No-Flake dandruffshampoo, manufactured by Shampoo Company. Thebox containing the bottle stated in part:“CAUTION--Use only 1 capful at most once a day.Greater use may cause severe damage to the scalp.”Oscar read the writing on the box, removed thebottle, and threw the box away. Oscar’s roommate,Paul, asked to use the No-Flake, and Oscar said,“Be careful not to use too much.” Paul thereafterused No-Flake twice a day, applying two or threecapfuls each time, notwithstanding the labelstatement that read: “Use no more than one capfulper day. See box instructions.” The more he usedNo-Flake, the more inflamed his scalp became, themore it itched, and the more he used. After threeweeks of such use, Paul finally consulted a doctorwho diagnosed his problem as a serious andirreversible case of dermatitis caused by excessiveexposure to the active ingredients in No-Flake.These ingredients are uniquely effective atcontrolling dandruff, but there is no way to removea remote risk to a small percentage of persons whomay contract dermatitis as the result of applying forprolonged periods of time amounts of No-Flakesubstantially in excess of the directions. Thisjurisdiction adheres to the traditional common-lawrules pertaining to contributory negligence andassumption of risk.

158. Based upon the foregoing facts, if Paul suesShampoo Company to recover damages for hisdermatitis, his most promising theory ofliability will be that the No-Flake shampoo

(A) had an unreasonably dangerousmanufacturing defect.

(B) had an unreasonably dangerous designdefect.

(C) was inherently dangerous.(D) was inadequately labeled to warn of its

dangers.

159. If Paul asserts a claim for his injuries againstShampoo Company based on strict liability intort, which of the following would constitute adefense?

I. Paul misused the No-Flake shampoo.II. Paul was contributorily negligent in

continuing to use No-Flake shampoowhen his scalp began to hurt and itch.

III. Paul was a remote user and not in privitywith Shampoo Company.

(A) I only.(B) I and II only.(C) II and III only.(D) Neither I, nor II, nor III.

160. If Paul asserts a claim against Oscar for hisdermatitis injuries, Oscar’s best defense willbe that

(A) Paul was contributorily negligent.(B) Paul assumed the risk.(C) Oscar had no duty toward Paul, who was

a gratuitous donee.(D) Oscar had no duty toward Paul, because

Shampoo Company created the risk andhad a nondelegable duty to foreseeableusers.

GO ON TO THE NEXT PAGE.

Page 80: Sample MBE II

-78-

161. Unprepared for a final examination, Slickasked his girlfriend, Hope, to set off the firealarms in the university building 15 minutesafter the test commenced. Hope did so.Several students were injured in the panic thatfollowed as people were trying to get out ofthe building. Slick and Hope are prosecutedfor battery and for conspiracy to commitbattery.

They are

(A) guilty of both crimes.(B) guilty of battery but not guilty of

conspiracy.(C) not guilty of battery but guilty of

conspiracy.(D) not guilty of either crime.

162. A statute of the state of Wasminia prohibits theuse of state-owned or state-operated facilitiesfor the performance of abortions that are not“necessary to save the life of the mother.” Thatstatute also prohibits state employees fromperforming any such abortions during thehours they are employed by the state.

Citizen was in her second month of pregnancy.She sought an abortion at the Wasminia StateHospital, a state-owned and state-operatedfacility. Citizen did not claim that therequested abortion was necessary to save herlife. The officials in charge of the hospitalrefused to perform the requested abortionsolely on the basis of the state statute. Citizenimmediately filed suit against those officials inan appropriate federal district court. Shechallenged the constitutionality of theWasminia statute and requested the court toorder the hospital to perform the abortion shesought.

In this case, the court will probably hold thatthe Wasminia statute is

(A) unconstitutional, because a limit on theavailability of abortions performed bystate employees or in state-owned orstate-operated facilities to situations inwhich it is necessary to save the life ofthe mother impermissibly interferes withthe fundamental right of Citizen todecide whether to have a child.

(B) unconstitutional, because itimpermissibly discriminates against poorpersons who cannot afford to pay forabortions in privately owned andoperated facilities and against personswho live far away from privately ownedand operated abortion clinics.

(C) constitutional, because it does notprohibit a woman from having anabortion or penalize her for doing so, itis rationally related to the legitimategovernmental goal of encouragingchildbirth, and it does not interfere withthe voluntary performance of abortionsby private physicians in private facilities.

(D) constitutional, because the use of state-owned or state-operated facilities andaccess to the services of state employeesare privileges and not rights and,therefore, a state may condition them onany basis it chooses.

GO ON TO THE NEXT PAGE.

Page 81: Sample MBE II

-79-

163. Oscar, owner of Greenacre, conveyedGreenacre by quitclaim deed as a gift to Ann,who did not then record her deed. Later, Oscarconveyed Greenacre by warranty deed toBelle, who paid valuable consideration, knewnothing of Ann’s claim, and promptly andproperly recorded. Next, Ann recorded herdeed. Then Belle conveyed Greenacre byquitclaim deed to her son Cal as a gift. Whenthe possible conflict with Ann was discovered,Cal recorded his deed.

Greenacre at all relevant times has been vacantunoccupied land.

The recording act of the jurisdiction provides:“No unrecorded conveyance or mortgage ofreal property shall be good against subsequentpurchasers for value without notice, who shallfirst record.” No other statute is applicable.

Cal has sued Ann to establish who ownsGreenacre.

The court will hold for

(A) Cal, because Ann was a donee.(B) Cal, because Belle’s purchase cut off

Ann’s rights.(C) Ann, because she recorded before Cal.(D) Ann, because Cal was a subsequent

donee.

164. While Driver was taking a leisurely springdrive, he momentarily took his eyes off theroad to look at some colorful trees in bloom.As a result, his car swerved a few feet off theroadway, directly toward Walker, who wasstanding on the shoulder of the road waitingfor a chance to cross. When Walker saw thecar bearing down on him, he jumpedbackwards, fell, and injured his knee.

Walker sued Driver for damages, and Drivermoved for summary judgment. The foregoingfacts are undisputed.

Driver’s motion should be

(A) denied, because the record shows thatWalker apprehended an imminent,harmful contact with Driver’s car.

(B) denied, because a jury could find thatDriver negligently caused Walker tosuffer a legally compensable injury.

(C) granted, because the proximate cause ofWalker’s injury was his own voluntaryact.

(D) granted, because it is not unreasonablefor a person to be distractedmomentarily.

GO ON TO THE NEXT PAGE.

Page 82: Sample MBE II

-80-

165. In which of the following situations is thedefendant most likely to be convicted, eventhough he did not intend to bring about theharm that the statute defining the offense isdesigned to prevent?

(A) Defendant was the president of an aspirinmanufacturing company. A federalinspector discovered that a large numberof aspirin tablets randomly scatteredthrough several bottles in a carton readyfor shipment were laced with arsenic.Defendant is charged with attemptedintroduction of adulterated drugs intointerstate commerce.

(B) Defendant struck Victim in the face witha baseball bat, intending to inflict aserious injury. Victim died after beinghospitalized for three days. Defendant ischarged with murder.

(C) Defendant burglarized a jewelry store,intending to steal some diamonds. As heentered the store, he short-circuited thestore’s burglar alarm system, therebypreventing a warning of his entry topolice. The smoldering wires eventuallycaused a fire that destroyed the store.Defendant is charged with arson.

(D) Defendant wanted to frighten Victim’sfriend by placing a plastic rattlesnake inhis lunch box. When Victim mistakenlytook the lunch box and opened it,believing it to be his own, the plasticrattlesnake popped out. As a result of thefright, Victim suffered a heart attack anddied. Defendant is charged withmanslaughter.

166. Happy-Time Beverages agreed in writing withFizzy Cola Company to serve for three yearsas a distributor in a six-county area of FizzyCola, which contains a small amount ofcaffeine. Happy-Time promised in the contractto “promote in good faith the sale of FizzyCola” in that area; but the contract saidnothing about restrictions on the products thatHappy-Time could distribute.

Six months later, Happy-Time agreed with theCool Cola Company to distribute its caffeine-free cola beverages in the same six-countyarea.

If Fizzy Cola Company sues Happy-Time forbreach of their distribution contract, which ofthe following facts, if established, would moststrengthen Fizzy’s case?

(A) Cool Cola’s national advertisingcampaign disparages the Fizzy Colaproduct by saying, “You don’t needcaffeine and neither does your cola.”

(B) Since Happy-Time began to distributeCool Cola, the sales of Fizzy Cola havedropped 3% in the six-county area.

(C) Prior to signing the contract with FizzyCola Company, a representative ofHappy-Time said that the deal with Fizzywould be “an exclusive.”

(D) For many years in the soft-drinkindustry, it has been uniform practice fordistributors to handle only one brand ofcola.

GO ON TO THE NEXT PAGE.

Page 83: Sample MBE II

-81-

167. Dove is on trial for theft. At trial, theprosecutor called John and May Wong. Theytestified that, as they looked out theirapartment window, they saw thieves across thestreet break the window of a jewelry store,take jewelry, and leave in a car. Mrs. Wongtelephoned the police and relayed to them thelicense number of the thieves’ car as Mr. Wonglooked out the window with binoculars andread it to her. Neither of them has any presentmemory of the number. The prosecutor offersas evidence a properly authenticated policetape recording of May Wong’s telephone callwith her voice giving the license number,which is independently shown to belong toDove’s car.

The tape recording of May Wong’s stating thelicense number is

(A) admissible, under the hearsay exceptionfor present sense impressions.

(B) admissible, as nonhearsay circumstantialevidence.

(C) inadmissible, because it is hearsay notwithin any exception.

(D) inadmissible, because May Wong neverhad firsthand knowledge of the licensenumber.

168. Diggers Construction Company was engagedin blasting operations to clear the way for anew road. Diggers had erected adequatebarriers and posted adequate warning signs inthe vicinity of the blasting. Although Paul readand understood the signs, he entered the areato walk his dog. As a result of the blasting,Paul was hit by a piece of rock and sustainedhead injuries. The jurisdiction follows thetraditional common-law rules governing thedefenses of contributory negligence,assumption of risk, and last clear chance.

In an action by Paul against Diggers to recoverdamages for his injuries, Paul will

(A) not prevail, if Diggers exercisedreasonable care to protect the publicfrom harm.

(B) not prevail, because Paul understood thesigns and disregarded the warnings.

(C) prevail, because Paul was harmed byDiggers’s abnormally dangerous activity.

(D) prevail, unless Paul failed to usereasonable care to protect himself fromharm.

GO ON TO THE NEXT PAGE.

Page 84: Sample MBE II

-82-

169. Pike sued Day City Community Church fordamages he suffered when Pike crashed hismotorcycle in an attempt to avoid a cow thathad escaped from its corral. The cow andcorral belonged to a farm that had recentlybeen left by will to the church. At trial, Pikeseeks to ask Defendant’s witness, Winters,whether she is a member of that church.

The question is

(A) improper, because evidence of awitness’s religious beliefs is notadmissible to impeach credibility.

(B) improper, because it violates FirstAmendment and privacy rights.

(C) proper, for the purpose of ascertainingpartiality or bias.

(D) proper, for the purpose of showingcapacity to appreciate the nature andobligation of an oath.

170. Radon is a harmful gas found in the soil ofcertain regions of the United States. A statuteof the state of Magenta requires occupants ofresidences with basements susceptible to theintrusion of radon to have their residencestested for the presence of radon and to takespecified remedial steps if the test indicatesthe presence of radon above specified levels.The statute also provides that the testing forradon may be done only by testers licensed bya state agency. According to the statute, a firmmay be licensed to test for radon only if itmeets specified rigorous standards relating tothe accuracy of its testing. These standardsmay easily be achieved with currenttechnology; but the technology required tomeet them is 50% more expensive than thetechnology required to measure radonaccumulations in a slightly less accuratemanner.

The United States Environmental ProtectionAgency (EPA) does not license radon testers.However, a federal statute authorizes the EPAto advise on the accuracy of various methodsof radon testing and to provide to the generalpublic a list of testers that use methods itbelieves to be reasonably accurate.

WeTest, a recently established Magenta firm,uses a testing method that the EPA has statedis reasonably accurate. WeTest is also includedby the EPA on the list of testers using methodsof testing it believes to be reasonably accurate.WeTest applies for a Magenta radon testinglicense, but its application is denied becauseWeTest cannot demonstrate that the method oftesting for radon it uses is sufficiently accurateto meet the rigorous Magenta statutorystandards. WeTest sues appropriate Magentaofficials in federal court claiming thatMagenta may not constitutionally excludeWeTest from performing the required radontests in Magenta.

In this suit, the court will probably rule infavor of

(A) WeTest, because the full faith and creditclause of the Constitution requiresMagenta to respect and give effect to theaction of the EPA in including WeTest onits list of testers that use reasonablyaccurate methods.

(B) WeTest, because the supremacy clause ofthe Constitution requires Magenta torespect and give effect to the action ofthe EPA in including WeTest on its list oftesters that use reasonably accuratemethods.

(C) Magenta, because the federal statute andthe action of the EPA in includingWeTest on its list of testers that usereasonably accurate methods are notinconsistent with the more rigorousMagenta licensing requirement, and thatrequirement is reasonably related to alegitimate public interest.

(D) Magenta, because radon exposure islimited to basement areas, which, bytheir very nature, cannot move ininterstate commerce.

GO ON TO THE NEXT PAGE.

Page 85: Sample MBE II

-83-

171. Bitz, an amateur computer whiz, agreed inwriting to design for the Presskey Corporation,a distributor of TV game systems, three newgames a year for a five-year period. Thewriting provided, in a clause separately signedby Bitz, that “No modification shall be bindingon Presskey unless made in writing and signedby Presskey’s authorized representative.”

Because of family problems, Bitz deliveredand Presskey accepted only two game-designsa year for the first three years; but the gameswere a commercial success and Presskey madeno objection. Accordingly, Bitz spentsubstantial sums on new computer equipmentthat would aid in speeding up future designwork. In the first quarter of the fourth year,however, Presskey terminated the contract onthe ground that Bitz had breached the annual-quantity term.

In Bitz’s suit against Presskey for damages,the jury found that the contract had beenmodified by conduct and the trial courtawarded Bitz substantial compensatorydamages.

Is this result likely to be reversed on appeal?

(A) Yes, because the contract’s no-oral-modification clause was not expresslywaived by Presskey.

(B) Yes, because the contract’s no-oral-modification clause was a material partof the agreed exchange and could not beavoided without new consideration.

(C) No, because the contract’s no-oral-modification clause was unconscionableas against an amateur designer.

(D) No, because Presskey by its conductwaived the annual-quantity term and Bitzmaterially changed his position inreasonable reliance on that waiver.

172. Test owned Blackacre, a vacant one-acre tractof land in State. Five years ago, he executed adeed conveying Blackacre to “Church for thepurpose of erecting a church buildingthereon.” Three years ago, Test died leavingSonny as his sole heir at law. His dulyprobated will left “all my Estate, both real andpersonal, to my friend Fanny.”

Church never constructed a church building onBlackacre and last month Church, for a validconsideration, conveyed Blackacre toDeveloper.

Developer brought an appropriate action toquiet title against Sonny, Fanny, and Church,and joined the appropriate state official. Suchofficial asserted that a charitable trust wascreated which has not terminated.

In such action, the court should find that titleis now in

(A) Developer.(B) Sonny.(C) Fanny.(D) the state official.

GO ON TO THE NEXT PAGE.

Page 86: Sample MBE II

-84-

173. Mr. Denby was charged with the sale ofnarcotics. The federal prosecutor arrangedwith Mrs. Denby for her to testify against herhusband in exchange for leniency in her case.At trial, the prosecution calls Mrs. Denby, whohad been granted immunity from prosecution,to testify, among other things, that she saw herhusband sell an ounce of heroin.

Which of the following statements is mostclearly correct in the federal courts?

(A) Mrs. Denby cannot be called as a witnessover her husband’s objection.

(B) Mrs. Denby can be called as a witnessbut cannot testify, over Mr. Denby’sobjection, that she saw him sell heroin.

(C) Mrs. Denby can refuse to be a witnessagainst her husband.

(D) Mrs. Denby can be required to be awitness and to testify that she saw herhusband sell heroin.

174. Freund, a U.S. west-coast manufacturer, gaveWrench, a hardware retailer who wasrelocating to the east coast, the following“letter of introduction” to Tuff, an east-coasthardware wholesaler.

This will introduce you to my goodfriend and former customer, Wrench,who will be seeking to arrange thepurchase of hardware inventory fromyou on credit. If you will let him havethe goods, I will make good any loss upto $25,000 in the event of his default.

/Signed/ Freund

Wrench presented the letter to Tuff, who thensold and delivered $20,000 worth of hardwareto Wrench on credit. Tuff promptly notifiedFreund of this sale.

Which of the following is NOT an accuratestatement concerning the arrangement betweenFreund and Tuff?

(A) It was important to enforceability ofFreund’s promise to Tuff that it beembodied in a signed writing.

(B) By extending the credit to Wrench, Tuffeffectively accepted Freund’s offer for aunilateral contract.

(C) Although Freund received noconsideration from Wrench, Freund’spromise is enforceable by Tuff.

(D) Freund’s promise is enforceable by Tuffwhether or not Tuff gave Freundseasonable notice of the extension ofcredit to Wrench.

GO ON TO THE NEXT PAGE.

Page 87: Sample MBE II

-85-

175. The legislature of the state of Gray recentlyenacted a statute forbidding public utilitiesregulated by the Gray Public ServiceCommission to increase their rates more thanonce every two years. Economy ElectricPower Company, a public utility regulated bythat commission, has just obtained approval ofthe commission for a general rate increase.Economy Electric has routinely filed for a rateincrease every ten to 14 months during the last20 years. Because of uncertainties about futurefuel prices, the power company cannotascertain with any certainty the date when itwill need a further rate increase; but it thinks itmay need such an increase sometime withinthe next 18 months.

Economy Electric files an action in the federaldistrict court in Gray requesting a declaratoryjudgment that this new statute of Grayforbidding public utility rate increases moreoften than once every two years isunconstitutional. Assume no federal statute isrelevant.

In this case, the court should

(A) hold the statute unconstitutional, becausesuch a moratorium on rate increasesdeprives utilities of their propertywithout due process of law.

(B) hold the statute constitutional, becausethe judgment of a legislature on a matterinvolving economic regulation is entitledto great deference.

(C) dismiss the complaint, because thisaction is not ripe for decision.

(D) dismiss the complaint, becausecontroversies over state-regulated utilityrates are outside of the jurisdictionconferred on federal courts by Article IIIof the Constitution.

176. Daniel is on trial for evading $100,000 intaxes. The prosecution offers in evidence ananonymous letter to the IRS, identified asbeing in Daniel’s handwriting, saying, “Ipromised my mother on her deathbed I wouldtry to pay my back taxes. Here is $10,000. I’llmake other payments if you promise not toprosecute. Answer yes by personal ad saying,‘OK on tax deal.’ ”

The letter is

(A) admissible, as a statement of presentintention or plan.

(B) admissible, as an admission of a partyopponent.

(C) inadmissible, because it is an effort tosettle a claim.

(D) inadmissible, because the probative valueis substantially outweighed by the risk ofunfair prejudice.

GO ON TO THE NEXT PAGE.

Page 88: Sample MBE II

-86-

Questions 177-178 are based on the following fact situation.

Broker needed a certain rare coin to complete a setthat he had contracted to assemble and sell toCollecta. On February 1, Broker obtained such acoin from Hoarda in exchange for $1,000 andBroker’s signed, written promise to re-deliver toHoarda “not later than December 31 this year” acomparable specimen of the same kind of coinwithout charge to Hoarda. On February 2, Brokerconsummated sale of the complete set to Collecta.

On October 1, the market price of rare coinssuddenly began a rapid, sustained rise; and onOctober 15 Hoarda wrote Broker for assurance thatthe latter would timely meet his coin-replacementcommitment. Broker replied, “In view of thesurprising market, it seems unfair that I should haveto replace your coin within the next few weeks.”

177. For this question only, assume the followingfacts. Having received Broker’s message onOctober 17, Hoarda sued Broker on November15 for the market value of a comparablereplacement-coin as promised by Broker inFebruary. The trial began on December 1.

If Broker moves to dismiss Hoarda’scomplaint, which of the following is Broker’sbest argument in support of the motion?

(A) Broker did not repudiate the contract onOctober 17, and may still perform nolater than the contract deadline ofDecember 31.

(B) Even if Broker repudiated on October17, Hoarda’s only action would be forspecific performance because the coin isa unique chattel.

(C) Under the doctrine of impossibility,which includes unusually burdensomeand unforeseen impracticability, Brokeris temporarily excused by the marketconditions from timely performance ofhis coin-replacement obligation.

(D) Even if Broker repudiated on October17, Hoarda has no remedy without firstdemanding in writing that Broker retracthis repudiation.

178. For this question only, assume the followingfacts. After receiving Broker’s message onOctober 17, Hoarda telephoned Broker, whosaid, “I absolutely will not replace your coinuntil the market drops far below its presentlevel.” Hoarda then sued Broker on November15 for the market value of a comparablereplacement-coin as promised by Broker inFebruary. The trial began on December 1.

If Broker moves to dismiss Hoarda’scomplaint, which of the following is Hoarda’sbest argument in opposing the motion?

(A) Hoarda’s implied duty of good faith andfair dealing in enforcement of thecontract required her to mitigate herlosses on the rising market by suingpromptly, as she did, after becomingreasonably apprehensive of a prospectivebreach by Broker.

(B) Although the doctrine of anticipatorybreach is not applicable under theprevailing view if, at the time ofrepudiation, the repudiatee owes therepudiator no remaining duty ofperformance, the doctrine applies in thiscase because Hoarda, the repudiatee,remains potentially liable under animplied warranty that the coin advancedto Broker was genuine.

(C) When either party to a sale-of-goodscontract repudiates with respect to aperformance not yet due, the loss ofwhich will substantially impair the valueof the contract to the other, the aggrievedparty may in good faith resort to anyappropriate remedy for breach.

(D) Anticipatory repudiation, as a deliberatedisruption without legal excuse of anongoing contractual relationship betweenthe parties, may be treated by therepudiatee at her election as a presenttort, actionable at once.

GO ON TO THE NEXT PAGE.

Page 89: Sample MBE II

-87-

179. Alice owned a commercial property, Eastgate,consisting of a one-story building rented tovarious retail stores and a very large parkinglot. Two years ago, Alice died and leftEastgate to her nephew, Paul, for life, withremainder to her godson, Richard, his heirsand assigns. Paul was 30 years old andRichard was 20 years old when Alice died.The devise of Eastgate was made subject toany mortgage on Eastgate in effect at the timeof Alice’s death.

When Alice executed her will, the balance ofthe mortgage debt on Eastgate was less than$5,000. A year before her death, Alice sufferedfinancial reverses; and in order to meet herdebts, she had mortgaged Eastgate to secure aloan of $150,000. The entire principal of themortgage remained outstanding when shedied. As a result, the net annual income fromEastgate was reduced not only by real estatetaxes and regular maintenance costs, but alsoby the substantial mortgage interest paymentsthat were due each month.

Paul was very dissatisfied with the limitedbenefit that he was receiving from the lifeestate. When, earlier this year, Acme, Inc.,proposed to purchase Eastgate, demolish thebuilding, pay off the mortgage, and construct a30-story office building, Paul was willing toaccept Acme’s offer. However, Richardadamantly refused the offer, even thoughRichard, as the remainderman, paid theprincipal portion of each monthly mortgageamortization payment. Richard wasindependently wealthy and wanted to convertEastgate into a public park when he becameentitled to possession.

When Acme realized that Richard would notchange his mind, Acme modified its proposalto a purchase of the life estate of Paul. Acmewas ready to go ahead with its building plans,relying upon a large life insurance policy onPaul’s life to protect it against the economicrisk of Paul’s death. Paul’s life expectancy was45 years.

When Richard learned that Paul had agreed toAcme’s modified proposal, Richard brought anappropriate action against them to enjoin theircarrying it out.

There is no applicable statute.

The best argument for Richard is that

(A) Acme cannot purchase Paul’s life estate,because life estates are not assignable.

(B) the proposed demolition of the buildingconstitutes waste.

(C) Richard’s payment of the mortgageprincipal has subrogated him to Paul’srights as a life tenant and bars Paul’sassignment of the life estate withoutRichard’s consent.

(D) continued existence of the one-storybuilding is more in harmony with theultimate use as a park than the proposedchange in use.

GO ON TO THE NEXT PAGE.

Page 90: Sample MBE II

-88-

180. Doppler is charged with aggravated assault onVezy, a game warden. Doppler testified that,when he was confronted by Vezy, who wasarmed and out of uniform, Doppler believedVezy was a robber and shot in self-defense.The state calls Willy to testify that a yearearlier, he had seen Doppler shoot a manwithout provocation and thereafter falselyclaim self-defense.

Willy’s testimony is

(A) admissible, as evidence of Doppler’suntruthfulness.

(B) admissible, as evidence that Doppler didnot act in self-defense on this occasion.

(C) inadmissible, because it is impropercharacter evidence.

(D) inadmissible, because it is irrelevant tothe defense Doppler raised.

181. Eddie worked as the cashier in a restaurant.One night after the restaurant had closed,Eddie discovered that the amount of cash inthe cash register did not match the cashregister receipt tapes. He took the cash and thetapes, put them in a bag, gave them to Rita,the manager of the restaurant, and reported thediscrepancy. Rita immediately accused him oftaking money from the register and threatenedto fire him if he did not make up thedifference. Rita placed the bag in the officesafe. Angered by what he considered to be anunjust accusation, Eddie waited until Rita leftthe room and then reached into the still opensafe, took the bag containing the cash, and left.

Eddie is guilty of

(A) larceny.(B) embezzlement.(C) either larceny or embezzlement but not

both.(D) neither larceny nor embezzlement.

182. A grand jury returned an indictment chargingDaniels with bank robbery, and when he couldnot make bond he was jailed pending trial. Hehad received Miranda warnings when arrestedand had made no statement at that time. Theprosecutor arranged to have Innis, aninformant, placed as Daniels’s cellmate andinstructed Innis to find out about the bankrobbery without asking any direct questionsabout it. Innis, once in the cell, constantlyboasted about the crimes that he hadcommitted. Not to be outdone, Daniels finallydeclared that he had committed the bankrobbery with which he was charged.

At Daniels’s trial, his attorney moved toexclude any testimony from Innis concerningDaniels’s boast.

The motion should be

(A) granted, because Daniels’s privilegeagainst self-incrimination was violated.

(B) granted, because Daniels’s right tocounsel was violated.

(C) denied, because Daniels had receivedMiranda warnings.

(D) denied, because Daniels was notinterrogated by Innis.

183. Pamela sued Driver for damages for the deathof Pamela’s husband Ronald, resulting from anautomobile collision. At trial, Driver callsRonald’s doctor to testify that the day beforehis death, Ronald, in great pain, said, “It wasmy own fault; there’s nobody to blame butme.”

The doctor’s testimony should be admitted as

(A) a statement against interest.(B) a dying declaration.(C) a statement of Ronald’s then existing

state of mind.(D) an excited utterance.

GO ON TO THE NEXT PAGE.

Page 91: Sample MBE II

-89-

184. Clerk is a clerical worker who has beenemployed for the past two years in apermanent position in the Wasmania CountyPublic Records Office in the state of Orange.Clerk has been responsible for copying andfiling records of real estate transactions in thatoffice. Clerk works in a nonpublic part of theoffice and has no contact with members of thepublic. However, state law provides that allreal estate records in that office are to be madeavailable for public inspection.

On the day an attempted assassination of thegovernor of Orange was reported on the radio,Clerk remarked to a coworker, “Our governoris such an evil man, I am sorry they did notget him.” Clerk’s coworker reported thisremark to Clerk’s employer, the countyrecorder. After Clerk admitted making theremark, the county recorder dismissed himstating that “there is no room in this office fora person who hates the governor so much.”

Clerk sued for reinstatement and back pay. Hisonly claim is that the dismissal violated hisconstitutional rights.

In this case, the court should hold that thecounty recorder’s dismissal of Clerk was

(A) unconstitutional, because it constitutes ataking without just compensation ofClerk’s property interest in hispermanent position with the county.

(B) unconstitutional, because in light ofClerk’s particular employment duties hisright to express himself on a matter ofpublic concern outweighed anylegitimate interest the state might havehad in discharging him.

(C) constitutional, because the compellinginterest of the state in having loyal andsupportive employees outweighs theinterest of any state employee in his orher job or in free speech on a matter ofpublic concern.

(D) nonjusticiable, because publicemployment is a privilege rather than aright and, therefore, Clerk lackedstanding to bring this suit.

185. Slalome, a ski-shop operator, in a telephoneconversation with Mitt, a glove manufacturer,ordered 12 pairs of vortex-lined ski gloves atMitt’s list price of $600 per dozen “fordelivery in 30 days.” Mitt orally accepted theoffer, and immediately faxed to Slalome thissigned memo: “Confirming our agreementtoday for your purchase of a dozen pairs ofvortex-lined ski gloves for $600, the shipmentwill be delivered in 30 days.” AlthoughSlalome received and read Mitt’s messagewithin minutes after its dispatch, she changedher mind three weeks later about the purchaseand rejected the conforming shipment when ittimely arrived.

On learning of the rejection, does Mitt have acause of action against Slalome for breach ofcontract?

(A) Yes, because the gloves were identifiedto the contract and tendered to Slalome.

(B) Yes, because Mitt’s faxed memo toSlalome was sufficient to make theagreement enforceable.

(C) No, because the agreed price was $600and Slalome never signed a writingevidencing a contract with Mitt.

(D) No, because Slalome neither paid for noraccepted any of the goods tendered.

GO ON TO THE NEXT PAGE.

Page 92: Sample MBE II

-90-

186. A burglar stole Collecta’s impressionistpainting valued at $400,000. Collecta, whohad insured the painting for $300,000 withArtistic Insurance Co., promised to pay$25,000 to Snoop, a full-time investigator forArtistic, if he effected the return of thepainting to her in good condition. By companyrules, Artistic permits its investigators toaccept and retain rewards from policyholdersfor the recovery of insured property. Snoop, bylong and skillful detective work, recovered thepicture and returned it undamaged to Collecta.

If Collecta refuses to pay Snoop anything, andhe sues her for $25,000, what is the probableresult under the prevailing modern rule?

(A) Collecta wins, because Snoop owedArtistic a preexisting duty to recover thepicture if possible.

(B) Collecta wins, because Artistic, Snoop’semployer, had a preexisting duty toreturn the recovered painting to Collecta.

(C) Snoop wins, because Collecta willbenefit more from return of the $400,000painting than from receiving the$300,000 policy proceeds.

(D) Snoop wins, because the preexisting dutyrule does not apply if the promisee’s(Snoop’s) duty was owed to a thirdperson.

187. Oren owned Purpleacre, a tract of land, in feesimple. By will duly admitted to probate afterhis death, Oren devised Purpleacre to “anywife who survives me with remainder to suchof my children as are living at her death.”

Oren was survived by Wen, his wife, and bythree children, Cynthia, Cam, and Camelia.Thereafter, Cam died and by will dulyadmitted to probate devised his entire estate toDavid. Cynthia and Camelia were Cam’s heirsat law.

Later Wen died. In appropriate lawsuit towhich Cynthia, Camelia, and David areparties, title to Purpleacre is at issue.

In such lawsuit, judgment should be that titleto Purpleacre is in

(A) Cynthia, Camelia, and David, becausethe earliest vesting of remainders isfavored and reference to Wen’s deathshould be construed as relating to time oftaking possession.

(B) Cynthia, Camelia, and David, becausethe provision requiring survival ofchildren violates the Rule AgainstPerpetuities since the surviving wifemight have been a person unborn at thetime of writing of the will.

(C) Cynthia and Camelia, because Cam’sremainder must descend by intestacy andis not devisable.

(D) Cynthia and Camelia, because theremainders were contingent uponsurviving the life tenant.

GO ON TO THE NEXT PAGE.

Page 93: Sample MBE II

-91-

188. Allen and Bradley were law school classmateswho had competed for the position of editor ofthe law review. Allen had the higher gradepoint average, but Bradley was elected editor,largely in recognition of a long and importantnote that had appeared in the review over hername.

During the following placement interviewseason, Allen was interviewed by arepresentative of a nationally prominent lawfirm. In response to the interviewer’s requestfor information about the authorship of the lawreview note, Allen said that he had heard thatthe note attributed to Bradley was largely thework of another student.

The firm told Bradley that it would notinterview her because of doubts about theauthorship of the note. This greatly distressedBradley. In fact the note had been prepared byBradley without assistance from anyone else.

If Bradley asserts a claim against Allen basedon defamation, Bradley will

(A) recover, because Allen’s statement wasfalse.

(B) recover, if Allen had substantial doubtsabout the accuracy of the information hegave the interviewer.

(C) not recover, unless Bradley provespecuniary loss.

(D) not recover, because the statement wasmade by Allen only after the interviewerinquired about the authorship of the note.

Questions 189-190 are based on the following fact situation.

Sue Starr, a minor both in fact and appearance,bought on credit and took delivery of a telescopefrom 30-year-old Paul Prism for an agreed price of$100. Upon reaching her majority soon thereafter,Starr encountered Prism and said, “I am sorry fornot having paid you that $100 for the telescopewhen the money was due, but I found out it wasonly worth $75. So I now promise to pay you $75.”Starr subsequently repudiated this promise andrefused to pay Prism anything.

189. In an action for breach of contract by Prismagainst Starr, Prism’s probable recovery is

(A) nothing, because Starr was a minor at thetime of the original transaction.

(B) nothing, because there was noconsideration for the promise made byStarr after reaching majority.

(C) $75.(D) $100.

190. For this question only, assume that Starrbought the telescope from Prism after reachingher majority and promised to pay $100 “assoon as I am able.”

What effect does this quoted language have onenforceability of the promise?

(A) None.(B) It makes the promise illusory.(C) It requires Starr to prove her inability to

pay.(D) It requires Prism to prove Starr’s ability

to pay.

GO ON TO THE NEXT PAGE.

Page 94: Sample MBE II

-92-

191. Beach owned a tract of land called Blackacre.An old road ran through Blackacre from theabutting public highway. The road had beenused to haul wood from Blackacre. WithoutBeach’s permission and with no initial right,Daniel, the owner of Whiteacre, whichadjoined Blackacre, traveled over the old roadfor a period of 15 years to obtain access toWhiteacre, although Whiteacre abutted anotherpublic road. Occasionally, Daniel made repairsto the old road.

The period of time to acquire rights byprescription in the jurisdiction is ten years.

After the expiration of 15 years, Beachconveyed a portion of Blackacre to Carrol.The deed included the following clause:“together with the right to pass and repass atall times and for all purposes over the oldroad.” Carrol built a house fronting on the oldroad.

The road was severely damaged by a springflood, and Carrol made substantial repairs tothe road. Carrol asked Daniel and Beach tocontribute one-third each to the cost ofrepairing the flood damage. They both refused,and Carrol brought an appropriate action tocompel contribution from Beach and Daniel.

In this action, Carrol will

(A) lose as to both defendants.(B) win as to both defendants.(C) win as to Beach, but lose as to Daniel.(D) win as to Daniel, but lose as to Beach.

192. Prine sued Dover for an assault that occurredMarch 5 in California. To support his defensethat he was in Utah on that date, Doveridentifies and seeks to introduce a letter hewrote to his sister a week before the assault inwhich he stated that he would see her in Utahon March 5.

The letter is

(A) admissible, within the state of mindexception to the hearsay rule.

(B) admissible, as a prior consistentstatement to support Dover’s credibilityas a witness.

(C) inadmissible, because it lacks sufficientprobative value.

(D) inadmissible, because it is a statement ofbelief to prove the fact believed.

GO ON TO THE NEXT PAGE.

Page 95: Sample MBE II

-93-

193. Maple City has an ordinance that prohibits thelocation of “adult theaters and bookstores”(theaters and bookstores presenting sexuallyexplicit performances or materials) inresidential or commercial zones within thecity. The ordinance was intended to protectsurrounding property from the likely adversesecondary effects of such establishments.“Adult theaters and bookstores” are freelypermitted in the areas of the city zonedindustrial, where those adverse secondaryeffects are not as likely. Storekeeper is denieda zoning permit to open an adult theater andbookstore in a building owned by him in anarea zoned commercial. As a result,Storekeeper brings suit in an appropriate courtchallenging the constitutionality of the zoningordinance.

Which of the following statements regardingthe constitutionality of this Maple Cityordinance is most accurate?

(A) The ordinance is valid, because a citymay enforce zoning restrictions onspeech-related businesses to ensure thatthe messages they disseminate areacceptable to the residents of adjacentproperty.

(B) The ordinance is valid, because a citymay enforce this type of time, place, andmanner regulation on speech-relatedbusinesses, so long as this type ofregulation is designed to serve asubstantial governmental interest anddoes not unreasonably limit alternativeavenues of communication.

(C) The ordinance is invalid, because a citymay not enforce zoning regulations thatdeprive potential operators of adulttheaters and bookstores of their freedomto choose the location of theirbusinesses.

(D) The ordinance is invalid, because a citymay not zone property in a mannercalculated to protect property from thelikely adverse secondary effects of adulttheaters and bookstores.

194. Kingsley was prosecuted for selling cocaine toan undercover police agent. At his trial, hetestified that he only sold the drugs to theagent, whom Kingsley knew as “Speedy,”because Speedy had told him that he (Speedy)would be killed by fellow gang membersunless he supplied them with cocaine. Theprosecution did not cross-examine Kingsley.As rebuttal evidence, however, the prosecutorintroduced records, over Kingsley’s objection,showing that Kingsley had two priorconvictions for narcotics-related offenses. Thecourt instructed the jury concerning thedefense of entrapment and added, also overKingsley’s objection but in accord with statelaw, that it should acquit on the ground ofentrapment only if it found that the defendanthad established the elements of the defense bya preponderance of the evidence. Kingsley wasconvicted.

On appeal, Kingsley’s conviction should be

(A) reversed, because it was an error for thecourt to admit the evidence of his priorconvictions as substantive evidence.

(B) reversed, because it was a violation ofdue process to impose on the defense aburden of persuasion concerningentrapment.

(C) reversed, for both of the above reasons.(D) affirmed, because neither of the above

reasons constitutes a ground for reversal.

GO ON TO THE NEXT PAGE.

Page 96: Sample MBE II

-94-

Questions 195-196 are based on the following fact situation.

Pat sustained personal injuries in a three-carcollision caused by the concurrent negligence of thethree drivers, Pat, Donald, and Drew. In Pat’s actionfor damages against Donald and Drew, the juryapportioned the negligence 30% to Pat, 30% toDonald, and 40% to Drew. Pat’s total damages were$100,000.

195. Assume for this question only that a statestatute provides for a system of purecomparative negligence, joint and severalliability of concurrent tortfeasors, andcontribution based upon proportionate fault.

If Pat chooses to execute against Donaldalone, she will be entitled to collect at most

(A) $70,000 from Donald, and then Donaldwill be entitled to collect $40,000 fromDrew.

(B) $30,000 from Donald, and then Donaldwill be entitled to collect $10,000 fromDrew.

(C) $30,000 from Donald, and then Donaldwill be entitled to collect nothing fromDrew.

(D) nothing from Donald, because Donald’spercentage of fault is not greater thanthat of Pat.

196. Assume for this question only that the statehas retained the common-law rule pertainingto contribution and that the state’s comparativenegligence statute provides for a system ofpure comparative negligence but abolishesjoint and several liability.

If Pat chooses to execute against Donaldalone, she will be entitled to collect at most

(A) $70,000 from Donald, and then Donaldwill be entitled to collect $40,000 fromDrew.

(B) $30,000 from Donald, and then Donaldwill be entitled to collect $10,000 fromDrew.

(C) $30,000 from Donald, and then Donaldwill be entitled to collect nothing fromDrew.

(D) nothing from Donald, because Donald’spercentage of fault is not greater thanthat of Pat.

GO ON TO THE NEXT PAGE.

Page 97: Sample MBE II

-95-

197. Tess Traviata owed Dr. Paula Pulmonary, aphysician, $25,000 for professional services.Dr. Pulmonary orally assigned this claim toher adult daughter, Bridey, as a wedding gift.Shortly thereafter, on suffering sudden, severelosses in the stock market, Dr. Pulmonaryassigned by a signed writing the same claim toher stockbroker, Margin, in partial satisfactionof advances legally made by Margin in Dr.Pulmonary’s previous stock-markettransactions. Subsequently, Traviata, withoutknowledge of either assignment, paid Dr.Pulmonary the $25,000 then due, which Dr.Pulmonary promptly lost at a horse track,although she remains solvent.

Assuming that Article 9 of the UniformCommercial Code does NOT apply to either ofthe assignments in this situation, which of thefollowing is a correct statement of the parties’rights and liabilities?

(A) As the assignee prior in time, Bridey canrecover $25,000 from Traviata, whoacted at her peril in paying Dr.Pulmonary.

(B) As the sole assignee for value, Margincan recover $25,000 from Traviata, whoacted at her peril in paying Dr.Pulmonary.

(C) Neither Bridey nor Margin can recoverfrom Traviata, but Bridey, though notMargin, can recover $25,000 from Dr.Pulmonary.

(D) Neither Bridey nor Margin can recoverfrom Traviata, but Margin, though notBridey, can recover $25,000 from Dr.Pulmonary.

198. Patten suffered from a serious, though notimmediately life-threatening, impairment ofhis circulatory system. Patten’s cardiologistrecommended a cardiac bypass operation andreferred Patten to Dr. Cutter. Cutter did notinform Patten of the 2% risk of deathassociated with this operation. Cutter defendedhis decision not to mention the risk statistics toPatten because “Patten was a worrier and itwould significantly lessen his chances ofsurvival to be worried about the nonsurvivalrate.”

Cutter successfully performed the bypassoperation and Patten made a good recovery.However, when Patten learned of the 2% riskof death associated with the operation, he wasfurious that Cutter had failed to disclose thisinformation to him.

If Patten asserts a claim against Cutter basedon negligence, will Patten prevail?

(A) No, if Cutter used his best personaljudgment in shielding Patten from therisk statistic.

(B) No, because the operation was successfuland Patten suffered no harm.

(C) Yes, if Patten would have refused theoperation had he been informed of therisk.

(D) Yes, because a patient must be told therisk factor associated with a surgicalprocedure in order to give an informedconsent.

GO ON TO THE NEXT PAGE.

Page 98: Sample MBE II

-96-

199. A statute of the state of Orrington providesthat assessments of real property for taxpurposes must represent the “actual value” ofthe property. The Blue County TaxCommission, in making its assessments, hasuniformly and consistently determined the“actual value” of real property solely byreference to the price at which the particularproperty was last sold. In recent years, themarket values of real property in Blue Countyhave been rising at the rate of 15% per year.

Owner is required to pay real estate taxes onher home in Blue County that are 200% to300% higher than those paid by many otherowners of similar homes in similarneighborhoods in that county, even though thecurrent market values of their respectivehomes and Owner’s home are nearly identical.The reason the taxes on Owner’s home arehigher than those imposed on the other similarhomes in similar neighborhoods is that shebought her home much more recently than theother owners and, therefore, it is assessed at amuch higher “actual value” than their homes.Persistent efforts by Owner to have herassessment reduced or the assessments of theothers raised by the Blue County TaxCommission have failed.

Owner has now filed suit against the BlueCounty Tax Commission, charging only thatthe tax assessment on her property isunconstitutional.

The strongest constitutional argument tosupport Owner’s claim is that the comparativeovervaluation of Owner’s property by the BlueCounty Tax Commission in making taxassessments over time

(A) deprives Owner of the equal protectionof the laws.

(B) deprives Owner of a privilege orimmunity of national citizenship.

(C) constitutes a taking of private propertyfor public use without just compensation.

(D) constitutes an ex post facto law.

200. Plaza Hotel sued Plaza House Hotel forinfringement of its trade name. To establish alikelihood of name confusion, Plaintiff PlazaHotel offers a series of memoranda which ithad asked its employees to prepare at the endof each day listing instances during the day inwhich telephone callers, cab drivers,customers, and others had confused the twonames.

The memoranda should be

(A) excluded, because they are more unfairlyprejudicial and confusing than probative.

(B) excluded, because they are hearsay notwithin any exception.

(C) admitted, because they are records ofregularly conducted business activity.

(D) admitted, because they are pastrecollection recorded.

STOPIF YOU FINISH BEFORE TIME IS CALLED, CHECK YOUR WORK ON THIS TEST.

Page 99: Sample MBE II

-97-

Item Answer Subject001 C REAL PROP.002 C TORTS003 A TORTS004 C CONTRACTS005 D CONTRACTS006 A CRIM. LAW007 A EVIDENCE008 C CONST. LAW009 B CRIM. LAW010 B EVIDENCE011 C TORTS012 D EVIDENCE013 D REAL PROP.014 B TORTS015 B EVIDENCE016 A CONTRACTS017 B CONTRACTS018 A TORTS019 A CRIM. LAW020 C CONST. LAW021 C REAL PROP.022 A,B,D* CRIM. LAW023 D REAL PROP.024 A CONTRACTS025 C TORTS026 C EVIDENCE027 C CRIM. LAW028 D CONST. LAW029 D CONTRACTS030 B REAL PROP.031 B CONTRACTS032 C CONTRACTS033 B CONST. LAW034 D CONTRACTS035 C TORTS036 C REAL PROP.037 A EVIDENCE038 B TORTS039 A CRIM. LAW040 B CONTRACTS041 C CONTRACTS042 C TORTS043 D TORTS044 D CRIM. LAW045 A EVIDENCE046 B CONST. LAW047 C TORTS048 B REAL PROP.049 C CONST. LAW050 D CRIM. LAW051 B CONTRACTS052 C CONTRACTS053 A TORTS

Item Answer Subject054 D CONST. LAW055 B REAL PROP.056 A EVIDENCE057 B TORTS058 C EVIDENCE059 A CONST. LAW060 A CRIM. LAW061 C CONST. LAW062 B CONTRACTS063 A CONTRACTS064 D REAL PROP.065 C EVIDENCE066 C TORTS067 D REAL PROP.068 D CRIM. LAW069 A CONST. LAW070 A EVIDENCE071 B TORTS072 D TORTS073 B CRIM. LAW074 C CONTRACTS075 A CONTRACTS076 B TORTS077 C REAL PROP.078 A EVIDENCE079 A CONST. LAW080 B CRIM. LAW081 C REAL PROP.082 D EVIDENCE083 C CONST. LAW084 A,C* TORTS085 A EVIDENCE086 A CONTRACTS087 A CONTRACTS088 C REAL PROP.089 A REAL PROP.090 B EVIDENCE091 A CRIM. LAW092 A REAL PROP.093 C CRIM. LAW094 D CONST. LAW095 D TORTS096 D CRIM. LAW097 A CONST. LAW098 A CONST. LAW099 C CONTRACTS100 B TORTS101 B CONST. LAW102 D CRIM. LAW103 A TORTS104 B REAL PROP.105 A EVIDENCE106 A CONTRACTS

ANSWER KEY

Page 100: Sample MBE II

-98-

Item Answer Subject107 D REAL PROP.108 D TORTS109 A EVIDENCE110 D CRIM. LAW111 B CONTRACTS112 D CONST. LAW113 D EVIDENCE114 C REAL PROP.115 D CONTRACTS116 D CONTRACTS117 D CRIM. LAW118 C CONST. LAW119 C EVIDENCE120 A REAL PROP.121 A TORTS122 D CRIM. LAW123 B CONTRACTS124 D REAL PROP.125 C TORTS126 D TORTS127 A CRIM. LAW128 B CRIM. LAW129 D TORTS130 C TORTS131 C CONST. LAW132 A EVIDENCE133 B CONTRACTS134 B CONTRACTS135 A REAL PROP.136 D EVIDENCE137 D CRIM. LAW138 C REAL PROP.139 B REAL PROP.140 D TORTS141 B CONTRACTS142 B CONTRACTS143 D CRIM. LAW144 D CRIM. LAW145 C CONST. LAW146 C CONTRACTS147 B REAL PROP.148 B TORTS149 C CONST. LAW150 D CONST. LAW151 A TORTS152 B EVIDENCE153 A CONST. LAW

Item Answer Subject154 D REAL PROP.155 D CONST. LAW156 A CRIM. LAW157 C TORTS158 D TORTS159 D TORTS160 A TORTS161 B,D* CRIM. LAW162 C CONST. LAW163 B REAL PROP.164 B TORTS165 B CRIM. LAW166 D CONTRACTS167 A EVIDENCE168 B TORTS169 C EVIDENCE170 C CONST. LAW171 D CONTRACTS172 A REAL PROP.173 C EVIDENCE174 D CONTRACTS175 C CONST. LAW176 B EVIDENCE177 A CONTRACTS178 C CONTRACTS179 B REAL PROP.180 C EVIDENCE181 A CRIM. LAW182 B CRIM. LAW183 A EVIDENCE184 B CONST. LAW185 B CONTRACTS186 D CONTRACTS187 D REAL PROP.188 B TORTS189 C CONTRACTS190 D CONTRACTS191 A REAL PROP.192 A EVIDENCE193 B CONST. LAW194 D CRIM. LAW195 A TORTS196 C TORTS197 D CONTRACTS198 B TORTS199 A CONST. LAW200 B EVIDENCE

*Immediately following the administration of an MBE, preliminary scoring is conducted to identify any unanticipateditem functioning or unusual response patterns. For example, an item might be flagged if a large number of applicantswho did well on the test overall selected an option other than the key on that item. Flagged items are then reviewed bythe MBE Drafting Committees to assure there are no ambiguities and that they have been keyed correctly. If a contentproblem is identified, an item may be rekeyed, double-keyed, or eliminated from scoring by having all four optionskeyed correct. In a typical administration of the MBE, more than one option may be scored as correct on two or threeof the 200 items.

Page 101: Sample MBE II

-99-

MBE RAW SCORE CONVERSION TABLE

RAW SCORE SCALED SCORE

161 - 167 170 - 175

154 - 160 163 - 169

147 - 153 157 - 162

140 - 146 151 - 156

133 - 139 145 - 150

126 - 132 139 - 144

119 - 125 133 - 138

112 - 118 126 - 132

105 - 111 120 - 125

The raw score is the total number of correct answers given by an examinee. A statisticalprocedure is used to convert raw scores to scaled scores to make scores comparable acrosstest forms. A particular scaled score is intended to represent the same level of achievementfor all forms of the MBE, and scaled scores on one test form can be used interchangeablywith the scaled scores on another test form.

Page 102: Sample MBE II

SAMPLE

-100-

Page 103: Sample MBE II

MULTISTATE BAR EXAMINATIONTime—6 hours

This test consists of two parts, one of which will be administered in the morning and one in theafternoon. You will be given three hours to work on each of the parts. Be sure that the questionnumbers on your answer sheet match the question numbers in your test booklet. You are not tobegin work until the supervisor tells you to do so.

Your score will be based on the number of questions you answer correctly. It is therefore to youradvantage to try to answer as many questions as you can. Use your time effectively. Do not hurry,but work steadily and as quickly as you can without sacrificing your accuracy. If a question seemstoo difficult, go on to the next one.

YOU ARE TO INDICATE YOUR ANSWERS TO ALL QUESTIONS ON THE SEPARATEANSWER SHEET. No credit will be given for anything written in the test booklet. After you havedecided which of the suggested answers you want to give for a question, blacken the correspondingspace on the answer sheet.

Example: Sample AnswerWhich of the following is the capitalof the United States? A B C D

(A) New York, NY(B) Houston, TX(C) Washington, DC(D) Chicago, IL

Give only one answer to each question; multiple answers will not be counted. If you wish to changean answer, erase your first mark completely and mark your new choice.

SampleMBEII.qxp 1/29/2010 10:05 AM Page 102